www.recruitment.guru

SBI P.O. Held on: 18-04-10 So lved Paper

REASONING ABILITY (1)2 (2)4 (3)5 (4)6 (5)None of these 1.In a certain code language ‘how many goals 6.Pratap corrrectly remembers that his mother’s scored’ is written as ‘5 3 9 7’; ‘many more birthday is before twenty third April but after matches’ is written as ‘9 8 2’ and ‘he scored five’ Nineteenth April, whert as his sister correctly is written as ‘1 6 3’. How is ‘goals’ written in remembers that their mother’s birthday is not on that code language ? or after twenty second April. On which day in (1)5 (2)7 (3)5 or 7 April is definitely their mother’s birthday ? (4)Data inadequate (1)Twentieth (2)Twenty-first (5)None of these (3)Twentieth or twenty-first 2.In a certain code TEMPORAL is written as (4)Cannot be determined OLDSMBSP. How is CONSIDER written in that (5)None of these code? 7.Ashok started walking towards South. After (1)RMNBSFEJ (2)BNMRSFEJ walking 50 meters he took a right turn and (3)RMNBJEFS (4)TOPDQDCH walked 30 meters. He then took a right turn and (5)None of these walked 100 meters. He again took a right turn 3.How many meaningful English words can be and walked 30 meters and stopped. How far and made with the letters DLEI using each letter only in which direction was he from the starting point? once in each word ? (1)50 meters South (2)150 meters North (1)None (2)One (3)Two (4)Three (5)More than three (3)180 meters East (4)50 meters North (5)None of these 4.Among A, B, C, D and E each having different ‘ ’ ‘ ’ weight, D is heavier than only A and C is lighter 8.If’–‘ means ‘+’; ‘–’means × ; × means ‘÷’and ‘ than B and E. Who among them is the heaviest ? +’means ‘–’; then 15 – 8 × 6 + 12 + 4 = ? (1)B (2)E (3)C 4 (1)20 (2)28 (3) 8 (4)Data inadequate 7 (5)None of these (4)2 3 (5)None of these 5.Each odd digit in the number 5263187 is 9.Town D is towards East of town F. Town B is substituted by the next higher digit and each even towards North of town D. Town H is towards digit is substituted by the previous lower digit South of town B. Towards which direction is and the digits so obtained are rearranged in town H from town F ? ascending order, which of the following will be (1)East (2)South-East , the third digit from the left end after the (3)North-East (4)Data inadequate rearrangement ? (5)None of these

The information provided here is only for refrence. It may vary from the original. www.recruitment.guru

10.How many such pairs of letters are there in the (1)Only I and III follow word SEARCHES each of which has as many (2)Only I, II and III follow letters between them in the word as in the English (3)Only II, III and IV follow alphabet ? (4)All I, II, III and IV follow (1)None (2)One (3)Two (5)None of these (4)Three (5)More than three 14. Statements: Some benches are walls. All walls Direction: In each of the questions below are given are houses. Some houses are jungles. All jungles four statements followed by four conclusions are roads. numbered I, II, III 8s IV. You have to take the given Conclusions: statements to be true even if they seem to be at given I.Some roads are benches. conclusions logically follows from the given II.Some jungles are walls. statements disregarding commonly known facts. III.Some houses are benches. IV.Some roads are houses. 11. Statements: All cups are bottles. Some bottles (1)Only land II follow are jugs. No jug is plate. Some plates are tables. (2)Only I and III follow Conclusions: (3)Only III and IV follow I.Some tables are bottles. (4)Only II, III and IV follow II.Some plates are cups. (5)None of these III.No table is bottle. 15. Statements: Some sticks are lamps. Some IV.Some jugs are cups. flowers are lamps. Some lamps are dresses. All (1)Only I follows (2)Only II follows dresses are shirts. (3)Only III follows (4)Only IV follows Conclusions: (5)Only either I or III follows I.Some shirts are sticks. 12. Statements: Some chairs are handles. All II.Some shirts are flowers. handles are pots. All pots are mats. Some mats III.Some flowers are sticks. are buses. IV.Some dresses are sticks. Conclusions: (1)None follows (2)Only I follows I.Some buses are handles. (3)Only II follows (4)Only III follows II.Some mats are chairs. (5)Only IV follows III.No bus is handle. IV.Some mats are handles. Direction: Study the following information (1)Only I, II and IV follow carefully and answer the questions given below: (2)Only II, III and IV follow A, B, C, D, E, F, G and H are eight employees (3)Only either I or III and II follow of an organization working in three departments viz. (4)Only either I or III and IV follow Personnel, Administration and Marketing with not (5)Only either I or III and II and IV follow more than three of them in any department. Each of 13. Statements: All birds are horses. All horses are them has a different choice of sports from football, tigers. Some tigers are lions. Some lions are cricket, volleyball, badminton, lawn tennis, basketball, monkeys. hockey and table tennis not necessarily in the same Conclusions: order. I.Some tigers are horses. D works in Administration and does not like II.Some monkeys are birds. either football or cricket. F works in Personnel with III.Some tigers are birds. only A who likes table tennis. Eand H do not work in IV.Some monkeys are horses. the same department as D. C likes hockey and does

The information provided here is only for refrence. It may vary from the original. www.recruitment.guru

not work in marketing. G does not work in them is/are definitely true and give your answer administration and does not like either cricket or accordingly. badminton. One of those who work in administration 21. Statements: H @ T, T # F, F δ E, E VV likes football. The one who likes volleyball works in personnel. None of those who work in administration Conclusions: I. V $ F II. E @ T likes either badminton or lawn tennis. H does not like III. H @ V IV. T # V cricket. (1)Only I, II and III are true (2)Only I, II and IV are true 16.Which of the following groups of employees (3)Only II, III and IV are true work in Administration department ? (4)Only I, III and IV are true (1)EGH (2)AF (3)BCD (5)All I, II, III and IV are true (4)BGD (5)Data inadequate 17.In which department does E work ? 22. Statements: D#R, R K, K@F, F$J (1)Personnel (2)Marketing Conclusions: I. J # R II. J # K (3)Administration (4)Data inadequate III. R # F IV. K @ D (5)None of these (1)Only I, II and III are true 18.Which of the following combinations of (2)Only II, III and IV are true employee-department-favourite sport is correct? (3)Only I, III and IV are true (1)E – Administration – Cricket (4)All I, II, III and IV are true (2)F – Personnel – Lawn Tennis (5)None of these (3)H – Marketing – Lawn Tennis 23. Statements: N δ B, B $ W, W # H, H M (4)B – Administration – Table Tennis Conclusions : I. M @ W II. H @ N (5)None of these III. W S N IV. W# N 19.What is E’s favourite sport ? (1)Only I is true (2)Only III is true (1)Cricket (2)Badminton (3)Only IV is true (3)Basketball (4)Lawn Tennis (4)Only either III or IV is true (5)None of these (5)Only either III or IV and I are true 20.What is G’s favourite sport ? 24. Statements: R D, D $ J, J # M, M @ K (1)Cricket (2)Badminton Conclusions: I. K # J II. D @ M (3)Basketball (4)Lawn Tennis III. R # M (5)None of these IV. D @ K Direction: In the following questions, the symbols (1)None is true (2)Only I is true # and S are used with the following meaning as (3)Only II is true (4)Only III is true illustrated below. (5)Only IV is true P $ Q’ means ‘P is not smaller than Q’. 25. Statements: M $ K, K @ N, N R, R # WW P @ Q’ means ‘P is neither smaller than nor equal to ‘Q Conclusions: I. W @ K II. M $ R ‘P # Q’ means ‘P is neither greater than nor equal to Q’. III. K @ W P δ Q’ means ‘P is neither greater than nor smaller than IV. M @ N Q’. (1)Only I and II are true (2)Only I, II and III are true ‘P Q’ means ‘P is not greater than Q’. (3)Only III and IV are true Now in each of the following questions (4)Only II, III and IV are true assuming the given statements to be true, find which (5)None of these of the four conclusions I, II, III and IV given below

The information provided here is only for refrence. It may vary from the original. www.recruitment.guru

Direction: Study the following information eight years out of which four years as Manager- carefully and answer the questions given below Credit in a bank after completing her B. A. degree with 60 percent marks. She was born on 12th Following are the conditions for selecting Senior September 1978. Manager-Credit in a bank. The candidate must— 27.Rohan Maskare was born on 8th March 1974. He (i)be a graduate in any discipline with atleast 60 has been working in a bank for the past twelve years percent marks. after completing his B.Com. degree with 70 percent (ii)have post qualification work experience of at marks. He has secured 50 percent marks in both least ten years in the Advances Section of a bank. the Group Discussion and the Interview. (iii)be at least 30 years and not more than 40 years 28.Prakash Gokhale was born on 4th August 1977. as on 1.4.2010. (iv) have secured at least 40 He has secured 65 percent marks in. post percent marks in the group discussion. (v) have graduation and 58 percent marks in graduation. secured at least 50 percent marks in interview. He has been working for the past ten years in In the case of a candidate who satisfies all the the Advances Department of a bank after conditions EXCEPT— completing his post graduation. He has secured (A)at (i) above but has secured at least 50 percent 45 percent marks in the Group Discussion and marks in graduation and at least 60 percent marks 50 percent marks in the Interview. in post graduation in any discipline the case is 29.Sudha Mehrotra has been working in the to be referred to the General Manager Advances. Advances department of a bank for the past (B)at (ii) above but has total post qualification work twelve years after completing her B.Com. degree experience of at least seven years out of which with 60 percent marks. She has secured 50 at least three years as Manager-Credit in a bank, percent marks in the Group Discussion and 40 the case is to be referred to Executive percent marks in the Interview. She was born on Director. 15th February 1972. In each question below details of one candidate 30.Amit Narayan was born on 28th May 1974. He is given. You have to take one of the following has been working in the Advances department of a courses of action based on the information bank for the past eleven years after completing his provided and the conditions and sub-conditions B.Sc. degree with 65 percent marks. He has secured given above and mark the number of that course 55 percent marks in the Group discussion and 50 of action as your answer. You are not to assume percent marks in the interview. anything other than the information provided in Direcion: In each question below is given a each question. All these cases are given to you statement followed by three courses of action as on 01.04.2010. numbered (A), (B) and (C). A course of action is a Give answer step or administrative decision to be taken for. (1)if the case is to be referred to Executive improvement, follow-up or further action in regard Director. to the problem, policy, etc. On the basis of the (2)if the case is to be referred to General information given in the statement, you have to Manager-Advances. assume everything in the statement to be true, then (3)if the data are inadequate to take a decision. decide which of the suggested courses of action (4)if the candidate is not to be selected. logically follow(s) for pursuing. (5)if the candidate is to be selected. 26.Shobha Gupta has secured 50 percent marks in 31. Statement : A heavy unseasonal downpour the Interview and 40 percent marks in the Group during tho last two days has paralysed the, Discussion. She has been working for the past normal life in the state in which five persons were

The information provided here is only for refrence. It may vary from the original. www.recruitment.guru

killed but this has provided a huge relief to the (3)Only (C) (4)Only (A) and (B) problem of acute water crisis in the state. (5)None of these Courses of action: 34. Statement: The school dropout rate in many (A)The state government should set up a districts in the state has increased sharply during committee to review the alarming situation. the last few years as the parents of these children (B)The state government should immediately make them work in the fields owned by others remove all the restrictions, on use of potable to earn enough for them to get at least one meal water in all the major cities in the state. a day. (C)The state government should send relief Courses of action: supplies to all the affected areas in the state. (A)The Govt. should put up a mechanism to (1)None (2)Only (A) provide foodgrains to the poor people in these (3)Only (B) and (C) (4)Only (C) districts through public distribution system (5)All (A), (B), (C) to encourage the parents to send their wards 32. Statement: A large private bank has decided to to school. retrench one-third of its employees in view of (B)The Govt. should close down some of these the huge losses incurred by it during the past schools in the district and deploy the teachers three quarters. of these schools to nearby schools and also Courses of action: ask remaining students to join these schools. (A)The Govt. should issue a notification to (C)Govt. should issue arrest warrants for all the general public to immediately: stop all parents who force their children to work in transactions with the bank. fields instead of attending classes. (B)The Govt. should direct the bank to refrain (1)Only (A) (2)Only (B) from retrenching its employees. (3)Only (C) (4)Only (A) and (B) (C)The Govt. should ask the central bank of the (5)None of these country to initiate an enquiry into the bank’s 35. Statement: One aspirant was killed due to activities and submit its report. stampede while participating in a recruitment (1)None (2)Only (A) drive of police constables. (3)Only (B) (4)Only (C) Courses of action: (5)Only (A) and (C) (A)The officials incharge of the recruitment 33. Statement: Many political activists have process should immediately be suspended. decided to stage demonstrations and block traffic (B)A team of officials should be asked to find movement in the city during peak hours to protest out the circumstances which led to the death against the steep rise in prices of essential of the aspirant and submit its report within a commodities. week. Courses of action: (C)The Govt. should ask the home department (A)The Govt. should immediately ban all forms to stagger the number of aspirants over more of agitations in the country. number of days to avoid such incidents in (B)The police authority of the city should deploy future. additional forces all over the city to help (1)Only (A) (2)Only (B) traffic movement in the city. (3)Only (C) (4)Only (B) and (C) (C)The state administration should carry out (5)None of these preventive arrests of the known criminals 36. Effect: Majority of the employees of the ailing staying in the city. organization opted for voluntary retirement (1)Only (A) (2)Only (B) scheme and left the organization with all their

The information provided here is only for refrence. It may vary from the original. www.recruitment.guru

retirement benefits within a fortnight of 39. Statement : The Govt. has decided to instruct launching the scheme. the banks to open new branches in such a way Which of the following can be a probable cause that there is one branch of any of the banks in of the above effect ? every village of population 1000 and above or a (1)The company has been making huge losses cluster of villages with population less than 1000 for the past five years and is unable to pay to provide banking services to all the citizens. salaryto its employees in time. Which of the following will weaken the step (2)The management of the company made huge taken by the Govt. ? personal gains through unlawful activities. (1)The private sector banks in India have (3)One of the competitors of the company went stepped up their branch expansion activities bankrupt last year. in rural India. (4)The company owns large tracts of land in the (2)Many Govt. owned banks have surplus state which will fetch huge sum to its owners. manpower in its urban branches. (5)None of these (3)All the banks including those in private sector 37. Statement: Most of the companies in IT and will follow the govt. directive. ITES sectors in India have started hiring from (4)Large number of branches of many Govt. engineering college campuses this year and are owned banks in the rural areas are making likely to recruit much more than yearly huge losses every year due to lack of adequate recruitment of the earlier years. business activities. Which of the following substantiates the facts (5)None of these stated in the above statement ? Direction: Study the following information (1)IT and ITES are the only sectors in India carefully and answer the questions given below. which are hiring from engineering college The centre reportedly wants to continue campuses. providing subsidy to consumers for cooking gas and (2)Govt. has stepped up recruitment activities kerosene for five more years. This is not good news after a gap of five years. from the point of view of reining in the fiscal deficit. (3)The IT and ITES companies have now Mounting subventions for subsidies means diversion decided to vi sit the engineering college of savings by the government from investment to campuses for tier II cities in India as well. consumption, raising the cost of capital in the process. (4)Availability of qualified engineers will The government must cut expenditure on subsidies substantially in crease in the near future. to create more fiscal space for investments in both (5)None of these physical and social infrastructure. It should outline a 38. Cause: The Govt. has recently increased its taxes plan for comprehensive reform in major subsidies or, petrol and diesel by about 10 percent. Which including petroleum, food and fertilizers and set goal of the following can be a possible effect of the posts. above cause ? (1)The petroleum companies will reduce the 40.Which of the following is a conclusion which prices of petrol and diesel by about 10 can be drawn from the facts stated in the above percent. paragraph ? (3)The petroleum companies will increase the (1)Subsidy provided by the government under prices of petrol and diesel by about 5 percent. various heads to the citizen increases the cost (4)The petrol pumps will stop selling petrol and of capital. diesel till the taxes are rolled back by the govt. (2)Govt. is unable to withdraw subsidies (5)None of these provided to various items.

The information provided here is only for refrence. It may vary from the original. www.recruitment.guru

(3)Govt. subsidy on kerosene is purely a crucial dynamism. In a study conducted on 13,000 political decision. households which represented the crucial dynamism. (4)Govt. does not have enough resources to In a study conducted on 13,000 households which continue providing subsidy on petroleum represented the entire country in 1993-94 and again products. on 2004-05, it was found that in the ten-year period (5)None of these 18.2% rural population moved out of poverty whereas 41.Which of the following is an inference which another 22.1 % fell into it over this period. This net can be made from the facts stated in the above increase of about four percentage points was seen to paragraph ? have a considerable variation across states and (1)India’s fiscal deficit is negligible in regions. (omparison to other emerging economies in 43.Which of the following is a conclusion which the world. can be drawn from the facts stated in the above (2)Subsidy on food and fertilizers are essential paragraph ? for growth of Indian economy. (1)Accurate estimates of number of people (3)Reform in financial sector will weaken living below poverty line in India is possible India’s position in the international arena. to be made. (4)Gradual withdrawal of subsidy is essential (2)Many expert groups in India are not for effectively managing fiscal deficit in interested to measure poverty objectively. India. (3)Process of poverty measurement needs to (5)None of these take into account various factors to tackle its 42.Which of the following is an assumption which dynamic nature. is implicit in the facts stated in the above (4)People living below poverty line remain in paragraph ? that position for a very long time. (1)People in India may not be able to pay more (5)None of these for petroleum products. 44.Which of the following is an assumption which (2)Many people in India are rich enough to buy is implicit in the facts stated in the above petroleum products at market cost. paragraph ? (3)Govt. may not be able to create more (1)It may not be possible to have an accurate infrastructural facilities if the present level poverty measurement in India. of subsidy continues for a longer time. (2)Level of poverty in India is static over the (4)Govt. of India has sought assistance from years. international financial organizations for its (3)Researchers avoid making conclusions on infrastructural projects poverty measurement data in India. (5)None of these (4)Govt. of India has a mechanism to measure Direcion: Study the following information carefully level of poverty effectively and accurately. and answer the questions given below. (5)None of these 45.Which of the following is an inference which Poverty measurement is an unsettled issue, both can be made from the facts stated in the above conceptually and methodologically. Since poverty is paragraph ? a process as well as an outcome; many come out of it (1)Poverty measurement tools in India are while others may be falling into it. The net effect of outdated. these two parallel processes is a proportion commonly (2)Increase in number of persons falling into identified as the ‘head count ratio’, but these ratios poverty varies considerably across the hide the fundamental dynamism that characterizes country over a period of time. poverty in practice. The most recent poverty (3)Govt. of India has stopped measuring poverty reestimates by an expert group has also missed the related studies.

The information provided here is only for refrence. It may vary from the original. www.recruitment.guru

(4)People living in rural areas are more 48.Problem Figure

susceptible to fall into poverty over the time. A A A T (5)None of these A Z O T T A Direction: In each of the questions given below T T Answer Figure which one of the five answer figures on the right should come after the problem figures on the left, if the sequence were continued ? 46.Problem Figure 1 2 3 4 5 49.Problem Figure A K A K OOOOK OO O Answer Figure A KK A A Answer Figure A A OO O O O A O KKKK O O 1 2 3 4 5 A A K 47.Problem Figure 1 2 3 4 5 50.Problem Figure D D Z Z O Z O

Z D O D Answer Figure O Z O D Answer Figure O O O Z O

Z Z D Z O Z D D 1 2 3 4 5 D D 1 2 3 4 5

DATA ANALYSIS AND INTERPRETATION

Direction: Study the following table carefully to answer the questions that follow Number (n) of candidates (in lakhs) appearing for an entrance examination From six different states and the percentage (p) of candidates clearing the same over the years STATE A B C D E F YEAR N P N P N P N P N P N P 2004 1.23 42 1.04 51 1.11 32 1.32 24 1.23 36 1.33 31 2005 1.05 43 1.12 62 1.07 47 1.15 49 1.18 55 1.24 24 2006 2.04 38 1.48 32 1.08 28 1.96 35 1.42 49 1.58 26 2007 1.98 41 2.07 43 1.19 30 1.88 46 1.36 47 1.79 29 2008 1.66 53 1.81 50 1.56 42 1.83 60 1.73 57 1.86 34 2009 1.57 39 1.73 36 1.64 52 2.01 56 1.69 55 1.95 37

The information provided here is only for refrence. It may vary from the original. www.recruitment.guru

51.What is the respective ratio of total number of 57.If two marbles are picked at random, what is the candidates clearing the entrance exam from State probability that both are red ? B in the year 2004 to those clearing the entrance 1 1 2 exam from State C in the same year (1) (2) (3) (1)221: 148 (2)218: 143 6 3 15 (3)148: 221 (4)143: 218 2 (4) (5)None of these (5)None of these 5 52.In which year did the highest number of candidates 58.If three marbles are picked at random, what is clear the entrance exam from State D ? the probability that two are blue and one is (1)2008 (2)2006 yellow? (3)2009 (4)2007 (5)None of these 3 1 18 (1) (2) (3) 53.What is the number of candidates not clearing 91 5 455 the entrance exam from State A in the year 2007? 7 (1)186820 (2) .11682 (4) (5)None of these (3)1868200 (4)1.16820 15 (5)None of these 59.If four marbles are picked at random, what is 54.What is the total numberof candidates clearing the probability that one is green, two are blue the entrance exam from States E and F together and one is red ? in the year 2006 ? 24 13 11 (1) (2) (3) (1)16160 (2)110660 455 35 15 (3)1.1066 (4)1106600 (5)None of these 7 (4) (5)None of these 55.What is the average number of candidates 91 appearing for the entrance exam from State D in 60.If two marbles are picked at random, what is the the years 2007, 2008 and 2009 together ? probability that either both are green or both are 2 1 yellow ? (1) 1.907 (2) 18666 3 3 5 1 1 (1) (2) (3) 91 35 3 1 2 (3) 1.866 (4) 190666 3 3 4 (4) (5)None of these (5)None of these 105 Direction: Study the given information carefully Direction: Study the given pie-charts carefully to and answer the questions that follow answer the questions that follow An urn contains 6 red, 4 blue, 2 green and 3 Breakup of number of employees working yellow marbles. in different departments of an organisation, 56.If four marbles are picked at random, what is the, number of males and the number of the probability that at least one is blue ? employees who recently got promoted in each department break-up of employees 4 69 11 (1) (2) (3) working in different departments total 15 91 15 number of employees = 3,600 Employees 22 working in different departments: (4) (5)None of these 91

The information provided here is only for refrence. It may vary from the original. www.recruitment.guru

62. What is the total number of females working in the Production and Marketing departments Accounts 20% together ? Production (1)468 (2)812 (3)582 35% Marketing (4)972 (5)None of these 18% 63.How many females work in the Accounts IT HR department ? 15% 12% (1)618 (2)592 (3)566 (4)624 (5)None of these Break-up of number of males in each 64.The total number of employees who got department promoted from all the departments together was Total number of males in the organisation what percent of the total number of employees = 2,040 working in all the departments together ? (Rounded off to the nearest integer) Break-up of number of males working in (1)56 (2)21 (3)45 each department (4)33 (5)51

% 65.The number of employees who got promoted 5

M .

a s from the HR department was what percent of r t k c

1 e c 5 t the total number of employees working in that % in A g department ? (rounded off to two digits after Production HR 10% 50% decimal) (1)36.18 (2)30.56 (3)47.22 IT 20% (4)28.16 (5)None of these Direction: Study the graph carefully to answer the Break-up of number of employees who questions that follow recently got promoted in each department Percent rise in profit of two companies over the years Total number of employees who got 40 promoted = 1,200 35 t

n 30

Number of employees who recently got e c r

promoted from each department e 25 P

t

i 20 t f n o

r

u

% 15

P o

8 c

c 10 A Marketing Production 5 33% 22% 0 % 1 2004 2005 2006 2007 2008 2009 1 R H IT 26% Company L Company M 66.If the profit earned by Company L in the year 61.If half of the number of employees who got 2005 was Rs. 1.84 lakhs, what was the profit promoted from the IT department were males, earned by the company in the year 2006 ? what was the approximate percentage of males (1)Rs. 2.12 lakhs (2)Rs. 2.3 lakhs who got promoted from the IT department ? (3)Rs. 2.04 lakhs (1)61 (2)29 (3)54 (4)Cannot be determined (4)42 (5)38 (5)None of these

The information provided here is only for refrence. It may vary from the original. www.recruitment.guru

67.If the profit earned by Company M in the year classes is two hundred percent of the boys enrolled 2008 was Rs. 3.63 lakhs, what was he amount in the same. One-thirteenth of the boys are enrolled of profit earned by it in the year 2006 ? in all the three classes together. The respective ratio (1)Rs. 2.16 lakhs (2)Rs.1.98 lakhs of boys enrolled in Dancing and Painting classes (3)Rs. 2.42 lakhs together to the girls enrolled in the same is 2 :1 (4)Cannot be determined respectively. Ten percent of the girls are enrolled in (5)None of these only Dancing classes whereas eight percent of the girls 68.What is the average percent rise in profit of are enrolled in both Dancing and Painting classes Company L over all the years together ? together. The remaining girls are enrolled in all the three classes together. The number of boys enrolled 1 1 5 (1) 15 (2) 25 (3) 18 in Singing and Dancing classes together is fifty 3 3 6 percent of the number of girls enrolled in the same. 5 The remaining boys are enrolled in only Painting (4) 21 (5)None of these classes. 6 69.Which of the following statements is TRUE with 71.What is the total number of boys who are respect to the above graph ? enrolled in Dancing ? (1)Company M made the highest profit in the (1)318 (2)364 year 2009 (3)292 (4)434 (2)Company L made least profit in the year 2005 (5)None of these (3)The respective ratio between the profits 72.Total number of girls enrolled in Singing is earned by Company L and M In the year 2006 approximately what percent of the total number was 6:5 of students in the school ? (4)Company L made the highest profit in the (1)37 (2)19 year 2008 (3)32 (4)14 (5)All are true (5)26 70.What is the percentage increase in percent rise 73.What is the total number of students enrolled in in profit of Company M in the year 2009 from all the three classes together ? the previour year ? (1)135 (2)164 (1)25 (2)15 (3)50 (3)187 (4)142 (4)75 (5)None of these (5)None of these 74.Number of girls enrolled in only Dancing classes Direction: Study th a information carefully to is what percent of the boys enrolled in the same? answer the questions that follow. (rounded off to two digits after decimal) A school consisting of a total of 1560 students (1)38.67 (2)35.71 has boys and girls in the ratio of 7:5 respectively. All (3)41.83 (4)28.62 the students are enrolled in different types of hobby (5)None of these classes, viz: Singing, Dancing and Painting. One-fifth 75.What is the respective ratio of the number of of the boys are enrolled in only Dancing classes. girls enrolled in only Painting classes to the Twenty percent of the girls are enrolled in only number of boys enrolled in the same ? Painting classes. Ten percent of the boys are enrolled (1)77 : 26 (2)21 : 73 in only Singing classes. Twenty four percent of the (3)26 : 77 (4)73 : 21 girls are enrolled in both Singing and Dancing classes (5)None of these together. The number of girls enrolled in only Singing

The information provided here is only for refrence. It may vary from the original. www.recruitment.guru

Direction: Study the table carefully to answer the questions that follow. Profit (in rs. ‘000) made by six different shopkeepers over the months Month October November December January February March Shopkeeper 2009 2009 2009 2010 2010 2010 P 5.25 6.04 5.84 6.10 5.95 6.02 Q 4.84 4.28 4.97 4.88 5.04 5.12 R 4.99 5.82 5.48 5.45 5.68 5.36 S 5.06 5.11 5.28 5.38 5.44 5.59 T 5.28 4.96 5.31 5.69 4.93 5.72 U 5.94 6.23 5.87 6.07 6.19 6.23

76.What is the respective ratio between the profit Number of days taken by three carpenters to earned by shopkeeper U in the months finish making one piece each of four different February-2010 and March - 2010 together to that items of furniture

earned by shopkeeper Q in the same months ? 16 (1)637: 512 (2)621 : 508 14 (3)512: 637 (4)508: 621 12 (5)None of these 10 77.What is the percent increase in profit of 8 shopkeeper S in the month of December - 2009 6 over the previous month ? (rounded off to two 4 digits after decimal) 2 (1)3.15 (2)2.67 0 (3)2.18 (4)3.33 Chair Table Bed Cupboard (5)None of these 78.Which shopkeeper’s profit kept increasing Company X Company Y Company Z continuously over the given months ? 81.If Carpenter X and Carpenter Y were to make a (1)R (2)Q (3)T chair together how many days would they take? (4)U (5)None of these (1)1 day (2)4 days (3)3 days 79.What is the difference in profit earned by (4)2 days (5)None of these shopkeeper T in January - 2010 from the 82.If Carpenters X, Y and Z were to make a table previous month ? together how many days would they take ? (1)Rs. 640/- (2)Rs. 420/- (1)4 days (2)3 days (3)1 day (3)Rs. 380/- (4)Rs. 760/- (4)2 days (5)None of these (5)None of these 83.What is the total number of days that Carpenter 80.What was the average profit earned by Z will take to make one piece each of all the shopkeeper R in the months of October – 2009 four items together ? and November – 2009 together ? (1)32 days (2)24 days (1)5405 (2)5040 (3)4825 1 1 (4)4950 (5)None of these (3) 1 days (4) 1 days 59 32 Direction: Study the given graph carefully to answer (5)None of these the questions that follow

The information provided here is only for refrence. It may vary from the original. www.recruitment.guru

84.The radius of a circular field is equal to the side 85.In how many different ways can the letters of of a square field whose perimeter is 784 feet. the word ‘STRESS’ be arranged What is the area of the circular field ? (1)360 (2)240 (1)107914 Sq.ft (2)120736 Sq.ft. (3)720 (3)107362 Sq.ft. (4)127306 Sq.ft. (4)120 (5)None of these (5)None of these

Direction: Study the given table carefully to answer the questions that follow Number of people staying in five different localities and the percentage breakup of men, women and children in them LOCALITY TOTAL NO. PERCENTAGE OF PEOPLE MEN WOMEN CHILDREN F 5640 55 35 10 G 4850 34 44 22 H 5200 48 39 13 I 6020 65 25 10 J 4900 42 41 17 86.Total number of people staying in locality J forms amount that Suresh got back at the end of two approximately what percent of the total number years in the form of principal plus interest of people staying in locality F ? earned? (1)81 (2)72 (3)78 (1)Rs. 9,414.4 (2)Rs. 9,914.4 (4)93 (5)87 (3)Rs. 9,014.4 (4)Rs. 8,914.4 87.What is the total number of children staying in (5)None of these localities H and I together ? 92.The respective ratio of the present ages of a (1)1287 (2)1278 (3)1827 mother and daughter is 7 : 1. Four years ago the (4)1728 (5)None of these respective ratio of their ages was 19:1. What will be the mother’s age four years from now ? 88.The number of women staying in which locality (1)42 years (2)38 years is the highest ? (3)46 years (4)36 years (1)H (2)J (3)F (5)None of these (4)G (5)None of these 93.Three friends J, K and Ljog around a circular 89.What is the total number of men and children stadium and complete one round in 12, 18 and staying in locality I together 20 seconds respectively. In how many minutes (1)4115 (2)4551 (3)4515 will all the three meet again at the starting point (4)4155 (5)None of these (1)5 (2)8 (3)12 90.What is the respective ratio of number of men (4)3 (5)None of these staying in locality F to the number of men staying 94.4 men can complete a piece of work in 2 days. 4 in locality H ? women can complete the same piece of work in (1)517: 416 (2)403: 522 4 days whereas 5 children can complete the same (3)416: 517 (4)522: 403 piece of work in 4 days. If, 2 men, 4 women (5)None of these anW 10 children work together, in how many 91.The compound interest earned by Suresh on a days can the work be completed ? certain amount at the end of two years at the rate (1)1 day (2)3 days (3)2 days of 8 p.c.p.a was Rs. 1,414.4. What was the total (4)4 days (5)None of these

The information provided here is only for refrence. It may vary from the original. www.recruitment.guru

95.The speed of a boat when travelling downstream 97.In which of the following years was the number is 32 Kms. / Hr. , whereas when travelling of candidates qualified from , the upstream it is 28 kms/hr. What is the speed of maximum among the given years ? the boat in still water ? (1)2007 (2)2006 (3)2005 (1)27 Kms./Hr. (2)29 Kms./ Hr. (4)2003 (5)None of these (3)31 Kms./ Hr. 98.Approximately what was the total number of (4)Cannot be determined candidates qualified from Delhi in 2002 and (5)None of these 2006 together ? Direction: Study the following tables carefully and (1)27250 (2)25230 (3)30150 answer the questions given below (4)28150 (5)26250 99.Approximately how many candidates appearing Number of Candidates appeared in a from Kolkata in 2004 qualified in the CompetitiveExamination from five centres over competitive examination ? the years (1)13230 (2)13540 (3)15130 NUMBER (4)15400 (5)19240 CenterMumbai DelhiKolkataHyderaChennai 100.Approximately what was the difference between Year -bad the number of candidates qualified from 2001 3514565139 45192 51124 37346 Hyderabad in 2001 and 2002 ? 2002 1726458248 52314 50248 48932 (1)1680 (2)2440 (3)1450 (4)2060 (5)1860 2003 2480063309 56469 52368 51406 2004 2831670316 71253 54196 52315 GENERAL AWARENESS / 2005 3650369294 69632 58350 55492 MARKETING / COMPUTERS 2006 2912959216 64178 48230 57365 2007 3243861345563041 49178 58492 101.What is the full form of ‘NBFC’ as used in the Approximate-Percentages of candidatesqualified Fir ancial Sector ? to appeared in the-Competitiveexamination from (1)New Banking Finance Company five centres over the years (2)National Banking & Finance Corporation PERCENTAGE (3)New Business Finance & Credit Mumbai DelhiKolkataHyderabadChennai (4)Non Business Fund Company (5)None of these 12 24 18 17 9 10 28 12 21 12 102.100% concession has been given for travelling in the Indian Railways for patients of ... 15 21 23 25 10 (1)AIDS (2)Cancer (3)Swine Flu 11 27 19 24 8 (4)T. B. (5)None of these 13 23 16 23 13 103.Many a times, we read about Special Drawing 14 20 21 19 11 Right (SDR) in newspapers. As per its definition, 16 19 24 20 14 SDR is a monetary unit of the reserve assets of 96.In which of the following years was the which of the following organizations / agencies? difference in number of candidates appeared (1)World Bank from Mumbai over the previous year the (2)International Monetary Fund (IMF) minimum ? (3)Asian Development Bank (1)2004 (2)2006 (3)2007 (4) (4)2002 (5)None of these (5)None of these

The information provided here is only for refrence. It may vary from the original. www.recruitment.guru

104.Which of the following is/are the highlights of 109.How much funds has been allocated to the the Union Budget 2010-11 ? Unique Identification Authority of India? (A)Number of new steps taken to simplify the (1)Rs 1,500 Crore (2)Rs 1,900 Crore Foreign Direct Investment (FDI) regime. (3)Rs 1,600 Crore (4)Rs 1,800 Crore (B)Rs. 16,500 crore provided to ensure that the (5)None of these Public Sector Banks are able to attain a 110.What is the rate of Income Tax for incomesz minimum 8% capital (Tier I) by March 2011. above Rs 1.6 lakh upto Rs 5 lakh? (C)More than Rs. 1,74,000 crore provided for (1)10% (2)12% (3)15% the development of the infrastructure in the (4)20% (5)None of these country. 111.Recently, India took part in “Nuclear New Build (1)Only A (2)Only B 2010 Conference” organized in - (3)Only C (4)All A, B & C (1)New Delhi (2)London (5)None of these (3)Paris (4)Hong Kong 105.In how many Routes special tourist trains called (5)None of these ‘Bharat Tirth’ is to start? 112.What is the reduction in Service Charges on e- (1)19 (2)16 (3)17 tickets? (4)18 (5)None of these (1)Sleeper Class Rs. 20 & AC Class Rs. 10 106.As per the newspaper reports, the Govt. of India (2)Sleeper Class Rs. 10 & AC Class Rs. 20 made an auction of the Third Generation (3)Sleeper Class Rs. 30 & AC Class Rs. 20 Spectrum (3G) recently. Which of the following (4)Sleeper Class Rs. 20 & AC Class Rs. 30 ministries was actively involved in the process a (5)None of these (1)Ministry of Heavy Industries 113.As we know, with the launch of Nano by Tatas, (2)Ministry of Science & Technology India has become favourite Small Car (3)Ministry of Commerce Destination of the world. Other than India, which (4)Ministry of Foreign Affairs of the following countries is also a popular (5)None of these destination of small cars ? 107.As per the recent announcement, the Govt. of (1)Britain (2)France India will provide an amount of Rs.48,000 crore (3)Germany (4)Thailand to develop Rural Infrastructure in the country. (5)None of these This planned development is being undertaken 114.Imports from China in the Year of 2008-09 in under which of the following schemes ? Rs... (1)Bharat Nirman (1)100,000 Crore (2)140,000 Crore (2)Indira Aawas Yojana (3)147,605 Crore (4)151,000 Crore (3)Backward Region Grant Fund (5)None of these (4)Drought Mitigation Fund 115.How many new teams have been added in IPL (5)None of these 2010? 108.Ladies special trains to be renamed with the (1)2 (2)1 (3)4 Name of ... (4)7 (5)None of these (1)Bharat Bhoomi Specials 116.Which countries have recently faced Tsunami (2)Sonia Gandhi Specials Waves? (3)Matri Bhoomi Specials (1)Japan & Chile (2)Sallie & Korea (4)Rajiv Gandhi Specials (3)China & Thailand(4)Japan & China (5)None of these (5)None of these

The information provided here is only for refrence. It may vary from the original. www.recruitment.guru

117.North Korea and USA decided to resume their 122.Which of the following groups of cricket teams peace talks after a gap of several months. Both was declared joint winner of the Col. C K Naidu the countries have a dispute over which of the Trophy for 2009 ? following issues ? (1) & Gujarat (1)Bailout package offered by USA (2)Maharashtra & Kerala (2)Membership of ASEAN to North Korea (3)Punjab & Delhi (3)Nuclear programme of North Korea (4)West Bengal & Maharashtra (4)Soaring relations of China with North Korea (5)None of these (5)None of these 123.Kaiane Aldorino who was crowned Miss World 118. Who amongst the following was the Chairperson 2009 is from which of the following countries? of the 13th Finance Commission which (1)Germany (2)Russia submitted its report to the President of India (3)Austria (4)Belgium recently ? (5)Gibraltar (1)Mr. M. V. Kamath(2)Dr. C. Rangarajan 124.Which of the following teams won the Davis Cup (3)Dr. D. Subbarao (4)Dr. Rakesh Mohan Tennis Finals 2009 ? (5)Dr. Vijay Kelkar (1)Germany (2)Spain 119.As per the news published in various (3)Russia (4)France newspapers, the RBI is considering the grant of (5)None of these licence to some new companies, particularly 125.Who amongst the following is the recipient of NBFCs to act as full-fledged banks. Which of the “CNN-IBN Indian of the Year” Award for the following will be considered NBFC ? 2009? (1)NABARD (1)Ratan Tata (2)A.R.Rahman (2)Life Insurance Corporation of India (3)Manmohan Singh(4)Sachin Tendulkar (3)Reliance Capital (5)None of these (4)SEBI 126.The deficit reduction plan of which of the (5)None of these following countries was reviewed recently in the 120.Who has scored the highest individual ODI meeting of the Finance Ministers of the European Score? Union ? (1)Saeed Anwar (Pak) (1)Germany (2)Romania (2)Charls Coventry (Zim) (3)Brazil (4)Hungary (3)Sachin Tendulkar (IND) (5)Greece (5)Ricky Pointing (Aus) 127.The database administrator’s function in an (5)None of these organization is — 121.What is an ‘intranet’ (1)To be responsible for the more technical (1)Internal internet used to transfer information aspects of managing the information internally contained in organizational databases (2)Internal internet used to transfer information (2)To be responsible for the executive level to the outside company aspects of decisions regarding the (3)Internal network designed to serve the information management internal informational needs of a single (3)To show the relationship among entity classes organization in a data warehouse (4)Internal network designed to transfer the (4)To define which data mining tools must be information between two organizations used to extract data (5)None of these (5)None of these

The information provided here is only for refrence. It may vary from the original. www.recruitment.guru

128.Every device on the Internet has a unique______133.A set of interrelated components that collect, address (also called an “Internet address”) that process, store, and distribute information to identifies it in the same way that a street address support decision making and control in an identifies the location of a house. organization best defines - (1)DH (2)DA (3)IP (1)communications technology (4)IA (5)None of these (2)a network 129.To send another station a message, the main thing (3)an information system a user has to know is (4)hardware (1)how the network works (5)None of these (2)the other station’s address 134.Ais a computer connected to two networks. (3)whether the network is packet-switched or (1)link (2)server circuit-switched (3)gateway (4)bridge way (4)whether this is a voice or data network (5)None of these (5)None of these 135.When you save a presentation, 130.In a client/ server model, a client orogram - (1)all slides in a presentation are saved in the (1)asks for information same file (2)provides information and files (2)two files are created; one for graphics and (3)serves software files to other computers one for content (4)distributes data files to other computers (3)a file is created for each slide (5)None of these (4)a file is created for each animation or graphic 131.Control in design of an information system is (5)None of these used to — 136.In a customer database, a customer’s surname (1)inspect the system and check that it is buiit would be keyed into a — as per specifications (1)row (2)text field (2)ensure that the system processes data as it (3)record (4)computed field was designed to and that the results are (5)None of these reliable 137.Who is the new Prime Minister of Hungry ? (3)ensure privacy of data processed by it (1)Victor Orban (2)Gorden Bajnai (4)protect data from accidental or intentional (3)Jeno Fock (4)Ference Gyurcsany loss (5)None of these (5)None of these 138.Storing same data in many places is called 132.Each of the following is a true statement (1)iteration (2)concurrency except— (3)redundancy (4)enumeration (1)on-line systems continually update the master (5)None of these file 139.Which of the following is the first step in the (2)in on-line processing, the user enters ‘transaction processing cycle’, which captures transactions into a device that is directly business data through various modes such as connected to the computer system optical scanning or at an electronic commerce (3)batch processing is still used today in older website ? systems or in some systems with massive (1)Document and report generation volumes of transactions (2)Database maintenance (4)information in batch systems will always be (3)Transaction processing up-to-date” (4)Data Entry (5)None of these (5)None of these

The information provided here is only for refrence. It may vary from the original. www.recruitment.guru

140.CRM (Customer Relationship Management) 147.______servers store and manages files for is— network users. (1)Apre-sales activity (1)Authentication (2)Main (2)A tool for lead generation (3)Web (4)File (3)An ongoing daily activity (5)None of these (4)The task of a DSA 148.One of the following is not included in the 7 P’s (5)All of the above of Marketing. Find the same 141.Who is the new prime minister of Denmark ? (1)Product (2)Price (1)Anders Fogh Rasmussen (3)Production (4)Promotion (2)Lars Looke Rasmussen (5)None of these (3)Poul Nyrup Rasmussen 149.The target group for SME loans is - (4)Poul Hartling (1)All Businessmen (2)All Professionals (5)None of these (3)All SSIs (4)All of the above 142.Who is the Author of the book “My China Diary” (5)None of these (1)Kanwal Sibal (2)Salman Haider 150.Home Loans can be best canvassed among - (3)J.N. Dixit (4)Natwar Singh (1)Builders (2)Flat owners (5)None of these (3)Land developers (4)Agriculturists 143.One of the following is not involved in the (5)Individuals wanting to buy a flat or house Growth Strategies of a Company - (1)Horizontal integration ENGLISH LANGUAGE (2)Vertical integration (3)Diversification Direction: Read the following passage carefully and (4)Intensification (5)None of these answer the questions given below it. Certain words/ 144.A successful “Blue Ocean Strategy” requires - phrases have been printec in bold to help you locate (1)Effective communication them while answering some of the questions. (2)Innovative skills Governments have traditionally equated (3)Motivation economic progress with steel mills and cement (4)All of the above (5)None of these factories. While urban centers thrive and city dwellers 145.Programs from the same developer , sold get rich, hundreds of millions of farmers remain mired bundled together , that provide better integration in poverty. However, fears of food shortages, a and share common features , toolbars and menus rethinking of antipoverty priorities and the crushing are known as .... recession in 2008 are causing a dramatic shift in world (1)software suites economic policy in favour of greater support for (2)integrated software packages agriculture. (3)software processing packages The last time when the world’s farmers felt such (4)personal information managers love was in the 1970s. At that time, as food prices (5)none of these spiked, there was real concern that the world was 146.A data warehouse is which of the following ? facing a crisis in which the planet was simply unable (1)Can be updated by the end users to produce enough grain and meat for an expanding (2)Contains numerous naming conventions and population. Governments across the developing world formats and international aid organisations plowed investment (3)Organized around important subject areas into agriculture in the early 1970s, while technological (4)Contains only current data breakthroughs, like high-yield strains of important (5)None of these

The information provided here is only for refrence. It may vary from the original. www.recruitment.guru

food crops, boosted production. The result was the In India on the other hand, with only 40% of its Green Revolution and food production exploded. farmland irrigated, entire economic boom currently But the Green Revolution became a victim of underway is held hostage by the unpredictable its own success. Food prices plunged by some 60% monsoon. With much of India’s farming areas by the late 1980s from their peak in the mid- 1970s. suffering from drought this year, the government will Policymakers and aid workers turned their attention have a tough time meeting its economic growth to the poor’s other pressing needs, such as health care targets. In a report, Goldman Sachs predicted that if and education. Farming got starved of resources and this year too receives weak rains, it could cause investment. By 2004, aid directed at agriculture sank agriculture to contract by 2% this fiscal year, making to 3.5% and “Agriculture lost its glitter.” Also, as the government’s 7% GDP-growth target look “a bit consumers in high-growth giants such as China and rich”. Another green revolution is the need of the hour India became wealthier, they began eating more meat, and to make it a reality, the global community still so grain once used for human consumption got has much backbreaking farm work to do. diverted to beef up livestock. By early 2008, panicked 151.What is the author’s main objective in writing buying by importing countries and restrictions slapped the passage on grain exports by some big producers helped drive (1)Criticising developed countries for not prices upto heights not seen for three decades. Making bolstering economic growth in poor nations matters worse, land and resources got reallocated to (2)Analysing the disadvantages of the Green produce cash crops such as biofuels and the result was Revolution that voluminous reserves of grain evaporated. Protests (3)Persuading experts that a strong economy broke out across the emerging world and fierce food depends on industrialization and not riots toppled governments. agriculture This spurred global leaders into action. This (4)Making a case for the international society made them aware that food security is one of the to engineer a second Green Revolution fundamental issues in the world that has to be dealt (5)Rationalising the faulty agriculture policies with in order to maintain administrative and political of emerging countries stability. This also spurred the U.S. which traditionally 152.Which of the following is an adverse impact of provisioned food aid from American grain surpluses the Green Revolution ? to help needy nations, to move towards investing in (1)Unchecked crop yields resulted in large tracts farm sectors around the globe to boost productivity. of land becoming barren This move helped countries become more productive (2)Withdrawal of fiscal impetus from agriculture for themselves and be in a better position to feed their to other sectors own people. (3)Farmers began soliciting government Africa, which missed out on the first Green subsidies for their produce Revolution due to poor policy and limited resources, (4)Farmers rioted as food prices fell so low that also witnessed a ‘change’. Swayed by the success of they could not make ends meet East Asia, the primary poverty-fighting method (5)None of these favoured by many policymakers in Africa was to get 153.What is the author trying to convey through the farmers off their farms and into modern jobs in phrase “making the government’s 7% GDP factories and urban centers. But that strategy proved growth target look “a bit rich” ? to be highly insufficient. Income levels in the (1)India is unlikely to achieve the targeted countryside badly trailed those in cities while the FAO growth rate estimated that the number of poor going hungry in (2)Allocation of funds to agriculture has raised 2009 reached an all time high at more than one billion. India’s chances of having a high GDP

The information provided here is only for refrence. It may vary from the original. www.recruitment.guru

(3)Agricultural growth has artificially inflated (4)To establish itself in the market before the India’s GDP and such growth is not real high-growth giants such as India and China (4)India is likely to rave one of the highest GDP could establish themselves growth rates (5)None of these (5)A large portion of India’s GDP is contributed 157.What impact did the economic recession of 2008 by agriculture have on agriculture ? 154.Which of the following factors was/were (1)Governments equated economic stability responsible for the neglect of the farming sector with industrial development and shifted away after the green revolution ? from agriculture (A)Steel and cement sectors generated more (2)Lack of implementation of several innovative revenue for the government as compared to agriculture programmes owing to shortage of agriculture. funds (B)Large scale protests against favouring (3)It prompted increased investment and interest agriculture at the cost of other important in agriculture sectors such as education and healthcare. (4)The GDP as targeted by India was never (C)Attention of policy makers and aid achieved because of losses in agriculture organizations was diverted from agriculture (5)None of these to other sectors. 158.What encouraged African policymakers to focus (1)None (2)Only (C) on urban jobs ? (3)Only (B) & (C) (4)Only (A) 8s (B) (1)Misapprehension that it would alleviate (5)All (A), (B) & (C) poverty as it did in other countries 155.What prompted leaders throughout the world to (2)Rural development outstripped urban take action to boost the agriculture sector in development in many parts of Africa 2008? (3)Breaking out of protests in the country and (1)Coercive tactics by the U.S. which restricted the fear that the government would topple food aid to poor nations (4)Blind imitation of western models of (2)The realization of the link between food development security and political stability (5)None of these (3)Awareness that performance in agriculture is 159.Which of the following had contributed to necessary in order to achieve the targeted exorbitant food prices in 2008 ? GDP (A)Hoarding of food stocks by local wholesalers (4)Reports that high-growth countries like China which inadvertently created a food shortage. and India were boosting their agriculture (B)Export of foodgrains was reduced by large sectors to capture the international markets producers. (5)Their desire to influence developing nations (C)Diverting resources from cultivation of to slow down their industrial development. foodgrains to that of more profitable crops. 156.What motivated the U.S. to focus on investing (1)None (2)Only (C) in agriculture across the globe ? (3)Only (B) (4)All (A), (B) & (C) (1)To make developing countries become more (5)Only (B) & (C) reliant on U.S. aid 160.Which of the following is true about the state of (2)To ensure grain surpluses so that the U.S. had agriculture in India at present ? no need to import food (A)Of all the sectors, agriculture needs the (3)To make those countries more self sufficient highest allocation of funds. to whom it previously provided food (B)Contribution of agriculture to India’s GDP

The information provided here is only for refrence. It may vary from the original. www.recruitment.guru

this year would depend greatly upon the (1)proved that such agitations result in loss of monsoon rains. governmental property (C)As India is one of the high-growth countries, (2)indicated that the formation of small states it has s jrplus food reserves to export to other does not necessarily improve the economy nations. (3)suggested that only large scale agitations have (1)Only (A) and (C) (2)Only (C) been effective in bringing out desired change (3)Only (B) (4)Only (B) and (C) in the past (5)None of these (4)recommended dividing large States into smaller ones to improve governance Direction: Choose the word/group of words which (5)None of these is most similar it meaning to the word printed in bold as used in the passage. 167.Overlooking the fact that water scarcity intensifies during summer, 161.STARVED (1)the government issued guidelines to all (1)Deprived (2)Disadvantaged builders to limit their consumption to (3)Hungry (4)Fasting acceptable limits (5)Emaciated (2)provision for rainwater harvesting has been 162.SLAPPED made to aid irrigation in drought prone areas (1)Beaten (2)Imposed (3)the water table did not improve even after (3)Withdrawn (4)Avoided receiving normal monsoon in the current year (5)Persuaded (4)many residential areas continue to use 163.PLOWED swimming pools, wasting large quantities of (1)Cultivated (2)Bulldozed water (3)Recovered (4)Instilled (5)None of these (5)Withdrew 168.He has lost most of his life’s earning in the stock market but Direction: Choose the word/phrase which is most (1)He still seems to be leading his life opposite in meaning to the word printed in bold as luxuriously and extravagantly used in the passage. (2)he could not save enough to repay his 164.PRESSING enormous debts (1)Unpopular (2)Undemanding (3)stock market is not a safe option to invest (3)Unobtrusive (4)Unsuitable money unless done with caution (5)Unimportant (4)experts have been suggesting to avoid 165. EVAPORATED investments in stock market because of its (1)Absorbed (2)Accelerated unpredictable nature (3)Grew (4)Plunged (5)None of these (5)Mismanaged 169.Achieving equality for women is not only a laudable goal, Direction: Which of the phrases (1), (2),(3) and (4) (1)political reforms are also neglected given below each statement should be placed in the preventing women from entering legislatures blank space provided so as to make a meaningful and positions of power and grammatically correct sentence ? If none of (2)the problem is also deep rooted in the society the sentences is appropriate, mark (5) i.e. ‘None of and supported by it these’ as the answer. (3)their empowerment is purposefully hampered 166.Refuting the rationale behind frequent agitations by people with vested interests in all sections for formation of separate States, a recent report of the society

The information provided here is only for refrence. It may vary from the original. www.recruitment.guru

(4)it is also equally difficult to achieve and (1)resulted, insist maintain for a long term (2)provoked, fear (5)None of these (3)incited, determined 170.______or else they would not keep electing him (4)activated, accept year after year. (5)angered, believe (1)The party leader gave a strong message to 174.______has been taken against some wholesale the mayor for improving his political style drug dealers for dealing in surgical items without (2)Owing to numerous scandals against the a valid license and maintaining a stock of _____ mayor, he was told to resign from the post drugs. immed iately (1)Note, overwhelming (3)The mayor threatened the residents against (2)Step, impressive filing a complaint against him (3)Execution, outdated (4)The residents must really be impressed with (4)Action, expired the political style of their mayor (5)Lawsuit, invalid (5)None of these 175.Even as the _____ else where in the world are Direction: Each question below has two blanks, struggling to come out of recession, Indian each blank indicating that something has been consumers are splurging on consumer goods and omitted. Choose the set of words for each blank that to _____ this growth, companies are investing best fits the meaning of the sentence as a whole. heavily in various sectors. (1)economies, meet 171.Drawing attention to the pitfalls of______solely (2)countries, inhibit on Uranium as a fuel for nuclear reactors, Indian (3)governments, measure scientists warned that Uranium will not last for (4)nations, inflict long and thus research on Thorium as its____ (5)companies, counter must be revived. (1)using, substitute Direction:Rearrange the following sentences (A), (2)believing, replacement (B), (C), (D), (E) and (F) to make a meaningful (3)depending, reserve paragraph and then answer the questions which (4)reckoning, option follow (5)relying, alternative (A)While these disadvantages of bio fuels are 172.In an effort to provide ______for higher serious, they are the only alternate energy source education to all, most of the universities have of the future and the sooner we find solutions to been providing education without adequate these problems the faster we will be able to solve infrastructure, thus churning out ______the problems wo are now facing with gasoline. graduates every year. (B)This fuel can also help to stimulate jobs locally (1)chances, fresh since they are also much safer to handle thaw” (2)platform, capable gasoline and can thus have the potential to (3)opportunities, unemployable turnaround a global economy. (4)prospects, eligible (C)These include dependence on fossil fuels for the (5)policy, incompetent machinery required to produce biofuel which 173.The move to allow dumping of mercury _____ ends up polluting as much as the burning of fossil an outcry from residents of the area who _____ fuels on roads and exorbitant cost of biofuels that high levels of mercury will affect their health which makes it very difficult for the common and destroy ecologically sensitive forest area. man to switch to this option.

The information provided here is only for refrence. It may vary from the original. www.recruitment.guru

(D)This turnaround can potentially help to bring (1)passed a bill that proposed world peace and end the need to depend on (2)passes a bill with purpose foreign countries for energy requirements. (3)pass a bill proposing (E)Biofuels are made from plant sources and since (4)passed a bill which propose these sources are available in abundance and can (5)No correction required be reproduced on a massive scale they form an 182.Denial of wages forced scientists and teachers energy source that is potentially unlimited. at the agriculture universities throughout the (F)However everything is not as green with the country to go on strike, crippling crucial biofuels as it seems as there are numerous research that could help the state of agriculture disadvantages involved which at times in the country. overshadow their positive impact. (1)from going on strike 176.Which of the following sentence should be the (2)which went on strike FIFTH after rearrangement ? (3)on going for a strike (1)A (2)B (3)C (4)for going to strike (4)E (5)F (5)No correction required 177.Which of the following sentence should be the 183.In an attempt to boost their profits many edible THIRD after rearrangement ? oil producing companies have been engaging (1)A (2)B (3)C themselves in propaganda against commonly (4)D (5)E used oils and. promoting exotic and expensive 178.Which of the following sentence should be the varieties of oils as more healthier options. FIRST after rearrangement ? (1)as most healthiest options (1)A (2)B (3)C (2)as less healthy option (4)D (5)E (3)as a healthier option (4)as much healthiest option 179.Which of the following sentence should be the (5)No correction required SIXTH (LAST) after rearrangement ? (1)A (2)C (3)D 184.Thanks to numerous government initiatives, rural (4)E (5)F masses which was earlier unaware of the luxuries of urban ways of living are now 180.Which of the following sentence should be the connected to the same lifestyle. SECOND after rearrangement ? (1)who was earlier unaware (1)A (2)B (3)D (2)which were earlier aware (4)E (5)F (3)who were earlier conversant Direction: Which of the phrases (1), (2), (3) and (4)who were earlier unaware (4) given below each statement should replace the (5)No correction required phrase printed in bold in the sentence to make it 185.Over the last few months, while most industries grammatically correct ? If the sentence is correct are busy in restructuring operations, cutting costs as it is given and ‘No correction is required’, mark and firing, the Indian pharmaceutical and (5) as the answer. healthcare industry was adding manpower and 181.Soon after the Tsunami had killed thousands of giving salary hikes. people along the coasts of southern India, (1)as many industries are parliament psssas a bill that proposed to set up (2)while most industries were an institutional mechanism to respond promptly (3)while many industries is to natural disasters. (4)where many industries were (5)No correction required

The information provided here is only for refrence. It may vary from the original. www.recruitment.guru

Direction: In the following passage there are blanks, 193.(1)aid . (2)jeopardise each of which ]’as been numbered. These numbers (3)promotes (4)endure are printed below the passage and against each, five (5)cater words/phrases are suggested, one of which fits the 194.(1)acute (2)utilising blank appropriately. Find out the appropriate word/ (3)restricting (4)inspiring phrase in each case. (5)increased 195.(1)both (2)besides There is a considerable amount of research about (3)combining (4)participating the factors that make a company innovate. So is it (5)a ;o possible to create an environment (186 ) to innovation? This is a particularly pertinent (187) for Direction: In each of the following questions four India today. Massive problems in health, education words are given of which two words are most nearly etc. (188) be solved using a conventional approach the same or opposite in meaning. Find the two words but (189) creative and innovative solutions that can which are most nearly the same or opposite in ensure radical change and (190). There are several meaning and indicate the number of the correct factors in India’s (191). Few countries have the rich letter combination, by darkening the appropriate diversity that India or its large, young population oval in your answer sheet. (192). While these (193) innovation policy 196.(A) consent (B) nascent interventions certain additional steps are also required. (C) emerging (D) insecure These include (194) investment in research and (1)A–C (2)B–D development by (195) the government and the private (3)B–C (4)A–D sector, easy transfer of technology from the academic (5)A–B world etc. To fulfill its promise of beng prosperous 197.(A) elated (B) eccentric and to be at the forefront, India must be innovative. (C) explicit (D) abnormal 186.(1)stimuli (2)conducive (1)A–B (2)B–D (3)incentive (4)facilitated (3)A–C (4)A–D (5)impetus (5)D–C 187.(1)objective (2)controversy 198.(A) abundance (B) incomparable (3)doubt (4)question (C) projection (D) plethora (5)inference (1)A–C (2)A–B 188.(1)cannot (2)possibly (3)C–D (4)B–D (3)should (4)never (5)A–D (5)must 199.(A) purposefully (B) inaccurately 189.(1)necessary (2)apply (C) inadvertently (D) unchangeably (3)need (4)consider (1)A–C (2)A–B (5)requires (3)B–C (4)B–D 190.(1)quantity (2)advantages (5)A–D (3)increase (4)chaos 200.(A) germane (B) generate (5)growth (C) reliable (D) irrelevant 191.(1)challenges (2)praises (1)B–D (2)B–C (3)favour (4)leverage (3)A–B (4)C–D (5)esteem (5)A–D 192.(1)blessed (2)enjoys (3)endows (4)prevails (5)occurs

The information provided here is only for refrence. It may vary from the original. www.gradeup.co

1 www.gradeup.co

Direction (1-10): Read the passage carefully A. A space station or particle accelerator. and answer the question that follow. Certain B. Computer software and business methods. words are printed in bold to help you locate C. Technical methods, skills, processes, them while answering some of the question: techniques, tools and raw materials.

Technology can be most broadly defined as D. The state of the respective field's the entities, both material and immaterial, knowledge and tools. created by the application of mental and E. None of the above. physical effort in order to achieve some value. 2. How do developments of technology bring out In this usage, technology refers to tools and cultural changes? machines that may be used to solve real- A. It has lessened barriers to human world problems. It is a far-reaching term that interaction and, as a result, has helped spawn may include simple tools, such as a crowbar new subcultures; the rise of cyber culture has, or wooden spoon, or more complex machines, at its basis, the development of the Internet such as a space station or particle accelerator. and the computer. Tools and machines need not be material; virtual technology, such as computer software B. It combines resources to produce desired and business methods, fall under this products, to solve problems, fulfill needs, or definition of technology. satisfies wants. The word "technology" can also be used to C. Technology refers to tools and machines refer to a collection of techniques. In this that may be used to solve real-world context, it is the current state of humanity's problems. knowledge of how to combine resources to D. As a cultural activity, technology predates produce desired products, to solve problems, both science and engineering. fulfill needs, or satisfies wants; it includes E. None of the above. technical methods, skills, processes, 3. Which two fields have been stated as an techniques, tools and raw materials. When example which when combined with combined with another term, such as "medical technology state the respective field's technology" or "space technology," it refers to knowledge and tools? the state of the respective field's knowledge A. Virtual technology, space technology. and tools. "State-of-the-art technology" refers B. Science, engineering technology. to the high technology available to humanity C. Medical technology, space technology. in any field. D. All of the above. Technology can be viewed as an activity that E. None of the above. forms or changes culture. Additionally, technology is the application of math, science, 4. Pick out the word that is most nearly the and the arts for the benefit of life as it is same in meaning as the word printed in BOLD known. A modern example is the rise of type, as used in the passage. communication technology, which has SPAWN lessened barriers to human interaction and, A. Generate B. Beget as a result, has helped spawn new C. Ruin D. Halt subcultures; the rise of cyber culture has, at E. None of these its basis, the development of the Internet and 5. Pick out the word that is most nearly the the computer. Not all technology enhances same in meaning as the word printed in BOLD culture in a creative way; technology can also type, as used in the passage. help facilitate political oppression and war OPPRESSION via tools such as guns. As a cultural activity, A. Persecution B. Democracy technology predates both science and C. Emergency D. Fair engineering, each of which formalizes some E. B and D. aspects of technological endeavour. 1. What is virtual technology, according to the 6. Pick out the word that is most opposite in passage? meaning as the word printed in BOLD type, as used in the passage.

2 www.gradeup.co

ENDEAVOR The widespread consternation over the rupee A. Negligence B. Avocation hitting a 27-month low against the dollar is C. Contemplate D. Exertion unwarranted, for the Indian currency has E. B and D. been among the better (###Q11###) [hiking] currencies over the last couple of 7. Pick out the word that is most opposite in years. While other (###Q12###) [trickling] meaning as the word printed in BOLD type, as market currencies such as the Russian rouble used in the passage. and the Brazilian real are down more than 20 PREDATES per cent this year, the rupee is lower by just 6 A. Antedate B. Follow per cent. This follows a strong performance in C. Antecede D. Forego 2014, when the Indian currency lost just 1.2 E. None of the above per cent against the greenback. It is obvious 8. According to the passage, how is technology that the rupee is in a sweet spot beneficial for human being? (###Q13###) [peculiar] to its emerging A. It helped spawn new subcultures. market peers, which have been hit hard by the (###Q14###) [ascent] in commodity B. Technology can be used to solve real-world prices. India, on the other hand, has benefited problems. from this fall. The crash in crude prices C. It has lessened barriers to human combined with the checks on gold imports interaction. have helped (###Q15###) [recede] the D. Only A and B current account deficit to just 1.27 per cent of E. All the above GDP. Strong foreign inflows — from both 9. According to the passage, why is development portfolio and direct investments — have of technology referred to as a problem solving pushed India’s forex reserves to $351 billion; tool? we are among the few countries that A. It consists of material and immaterial have (###Q16###) [considered] to entities which help in problem solving. increase forex reserves since the middle of B. It is a far-reaching term that includes last year. These reserves provide the Indian complex tools which help to solve the central bank with (###Q17###) problems. [ammunition] to protect the rupee from short- C. It combines resources to produce desired term volatility that may arise once the Federal products and to solve problems. Reserve goes through with its long-anticipated rate hike. Since the Fed has given financial D. Only B and D markets sufficient time to (###Q18###) E. All the above [discern] the move, a 25 basis points move is 10. According to the author, the central idea of not likely to cause too much turbulence. True, the passage is to…. some short-term money will flow out of the A. Demonstrate award winning achievements equity markets; foreign portfolio investors of technology. have (###Q19###) [turned] net sellers B. Support space technology. since November. But long-term investors are C. Criticize the technological changes. likely to stay put due to the better growth D. Highlight the positive changes brought by (###Q20###) [contrariety] of Indian technology. companies. The superior real yield, falling E. All the above. inflation and a stable rupee also make a strong case for staying invested in Indian debt Directions (11-20): In the passage given instruments. below there are blanks, each followed by a word given in the brackets. Every blank has 11. Choose the correct answer from the given five alternative words given in options. Find options to fill the blanks which are numbered. the word which best suits the respective A. moving B. performing blank. If the given word suits the blank, mark C. doing D. stagnating 'no correction required' as the answer. E. No correction required

3 www.gradeup.co

12. Choose the correct answer from the given 21. When we went to wonder of worlds, I loved options to fill the blanks which are numbered. that really big old silver antique car that A. emerging B. on-going was parked in the parking lot of the East India C. suiting D. menacing Mall. E. No correction required A. Really old silver big antique car 13. Choose the correct answer from the given B. Really big silver old antique car options to fill the blanks which are numbered. C. Really big silver antique car A. known B. employed D. Really big old silver antique car C. straight D. relative E. No correction required E. No change required 22. Each individual knows how to protected on 14. Choose the correct answer from the given own life and it should be left to his discretion. options to fill the blanks which are numbered. A. Protected by own A. spree B. rise B. Protect by own C. plunge D. demolition C. Protect his own E. No correction required D. Protected his own 15. Choose the correct answer from the given E. No change required options to fill the blanks which are numbered. A. boost B. recover 23. The last few decades has seen a great deal C. maintain D. sustain of political instability in India. E. No correction required A. Decade had seen 16. Choose the correct answer from the given B. Decade has seen options to fill the blanks which are numbered. C. Decades have seen A. managed B. agitated D. Decade have seen C. breached D. stood E. No change required E. No correction required 24. He admired the speed with which Koti 17. Choose the correct answer from the given completed the work and appreciating the options to fill the blanks which are numbered. method adopted by him A. gadget B. thing A. appreciate the method being adopted C. product D. process B. appreciated the method adopted E. No correction required C. appreciate the method of adoption 18. Choose the correct answer from the given D. appreciate the adopting method options to fill the blanks which are numbered. E. No correction required A. hinder B. assimilate 25. I had met him after the party where he had C. nullify D. ostracize been given an inspiring speech E. No correction required A. when he had 19. Choose the correct answer from the given B. where he would have options to fill the blanks which are numbered. C. in which he was given A. spanned B. produced D. where he had C. acquired D. raised E. No correction required E. No correction required. 20. Choose the correct answer from the given 26. After the success of our project we have been options to fill the blanks which are numbered. receiving more requests than we do not A. features B. plight have the resources to handle them. C. matters D. prospects A. many requests but E. No correction required B. most of the requests

Directions (21-30): Which of the phrases C. more requests that (A), (B), (C) and (D) given below each D. too many requests sentence should replace the phrase printed E. No correction required in bold type to make the sentence 27. The poor Brahmin led a hand to mouthful grammatically correct. If the sentence is existence and could use any job which paid correct as it is, mark (E), i.e., 'No correction him a little. required' as the answer.

4 www.gradeup.co

A. handful to mouthful existence 34. I. B. hand to mouth existence C. handing for mouthful existing II. D. hand and mouth exist A. x < y B. x > y E. No correction required C. x ≤ y D. x ≥ y 28. In order to earning decent living we need E. x = y or no relation can be established. to have a good job which pays a substantial amount of money. 35. I. A. earned decency life B. earning decency live II. C. earn a decent living D. earned decently life A. x < y B. x > y E. No correction required C. x ≤ y D. x ≥ y E. x = y or no relation can be established. 29. We went to the famous restaurant to eat and were served piped hot food. 36. Direction: What will come in place of the A. served piping hotter question mark (?) in the following number B. serving pipe hot series? C. served piping hot 14, 8, 9, 14.5, 30, ? D. serve pipe hotten A. 72 B. 73 E. No correction required C. 74 D. 75 30. When he fell down the ditch, he shouted with E. 76 all his might so that to catch someone’s 37. Direction: What will come in place of the attention. question mark (?) in the following number A. such that to catch series? B. so as to catch 77, 85, 69, 101, 37, ? C. so that to catching A. 105 B. 125 D. so then to catch C. 145 D. 165 E. No correction required E. 185

Direction (31-35): In the following question, 38. In the following number series, one number is two equations I and II are given. Solve both missing. What should come at the place of the equations carefully & answer the missing number (?) questions given below: 20, 29, 54, 103, 184, ? 31. I. A. 301 B. 302 C. 303 D. 304 II. E. 305 A. x < y B. x > y 39. Direction: What will come in place of the C. x ≤ y D. x ≥ y question mark (?) in the following number E. x = y or no relation can be established. series? 32. I. 7, 8, 18, 57, ?, 1165 A. 212 B. 217 II. C. 232 D. 247 A. x < y B. x > y E. 275 C. x ≤ y D. x ≥ y E. x = y or no relation can be established. 40. Direction: What will come in place of the question mark (?) in the following number 33. I. series? II. 5, 7, 18, 47, 103, ? A. x < y B. x > y A. 155 B. 175 C. x ≤ y D. x ≥ y C. 195 D. 215 E. x = y or no relation can be established. E. 235

5 www.gradeup.co

41. Direction: What should come in place of question mark (?) in the following question? (You do not have to calculate the exact value.)

A. 12 B. 42 C. 92 D. 132 E. 172 42. Direction: What approximate value will come in place of the question mark (?) in the following question?

46. If the demand for brand C product increase by A. 24 B. 39 75% then to meet the demand production C. 36 D. 28 should be increased by what percent? E. 32 A. 32.65% B. 48.48% 43. What approximate value will come at the C. 57.14% D. 31.25% place of question mark (?) in the following E. None of these question? 47. Brand A increase its production to meet its demand. With every 160 unit produced the brand increases its price by 10%. If the A. 19 B. 18 earlier price of one product was INR 5000 C. 21 D. 25 then find the new price of the product. E. 15 A. INR 5500 B. INR 6655 44. Directions: What approximate value will C. INR 7320.5 D. INR 8052.55 come in place of the question mark (?) in the E. Cannot be determined following question? (You are not expected to 48. The demand for brand D product fell. The new calculate the exact value) demand is 20% less than its production. Find by what percentage demand fell? 7441 ÷34 × 12 = ? × 9 + 110 A. 28% B. 30% A. 420 B. 280 C. 38.88% D. 72% C. 590 D. 350 E. 32% E. 220 49. Brand B decreased its price of the product to 45. Directions: What approximate value will meet its demand to its production. When the come in place of question mark (?) in the price decreased by 12% the demand following questions? (You are not expected to increased by 25%. If the ratio between the calculate the exact value.) new price and new demand is 11:20 then find 5466.97-3245.01+1122.99=? + 2309.99 the price of the product before the decrease. A. 1130 B. 1000 A. INR 2500 B. INR 2300 C. 1100 D. 1030 C. INR 2200 D. INR 2000 E. 1060 E. INR 2800 50. The production of brand E and F took together Direction (46-50): Study the following is approx. what percent of total demand of E graph carefully to answer the question given and F? below: A. 81% B. 21%

Given below is the demand and production of C. 123% D. 121% 6 brands (in units) of a product in the year E. 23% 2016.

6 www.gradeup.co

51. Ratio between height of 2 cylinder is in the 57. In a 40 litres mixture acetic acid and sodium ratio 3:5. Their volumes are in the ratio acetate are in the ratio 3:1,find the amount of 27:80. Find ratio between their radius. sodium acetate solution to be added to make the ratio 2:3. A. 40 litres B. 20 litres A. B. C. 15 litres D. 30 litres E. 35 litres C. D. 58. Radhika has two daughters by name Rinku E. None of these and Sindhu. The ratio of the age of Radhika 52. B is 20% efficient than A. B started the work and Rinku is 3:1 and that of Rinku and Sindhu & do it for x days. And then B is replaced by is 8:5. Given that Rinku is six years elder to A. And A completed the remaining work in Sindhu. Find the ratio of their ages after 12 x+8 days. Ratio of work done by A & B is 3:2. years. In how many day A & B working together to A. 29:15:12 B. 30:14:10 complete the whole work? C. 29:14:11 D. 30:14:11 A. 120/12 B. 150/11 days E. 30:13:11 C. 140/13 days D. 100/33 days 59. Three years ago the average age of Mohan’s E. 75/12 days family having 5 members was 17 years. 53. The time taken for covering ‘X’ Km by Mohan becomes father but the average age of downstream is equal to 'X-18' by covering his family is same today. What is the present upstream. Upstream speed is 6 km/ hr less age of baby? than that of downstream. If the speed of the A. 1 year B. 2 years boat in still water is 15 km/hr. What is the C. 3years D. 4 years value of 'X'. E. 5 years A. 51 B. 52 60. Out of 12 persons, 11 spend Rs.2000 monthly C. 53 D. 54 each. The twelfth person spends Rs.110 more E. 55 than the average spending of the 12 people. 54. A sum of Rs. 91,000 is borrowed at 20% per How much money does the twelfth person annum compounded annually. If the amount spend? is to be paid in two years, the amount will be? A. Rs.2200 B. Rs.2120 A. 1,20,000 Rs. B. 1,25,760 Rs. C. Rs.3300 D. Rs.1800 C. 1,27,526 Rs. D. 1,31,040 Rs. E. Rs.2010 E. 1,34,034 Rs. 55. If an article is marked 40% above the cost Direction (61-65): Read the following price. If discount of x% is given on the information carefully and answer the marked price of the article then final profit of questions given below.

12% is obtained. Now if CP of a new article is 120 Rs. and x% profit is desired then what should be the selling price of that new article? A. 140 B. 142 C. 144 D. 146 E. 148 56. A, B and C started a business and invested in the ratio of 3:4:5. After 4 months, A withdrew 1/12th amount of what B and C invested. If

the annual income was 9200, then what was 61. Find the average of the number of female in the share of B? all the colleges except college C and E? A. 3280 B. 3480 A. 672.8 B. 683.5 C. 3200 D. 3880 C. 750 D. 753.5 E. 4080 E. 602.8

7 www.gradeup.co

62. Find the average difference between the A. F B. D number of male and female students in all the C. B D. H colleges? E. E A. 312.833 B. 314.60 68. Who among the following does not belongs to C. 313 D. 314.50 the group? E. None of these A. A B. D 63. The number of female students in college C is C. G D. B what approx. percent of the number of male E. H students of college A? 69. How many persons have birthday between D A. 70% B. 72% and E? C. 74% D. 76% E. 77% A. 1 B. 2 64. Out of total female in college E, 30% are in C. 3 D. 4 Arts department which is 35% of the total E. 5 students in Arts department. Find out 70. Who among the following in not born before approximately how much percent of male F? students from E are in Arts department? A. A B. D A. 20% B. 22% C. C D. H C. 19% D. 23% E. G E. 25% 65. Find the ratio of 2/3rd of college B male Directions (71-73): In these questions, students and female students of college F. relationship between different elements is A. 29: 11 B. 29: 33 shown in the statements. The statements are C. 33: 29 D. 29: 22 followed by two conclusions. Find the E. 22: 29 conclusion which is definitely true. 71. Statements: Direction (66-70): Study the following A > B = C < D < E > F information carefully and answer the question Conclusions: based on it. I. F < C 8 persons A, B, C, D, E, F, G and H have their II. A>D birthdays on 14th and 21st of four different months, January, March, April and June not A. If only Conclusion I is true. necessarily in the same order. A has birthday B. If only Conclusion II is true. on even date and a month which having 31 C. If either Conclusion I or II is true. days. Number of persons between C and F is D. If neither Conclusion I nor II is true. same as the number of persons between B E. If both Conclusions I and II are true. and H. H’s birthday is neither on even number 72. Statements: date nor in a month which having 31 days. B A < B > C > D; A > E, D > F is not born after H. C is born before F and B, Conclusions: who is not born in January. D is not born in I. F > B June. F was born on even number date. D’s II. B > E birthday is on odd number date and a month A. If only Conclusion I is true. which having 31 days. B and H were not born B. If only Conclusion II is true. in the same month. E was born after F. D is C. If either Conclusion I or II is true. born before G. D. If neither Conclusion I nor II is true. 66. How many persons born after F? E. If both Conclusions I and II are true. A. 1 B. 2 73. Statements: C. 3 D. 4 A = B < C > D; E > C < F E. 5 Conclusions: 67. Who among the following is not born in the I. E > A month having 30 days? II. F > D

8 www.gradeup.co

A. If only Conclusion I is true. 78. Which of the following pair is neighbour of U? B. If only Conclusion II is true. A. R and W B. S and P C. If either Conclusion I or II is true. C. W and V D. R and V D. If neither Conclusion I nor II is true. E. Q and R E. If both Conclusions I and II are true. 79. Who among the following is 14 years old? 74. Which of the following symbols should replace A. P B. Q the sign ($) and (#) in the given expression in C. R D. U order to make the expression S > J and M ≥ J E. S is definitely true? 80. How many persons are younger than W? S ≥ T $ P > K ≤ N, O < M = K # J > L A. 1 B. 2 A. =, < B. >, ≤ C. 3 D. 4 C. <, ≥ D. <, ≤ E. None E. >, ≥ 81. Direction: If 1 number is added in even digit 75. Which of the following symbols should replace in the given number 9458732 and 2 is the $ and # in the given expression in order subtracted in odd digit, then how many digit to make the expression L ≥ S and N > T is is repeated in the newly formed number? definitely true? N > O ≤ L = P $ T, H > M ≤ T # S < N A. 1 B. 2 C. 3 D. 4 A. >, < B. ≥, ≤ E. 5 C. >, = D. <, ≤ E. >, ≥ 82. In a certain code language ‘green grass everywhere’ is written as ‘dik pa sok’ and ‘cow Direction (76-80): Study the following eats grass’ is written as ‘nok ta pa’. How is information carefully and answer the ‘cow’ written in that code language? questions given below: A. nok B. ta Eight persons P, Q, R, S, T, U, V and W are C. nok or ta D. Data inadequate sitting in a straight line and facing north E. None of these

direction. Their ages are, 12, 14, 18, 26, 29, Direction (83-85): Study the following 35 ,42 and 67. The one who is 12 years old is information carefully and answer the th 4 to the left of the eldest person, who is questions given below: sitting at the end. The sum of the ages of S and Q is equal to P. S is not the youngest Each of the six students got different marks in person. S and W are neighbours of Q. R’s age an exam. D got less marks than only B. E got is not an even number and he is older than W more marks than only two students. A didn’t and younger than U. Only 3 persons are get less marks than E. C did not get least sitting between S and U. Only 2 persons are marks. The one who got second lowest, got sitting between Q and T, who is 29 years old. 160 marks. A got 205 marks. P is not the neighbour of U. R is not sitting to 83. The one who got maximum marks, got 100 the left of Q. marks more than C. Which of the following 76. Who among the following sits at the end? mark is got by one who got maximum marks? A. T B. P A. 225 B. 270 C. Q D. S C. 260 D. 300 E. W E. None of these 77. Who among the following sits third to the left 84. Which of the following is true with respect to F of W? as per the given information? A. S A. The possible marks, got by F is 165 B. The one who is 12 years old B. F got minimum marks. C. The one who is 26 years old C. Only two students got more marks than F D. T D. F definitely got more marks than C E. U E. None of these

9 www.gradeup.co

85. Which of the following may be the possible Seven persons A, B, C, D, E, F and G attends number of marks which E got? seminar on different dates starting from A. 210 B. 140 21st June to 27th June but not necessarily in C. 185 D. 159 the same order. They like different brands E. None of these laptop, Lenovo, Dell, Apple, Sony, Samsung, Asus, and HP. They have different brands of Direction (86-90): Study the following watch, Sonata, Rolex, Titan, Maxima, Casio, information carefully and answer the question Fossil and Diesel. given below: The one who has Diesel attends a seminar on There are eight persons A, B, C, D, E, F, G an even numbered date. E has Rolex and F and H. They are born in the month (Same for doesn’t have Maxima. G attends the seminar each person) of different years i.e. 1976, on the last day. The one who has Diesel likes 1980, 1982, 1990, 1991, 1995, 2000 and either Apple or Asus laptop. A has Titan but 2005. There ages are considered as on the he doesn’t like HP and is not attend the same month of 2017. B is born in odd number seminar just before or just after B. F likes Dell year but he is not the youngest. E is 37 years and attends the seminar three days before old now. C was born in 1990. G is at least 9 the one who likes Samsung. D neither likes years older than B. D is younger than B and Samsung nor Asus laptop. The one who likes born in odd number year. A is 18 years HP attends the seminar on 25th June. B likes younger than G. F is older than D but not the Sony wearing also have Sonata watch and not older person. Not more than 2 persons are attends the seminar just before or just after younger than F. F, who attends the seminar on an odd 86. Who among the following is youngest person? numbered date. The one who likes Apple A. A B. C neither has Casio nor Maxima. B attends the C. D D. B seminar just before D and none of them likes E. G HP. The one who has Casio attends the nd 87. How many persons are older than C? seminar on the 2 day. A. 2 B. 3 91. Who among the following attends the seminar rd C. 1 D. None on 3 day? E. more than 3 A. D 88. What is the difference of age between F and B. The one who likes Dell E? C. C A. 15 years B. 10 years D. The one who has Diesel C. 30 years D. 4 years E. The one who likes Samsung E. 12 years 92. Which of the following is correct combination 89. If all the persons are arranged according to for D? their names according to alphabet series from A. Lenovo-Casio youngest to oldest then how many of them B. Dell-Sonata position will remain unchanged? C. Samsung-Rolex A. 0 B. 1 D. Sony-Fossil C. 2 D. 3 E. HP-Rolex E. 4 93. How many persons attend the seminar 90. Who among the following is born in 1982? between the one who likes Samsung and the A. D B. G one who has Fossil? C. A D. C A. 0 B. 1 E. H C. 2 D. 3

Direction (91-96): Study the following E. 4 information carefully and answer the 94. If all the persons rearranged according to questions given below: their names (alphabetically) from the starting date then who among the following likes Apple?

10 www.gradeup.co

A. A B. B 98. How is B related to D? C. C D. D A. Mother-in-law B. Sister E. E C. Mother D. Can’t be determined 95. Which of the following combination is correct? E. None of these A. D-Lenovo-Fossil B. E-HP-Titan Directions (99-100): Study the following C. A-Sony-Maxima D. F-Dell-Diesel information carefully and answer the E. A-Samsung-Titan questions given below: 96. ‘B’ is related to ‘Dell’ and ‘F’ is related to Rahul starts running from point A and runs ‘Rolex’ in the same way then who among the 15 km towards south. He takes a left turn and following is related to ‘Maxima’? runs 20 km. Now he runs 9 km after taking a A. A B. B right turn. He finally takes a right turn and C. C D. D runs 20 km and stops at point B. E. E 99. How far is point B with respect to point A? Directions (97-98): Study the following A. 24 km B. 40 km information carefully and answer the C. 45 km D. 29 km questions given below:

E. 49 km * D is the mother of E, who is the sister of F. 100. Towards which directions was Rahul moving * A is the father of C. B is wife of A. before he stopped at point B? * F is son of C. A. North B. East 97. How is E related to A? C. West D. South A. Son B. Grandson E. North-West C. Granddaughter D. Daughter E. None of these

***

11 www.gradeup.co

12

www.recruitment.guru

STATE BANK OF INDIA PAPER ON 7th JANUARY 2007 Question Paper

Section 1: Reasoning

Directions (1-5): In the questions given below, certain symbols are used with the following meaning: A @ B means A is greater than B. A * B means A is either greater than or equal to B. A # B means A is equal to B A $ B means A is either smaller than or equal to B A + B means A is smaller than B

Now in each of the following questions assuming the given statements to be true, find which of the two conclusions I and II given below them is/are definitely True? Give answer (a) if only conclusion I is true. Give answer (b) if only conclusion II is true. Give answer (c) if either conclusion I and II are true. Give answer (d) if neither conclusion I nor II is true. Give answer (e) if both conclusions I and II are true.

1. Statements: D + T ; E $ V ; F * T ; E @ D Conclusions: I. D $ V II. D + F Ans: (b)

2. Statements: B + D ; E $ T ; T * P ; P @ B Conclusions: I. P $ D II. P @ D Ans: (c)

3.Statements: T * U ; U $ W ; V @ L ; W + V Conclusions: 1. V @ T II. L # W Ans: (d)

4.Statements: P $ Q ; N # M ; M @ R ; R * P Conclusions: I. P + N II. Q $ M Ans: (a)

The information provided here is only for refrence. It may vary from the original. www.recruitment.guru

5.Statements: E * F ; G $ H ; H # E ; G @ K Conclusions: I. H @ K II H * F Ans: (e)

Directions(6-11): Study the following letter-number-symbol sequence carefully and answer the questions given below: 3 D 5 F E 3 8 $ M 2 1 K * P T @ U 9 A 7 1 £ H J 4 Q 6

6. What should come in place of the question mark (?) in the following Sequence? 5ES, MIP, ?, 1HQ (a) TUA (b) TU7 (c) @ 91 (d) T91 (e) None of these Ans: (b)

7.Which of the following is exactly in the midway between the eleventh from the left end and the 7th from the right end? (a) P (b) @ (c) T (d) U (e) None of these Ans: (b)

8. Which of the following is the sixth to the right of the twentieth from the right end? (a) 5 (b) F (c) P (d) K (e) None of these Ans: (c)

9. How many such digits are there in the above sequence which are immediately preceded as well as followed by digits ? (a) None (b) One (c) Two (d) Three (e) more than three Ans: (a)

10. If the first fifteen elements are written in the reverse order then which of the following will be eighth to the left of the thirteenth element from right end ? (a) M (b) 8 (c) $ (d) * (e) None of these. Ans: (a)

11. If all the consonants starting from B are given sequentially the value of even numbers such as B=2, C=4 and so on and all the vowels are given the value of 5 each, then what will be the value of the letters of the word CUSTOM ? (a) 92 (b) 86 (c) 82 (d) 96 (e) None of these Ans: (d)

12. How many such 5s are there in the following sequence that the sum of the two immediately following digits is greater than the sum of the two immediately

The information provided here is only for refrence. It may vary from the original. www.recruitment.guru

preceding digits ? 3 7 6 5 8 3 2 4 5 5 4 8 7 9 1 5 3 4 8 7 5 9 8 7 6 4 (a) One (b) Two (c) Three (d) Four (e) None of these Ans: (c)

13. If A + B means "A is the sister of B", A × B means "A is the wife of B", A ÷ B means "A is the father of B" and A-B means "A is the brother of B"' then which of following expresses the relationship that "t is the daughter of p"? (a) P × Q ÷ R + S - T (b) P × Q ÷ R - T + S (c) P × Q ÷ R + T - S (d) P × Q ÷ R + S + T (e) None of these Ans: (b)

14.If the position of the first letter of English alphabet is interchanged with the position of the fourteenth letter, second letter with the fifteenth letter in such a way that M is interchanged with Z, then which of the following letters will be 9th to the right of 17th letter from the right? (a) F (b) E (c) R (d) T (e) None of these Ans: (a)

Directions(15-18) : Read the following information and answer the questions given below:- (i) Seven friends P, Q, R, S, T, U and W have gathered at the Mumbai airport. Five of them are scheduled to go to five different places-Delhi, Chennai, Lucknow, Bangalore and Calcutta. (ii) Five of them are executives, each specialising in viz. Administration (admn.), Human Resource Management (HRM), Marketing ,Systems and Finance. (iii) T, an executive is going to Chennai and is neither from finance nor Marketing. (iv) W is a system specialist and is leaving for Delhi. U is an executive but is not going to one of the five places. (v) Q is an executive from HRM but has come at the airport to see his friends. (vi) P is an executive but not from Marketing and is flying to one of the destinations but not to Bangalore or Calcutta.

15. Who is going to fly to Bangalore? (a) Data inadequate (b) R (c) S (d) P (e) None of these Ans: (a)

16. Who among the following specialises in Marketing? (a) S (b) P (c) U (d) Data inadequate (e) None of these Ans: (c)

17. R has specialisation in which of the following fields? (a) Finance (b) Marketing (c) Either Marketing or Finance (d)

The information provided here is only for refrence. It may vary from the original. www.recruitment.guru

None (e) All of these Ans: (d)

18.The one who is going to fly to Chennai is (a) Not an executive (b) From administration (c) S (d) From Finance (e) None of these Ans:(b)

19. How many pairs of letters are there in the word 'NURSING' which have as many letters between them as in the alphabet? (a) One (b) Three (c) Five (d) Six (e) None of these Ans: (b)

Directions(20-26): Read the following information carefully and answer the questions given below. A famous museum issues entry passes to all its visitors for security reasons. Visitors are allowed in batches after every one hour. In a day there are six batches. A code is printed on entry pass which keeps on changing for every batch. Following is an illustration of pass-codes issued for each batch. Batch I: clothes neat and clean liked are all by Batch II: by clothes neat all are and clean liked Batch III: liked by clothes clean and neat all are and so on....

20. If pass-Code for the third batch is 'night succeed day and hard work to for ', what will be the pass code for the sixth batch? (a) Work hard to for succeed night and day (b) Hard work for and succeed night to day (c) Work hard for to succeed night and day (d) Hard work for to succeed night and day (e) None of these Ans: (c)

21. If 'visit in zoo should the we time day' is the pass code for the fifth batch, 'zoo we the should visit day time in ' will be the pass code for which of the following batches? (a) II (b) IV (c) I (d) III (e) VI Ans: (d)

22. Sanjay visited the museum in the fourth batch and was issued a pass-code 'to fast rush avoid not do very run'. What would have been the pass-code for him had he visited the museum in the second batch? (a) rush do not avoid to run very fast (b) rush not do avoid to run very fast (c) avoid rush not do to run very fast (d) Data inadequate

The information provided here is only for refrence. It may vary from the original. www.recruitment.guru

(e) None of these Ans: (a)

23. Subodh went to visit the museum in the second batch. He was issued a pass- code 'length the day equal of and night are'. However, he could not visit the museum in the second batch as he was little late. He then preferred to visit in the fifth batch. What will be the new pass -code issued to him? (a) and of are night the length equal day (b) and are of night the length equal day (c) and of are night the equal (d) and of are the night length day equal (e) None of these Ans: (a)

24. If pass-code for the second batch is 'to confidence hard you leads work and success', what will be the pass -code for the fourth batch? (a) leads success to you hard confidence and work (b) leads success you to hard confidence and work (c) leads success to you hard confidence work and (d) leads to success you hard confidence and work (e) None of these Ans: (a)

25. If the pass-code issued for the last (sixth) batch is 'and pencil by all boys used are pen'. What will be the pass-code for the first batch? (a) pencil and pen are used by all boys (b) pen and pencil used are by all boys (c) pen and pencil are used by all boys (d) pencil and pen are used all by boys (e) None of these Ans: (c)

26. If the pass-code for the sixth batch is 'not go the way to of out do'. What will be the pass-code for the third batch? (a) of do to out go not way the (b) of to do out not go way the (c) of to go out do not way the (d) Data inadequate (e) None of these Ans: (e)

Directions(27-32): in each question below are given three statements followed by four conclusions-I, II, III and IV. You have to take the given statements to be true even if they seem to be at variance with commonly known facts. Read the all conclusions and then decide which of the given conclusions logically follow (s) from the given statements disregarding commonly known facts.

27.Statements Some books are pens All pens are chairs.

The information provided here is only for refrence. It may vary from the original. www.recruitment.guru

Some chairs are Tables. Conclusions I. Some books are chairs. II . Some chairs are books. III. All tables are chairs. IV. Some tables are chairs. (a) All follow ( b) Only I, II, III follow (c) Only I, II, IV follow (d) Only II, III and IV follow (e) none of these Ans:(c)

28. Statements All cars are jeeps. All jeeps are buses. All buses are trucks. Conclusions I. All trucks are buses. II. All buses are jeeps. III. All jeeps are cars. IV. All cars are trucks. (a). None follows (b) All follow (c) Only III and IV follow (d) Only IV follows (e) None of these Ans:(d)

29.Statements Some trees are flowers. Some flowers are pencils. Some pencils are tables. Conclusions I. Some tables are flowers. II. Some pencils are trees. III. Some tables are trees. IV. Some trees are pencils. (a) All follow (b) None follows (c) Only I and III follow (d) Only II and IV follow (e) None of these Ans: (b)

30.Statements All rods are bricks. Some bricks are ropes. All ropes are doors. Conclusions I. Some rods are doors. II. Some doors are bricks. III. Some rods are not doors. IV. All doors are ropes. (a) Only I and II follow

The information provided here is only for refrence. It may vary from the original. www.recruitment.guru

(b) Only I, II and III follow (c) Either I or III and II follow (d) Either I or III and IV follow. (e) None of these Ans.(c)

31. Statements Some books are pens. Some pens are watches. Some watches are books . Some books are watches. Conclusions I. Some radios are watches. II. Some radios are pens. III. Some watches are books IV. Some books are watches. (a) All follow (b) Only I and III follow (c) Only II and IV follow (d) Only I and IV follow (e) None of these Ans:(e)

32. Statements All towns are villages No village is forest Some forests are rivers. Conclusions I. Some forests are villages. II. Some forests are not villages. III.Some rivers are not villages. IV.All villages are towns. (a) All follow (b) Only either I or II follows (c) Only either I or II and III follow (d) None of these (e) None of these Ans: (e)

33. In a row of boys facing north, Sudhanshu is twelfth from his left. When shifted to his right by four places, he becomes eighteenth from the right end of the row? (a) 32 (b) 33 (c) 34 (d) Data inadequate (e) None of these Ans: (b)

34.In a certain code language PROBLEM is written as MPERLOB. How will NUMBERS be Written in that code? (a) SNUREMB (b) SNRUBME (c) SNRUEMB (d) SNRUMEB

The information provided here is only for refrence. It may vary from the original. www.recruitment.guru

(e) None of these Ans:(c)

Directions(35-40):In each question below is given a statement followed by two assumptions numbered I and II. An assumption is something supposed or taken for granted. You have to consider the statement and the following assumptions and decide which of the assumptions is implicit in the statement. Give Answer (a) if only assumption I is implicit. Give Answer (b) if only assumption II, is implicit. Give Answer (c) if either I or II is implicit. Give Answer (d) if neither I nor II is implicit. Give Answer (e) if both I and II are implicit.

35.Statement: The Government has recently hiked the prices of diesel and petrol to reduce the oil pool deficit. Assumptions: I. The amount earned by this increase may be substantial enough to reduce the deficit. II. There may be wide spread protests against the price hike. Ans:(e)

36. Statement: The X passenger car manufacturing company announced a sharp reduction in the prices of their luxury cars. Assumptions: I. There may be an increase in the sale of their luxury cars. II. The other, such car manufacturers may also reduce their prices. Ans:(e)

37. Statement : A foreign film producer rendered his apology before Indian society for misinterpreting a part of Indian epic. Assumptions : I. Indians are very sensitive to the misinterpretation of their epic. II. It is possible to derive wrong meaning from the epic. Ans:(d)

38.Statement : Aswin's mother instructed him to return home by train if it rains heavily. Assumptions I. Aswin may not be to decide himself if it rains heavily. II. The trains may ply even if it rains heavily. Ans:(b)

39. Statements : The Government of India decided to start a track II dialogue with its neighbour to reduce tension in the area. Assumption: I. The neighbouring country may agree to participate in the track II dialogue. II. The people involved in track II dialogue may be able to persuade their respective Governments. Ans:(e)

The information provided here is only for refrence. It may vary from the original. www.recruitment.guru

40. Statements :The host in one of the popular T.V programmes announced that the channel will contact the viewers between 9.00 a.m. to 6.00 p.m on weekdays and the lucky ones will be given fabulous prizes. Assumptions: I. The people may remain indoors to receive the phone call. II. More people may start watching the programme. Ans:(e)

Directions(41-45): In making decisions about important questions, it is desirable to be able to distinguish between "Strong" arguments and "Weak" arguments are those which are both important and directly related to the question. "Weak" arguments are those which are of minor importance and also may not be directly related to the question or may be related to a trivial aspect of the questions. Instructions: Each question below is followed by a statement and two arguments numbered I & II. You have to decide which of the argument is a "Strong" argument and which is a "Weak" argument. Give answer (a) if only argument I is strong. Give answer (b) if only argument II is strong. Give answer (c) if their I or II is strong. Give answer (d) if neither I nor II is strong. Give answer (e) if both I and II are strong.

41.Statement: should the habit of late coming in educational institutions be checked? Arguments: I. No. Until it affects the work. II. Yes. Discipline must be maintained. Ans:(b)

42.Statement: Should seniority be the only criterion for the promotion? Arguments: I. No. All the senior employees are not interested in promotion. II. Yes. Otherwise senior employees do feel humiliated. Ans:(d)

43. Statement: Should children be prevented completely from watching television? Arguments: I. No. We get vital information regarding education through television. II. Yes. It hampers the study of children. Ans:(e)

44. Statement: Should trade unions be banned completely? Arguments: I. No. This is the only way through which employees can put their demands before management. II. Yes. Employees get their illegal demands fulfilled through these unions. Ans:(a)

The information provided here is only for refrence. It may vary from the original. www.recruitment.guru

45. Statement: should women be given equal opportunity in matter of employment in every field? Argument: I. Yes. They are equally capable. II. They have to shoulder household responsibilities too. Ans:(a)

Directions(46-50): In each question below is given a statement followed by two conclusions numbered I and II.You have to assume everything in the statement to be true, then consider the two conclusions together and decide which of them logically follows beyond a reasonable doubt from the information given in the statement. Give answer (a) if only conclusion I follows. Give answer (b) if only conclusion II follows. Give answer (c) if either I or II follows. Give answer (d) if neither I nor II follows, and Give answer (e) if both I and II follow.

46. Statement: The cabinet of State 'X' took certain steps to tackle the milk glut in the state as the cooperatives and Government diaries failed to use the available milk-A news report Conclusion: I. The milk production of State 'X' is more than its need. II. The Government and co-operative diaries in State 'X' are not equipped in terms of resources and technology to handle such excess milk. Ans:(e)

47. Statement: It has been decided or the Government to withdraw 33% of the subsidy on cooking gas from the beginning of next month-A spokesman of the Government. Conclusions: I. People now no more desire of need such subsidy from Government as they can afford increased price of the cooking gas. II. The price of the cooking gas will increase at least by 33% from the next month. Ans:(d)

48. Statement: "The Government will review the present policy of the diesel price in view of further spurt in the international oil prices"-A spokesman of the Government. Conclusions: I. The Government will increase the price of the diesel after the imminent spurt in the international oil [prices. II. The Government will not increase the price of the diesel even after the imminent spurt in the international oil prices. Ans:(c)

49.Statement: My first and foremost task is to beautify this city-if city 'X' and Y can do it- why can't we do it-statement of Municipal Commissioner of City 'Z' after taking over charge.

The information provided here is only for refrence. It may vary from the original. www.recruitment.guru

Conclusions: I. The people of city 'Z' are not aware about the present state of their city. II. The present commissioner has worked in city 'X' and Y and has good experience of beautifying cities. Ans:(d)

50.Statement: Women's Organisations in India have welcomed the amendment of the Industrial Employment Rules 1946 to curb sexual harassment at the work place. Conclusions: I. Sexual harassment of women at work place is more prevalent in India as compared to other developed countries. II. Many organisations in India will stop recruiting women to avoid such problems. Ans:(d)

Directions (51-55): Each of the questions below consists of a question and two statements numbered I and II given below it. You have to decide whether the data provided in the statements numbered I and II given below it. You have to decide whether the data provided in the statements are sufficient to answer the questions. Read both the statements and - Give Answer (a) if the data in statement I alone are sufficient to answer the question, while the data in statement II alone are not sufficient to answer the question. Give answer (b) if the data in statement II alone are sufficient to answer the question, while, the data in statement I alone are not sufficient to answer the question. Give answer (c) if the data either in the statement I alone or in statement II alone are sufficient to answer the question. Give answer (d) if the data even in both statement I and II together are not sufficient to answer the question. Give answer (e) if the data in both statements I and II together are necessary to answer the question.

51. In which direction is Ravi facing? I. Asok is to the right of Ravi II. Samir is sitting opposite of Asok facing north. Ans:(d)

52. How M is related to I. The sister of K is the mother of N who is daughter of M . II. P is the sister of M. Ans: (e)

53. Is D brother of T? I. T is the sister of M and K. II. K is the brother of D. Ans: (d)

The information provided here is only for refrence. It may vary from the original. www.recruitment.guru

54. How many sons does P have? I. S and T are brothers of M. II. The mother of T is P who has Only One daughter. Ans: (e)

55. Who is the tallest among P, Q, R, S and T? I. R is taller than Q and T. II. T is taller than S and P and S is taller than Q and R. Ans: (b)

Directions (56-60) : Below is given a passage followed by several possible inferences which can be drawn from the fact stated in the passage. You have to examine each inference separately in the context of the passage and decide up on its degree of truth or falsity. Mark answer (a) if the inference is "definitely true" i.e., it properly follows from the statement of facts given. Mark answer (b) if the inference is "probably true" though not "definitely true" in the light of the facts given. Mark answer (c) if the data are inadequate i.e, from the facts given you cannot say whether the inference is likely to be true or false. Mark answer (d) if the inference is "probably false" in the light of the facts given. Mark answer (e) if the inference is "definitely false" i.e, it cannot possibly be drawn from the facts given or it contradicts the given facts. With the purpose of upliftment of Gonda district in Uttar Pradesh, a new formula way evolved for practical success in several fields, such as, irrigation, animal husbandry, dairy farming, moral uplift and creation of financial resources. Small farms were clustered for irrigation by one diesel pump which could irrigate about 20 acres of land. Youth were prompted to take loans from the banks for purchase of engine pumps to be supplied to the farmers on rent. This formula worked so well that the villages in Gonda district were saturated with irrigation facilities. Cattle rearing was linked with multiple cropping . Most of the targets fixed for different areas were achieved, which was an unusual phenomenon. This could be possible only because of right motivation, participation and initiative of the people. Imagination and creativity combined together helped in finding out workable solutions to the problems of the community.

56. There was no problem and complaint of people residing in entire Gonda district, before the beginning of the project. Ans: (e)

57. Purchasing of engine pumps by individual farmers may be beyond their affordable limits. Ans: (a)

58. Earlier farming was not basically one of the professions of people in Gonda district. Ans: (e)

The information provided here is only for refrence. It may vary from the original. www.recruitment.guru

59.By using the same formula upliftment of any other district is possible. Ans: (a)

60.There are very few people who can motivate others in the right direction. Ans: (c)

Directions (61-65)

In each of the following questions, there are two sets of figures. The figures on the top are problem figures (four figures and one question marked space) and those on the bottom are Answer Figures indicated by numbers a, b, c, d and e. A series is established if one of the five answer figures is placed in the ‘question marked space’. Choose the right answer figure

61)

Ans (d)

62)

Ans (a)

The information provided here is only for refrence. It may vary from the original. www.recruitment.guru

63)

Ans (b)

64)

Ans (c)

65)

The information provided here is only for refrence. It may vary from the original. www.recruitment.guru

Ans (a)

Directions (66-70) In each of the following questions, a series begins with unnumbered figure on the extreme left. One of the five numbered figures in the series does not belong to the series. The two unnumbered figures one each on extreme left and extreme right fit into the series. You have to take as many aspects into account as possible of the figures in the series and find out the one and only one of the five numbered figures which does not fit into the series. The number of that figure is the answer

66)

Ans (d)

67)

Ans (e)

The information provided here is only for refrence. It may vary from the original. www.recruitment.guru

68)

Ans (e)

69)

Ans (a)

70)

Ans (d)

71) Directions (71-75) In each of the following questions, which one of the five answer figures on the right should come after the problem figures on the left, if the sequences were continued?

The information provided here is only for refrence. It may vary from the original. www.recruitment.guru

Ans (b)

72)

Ans (c)

73)

The information provided here is only for refrence. It may vary from the original. www.recruitment.guru

Ans (a)

74)

Ans (c)

75)

The information provided here is only for refrence. It may vary from the original. www.recruitment.guru

Ans (e)

Section 2: Quantitative Aptitude

76. The difference between a two digit number and the number obtained by interchanging the positions of its digits is 36. What is the difference between the two digits of that number? (a) 4 (b) 9 (c) 3 (d) Cannot be determined (e) None of these Ans: (a)

77. By how much is two-fifth of 200 greater than three-fifth of 125? (a) 15 (b) 3 (c) 5 (d) 30 (e) None of these Ans: (c)

78. The area of a rectangular field is 460 square metres. If the length is 15 percent more than the breadth, what is breadth of the rectangular field? (a) 15 metres (b) 26 metres (c) 34.5 metres (d) Cannot be determined (e) None of these Ans: (e)

79. Deepa brought a calculator with 30% discount on the listed price. Had she not got the discount, she would have paid Rs.82.50 extra. At what price did she buy the calculator? (a) Rs. 192.50 (b) Rs. 275 (c) Rs.117.85 (d) Cannot be determined. (e) None of these. Ans: (a)

The information provided here is only for refrence. It may vary from the original. www.recruitment.guru

80. How many different ways can be letters of the word SOFTWARE be arranged in such a way that the vowels always come together? (a) 13440 (b)1440 (c) 360 (d) 120 (e) None of these Ans: (e)

81. A motor starts with the speed of 70 kmph with its speed increase in every two hours by 10kmph. In how many hours will it cover 345 Kms? (a) 2 1/4 Hours (b) 4 1/2 Hours (c) 4 Hours 5 Minutes (d) Cannot be determined (e) None of these. Ans: (b)

82. What will be the cost of gardening 1 metre broad boundary around a rectangular plot having perimeter of 340 metres at the rate of Rs. 10per Square Metre? (a) Rs. 3,400 (b) Rs. 1,700 (c) Rs. 3,440 (d) Cannot be determined (e) None of these. Ans: (c)

83. In how many different ways a group of 4 men and 4 women be formed out of 7 men and 8 women ? (a) 2450 (b) 105 (c) 1170 (d) Cannot be determined (e) None of these. Ans: (a)

84. A certain amount earns simple interest of Rs. 1,750/- after 7 years. Had the interest been 2% more, how much more interest would it have earned? (a) Rs. 35 (b) Rs. 350 (c) Rs. 245 (d) Cannot be determined (e) None of these. Ans: (d)

85.Seats for Maths, Physics and Biology are in the ratio of 5:7:8 respectively. There is a proposal to increase these seats by 40%, 50% and 75% respectively. What will be the respective ratio of increased seats? (a) 2:3:4 (b) 6:7:8 (c) 6:8:9 (d) Cannot be determined (e) None of these. Ans: (a)

Directions 86-90 Study the following information to answer these questions

86. Financial information about a company for 2 years

The information provided here is only for refrence. It may vary from the original. www.recruitment.guru

Revenue Expenditure Profit

2006 2007

Which of the following can be inferred from above?

a) The profit in both the years was same because the revenue and expenditure were also the same b) Inspite of revenue being less in 2007, the profit was same as 2006 because the expenditure also was correspondingly less c) The profit and revenue in 2007 were slightly less than that of 2006 d) The profit is entirely related to revenue e) The expenditure is directly related to revenue

Ans (b)

87.

Food type % of fat per 100 gms A 28 B 20 C 13

Which of the following diets would supply the most fat?

a. 200 gms of A and 300 gms of C b. 200 gms of B and 200 gms of A c. 100 gms of G and 300 gms of A d. 300 gms of C and 100 gms each of A and B e. 50gms each of A, B and C

Ans (c)

The information provided here is only for refrence. It may vary from the original. www.recruitment.guru

88. A=x% of y, B=y% of x. Which of the following is true based on above?

a) A is greater than B

b) A is smaller than B

c) Relationship between A and B cannot be determined

d) If x is smaller than y then A is greater than B

e) None of these

Ans (e)

89. Percentage of male, female, literate and illiterate

N=2,50,000

8%

33% Illiterate female Literate male 35% Illiterate male Literate female

24%

What is the difference between number of Literate male and Literate female

a) 75,000 b) 1,500 c) 5,000 d) 500 e) None of these

Ans (c)

90. x= a__ a – 1 y= 1__ a – 1 a) x is greater than y b) y is greater than x only if a<1 c) x is equal to y

The information provided here is only for refrence. It may vary from the original. www.recruitment.guru

d) x is equal to y only if a<1 e) x is greater than y only if a<1

Ans (a)

Directions (91-95)In each of the following questions, two equations are given. You have to solve them and a) if p < q b) if p > q c) if p ≤ q d) if p ≥ q e) if p = q

91. I. p2-7p = -12 II. q2 - 3q + 2= 0

Ans (b)

92. I. 12p2-7p=1 II. 6q2 - 7q + 2= 0

Ans (a)

93 I. p2 + 12p + 35= 0 II. 2q2 + 22q + 56= 0

Ans (c)

94 I. p2-8p + 15 = 0 II. q2 - 5q = -6

Ans (d)

95 I. 2p2 + 20p + 50 = 0 II. q2 = 25

Ans (c)

Directions (96-100) Study the following graph to answer the given questions

Production of 2 companies A & B over the years (Production in Lakh Units)

The information provided here is only for refrence. It may vary from the original. www.recruitment.guru

13 12 11 10 9 8 7 A 6 B 5 4 3 2 1 0 2000 2001 2002 2003 2004 2005 2006 2007

96. For Company A, what is the percent decrease in production from 2000 to 2001?

a) 75

b) 50

c) 25

d) 10

e) None of these

Ans (c)

97. In 2007, the production of Company B is approximately what percent of that of in 2006?

a) 60

b) 157

c) 192

d) 50

e) 92

Ans (b)

The information provided here is only for refrence. It may vary from the original. www.recruitment.guru

98. For Company A, in which year is the percentage increase/ decrease in the production from the previous years is the highest?

a) 2007

b) 2008

c) 2005

d) 2002

e) None of these

Ans (d)

99. What is the difference in the total production of the two companies for the given years?

a) 27,00,000

b) 31,00,000

c) 2,70,000

d) 3,10,000

e) None of these

Ans (a)

100. Which of the following is the closest average production in lakh units of Company B for the given years?

a) 4.2

b) 3.5

c) 4.3

d) 3.75

e) 3.9

Ans (e)

The information provided here is only for refrence. It may vary from the original. www.recruitment.guru

Directions 101-105 Study the following information to answer the given questions.

Percentage of students in various courses (A, B, C, D, E, F) and percentage of girls out of those

Total students: 1,200 (800 girls + 400 boys)

Percentage of various courses

F A 13% 20% E 12%

B 15%

C D 5% 35%

Percentage of Girls in courses

F 14% A 30% E 14%

B 10% D C 30% 2%

101. For course D, what is the respective ratio of boy and girls?

a) 3:4

b) 4:5

c) 3:5

The information provided here is only for refrence. It may vary from the original. www.recruitment.guru

d) 5:6

e) None of these

Ans (a)

102. For which pair of courses is the number of boys the same?

a) E & F

b) A & D

c) C & F

d) B & D

e) None of these

Ans (c)

103. For course E, the number of girls is how much per cent more than the boys for course E?

a) 250

b) 350

c) 150

d) 80

e) None of these

Ans (a)

104. For which course is the number of boys the minimum?

a) E

b) F

c) C

d) A

The information provided here is only for refrence. It may vary from the original. www.recruitment.guru

e) None of these

Ans (d)

105. How many girls are there in course C?

a) 44

b) 16

c) 40

d) 160

e) None of these

Ans (b)

Directions (106-110): In each of these questions a number series is given . Only one number is wrong in each series. You have to find out the wrong number.

106. 10 15 24 35 54 75 100 (a) 35 (b) 75 (c) 24 (d) 15 (e) 54 Ans: (a)

107. 1 3 4 7 11 18 27 47 (a) 4 (b) 11 (c) 18 (d) 7 (e) 27 Ans: (e)

108. 3 2 3 6 12 37.5 115.5 (a) 37.5 (b) 3 (c) 6 (d) 2 (e) 12 Ans: (e)

109. 2 8 32 148 765 4626 32431 (a) 765 (b) 148 (c) 8 (d) 32 (e) 4626 Ans: (d)

The information provided here is only for refrence. It may vary from the original. www.recruitment.guru

110. 2 3 11 38 102 229 443 (a) 11 (b) 229 (c) 102 (d) 38 (e) 3 Ans: (b)

Directions(111-115): Each of the questions below consists of a questions and two statements numbered I and II given below it. You have to decide whether the data provided in the statements are sufficient to answer the questions. Read both the statements and- Give answer (a) if the data in statement I alone are sufficient to answer the questions, while the data in statement II alone are not sufficient to answer the question. Give answer (b) if the data in statement II alone are sufficient to answer the question, while the data in statement I alone are not sufficient to answer the question. Give answer (c) if the data either in statement I alone or in statement II alone are sufficient to answer the questions. Give answer (d) if the data even in both the statement I and II together are not sufficient to answer the question. Give answer (e) If the data in both the statements I and II together are necessary to answer the question.

111. By selling a product for Rs.100/- how much profit was earned ? (I) 20% profit would have been earned if it had been sold for Rs.90/- (II) The profit was one-third of the purchase price. Ans: (c)

112. A train crosses another train running in the opposite direction in x seconds. What is the speed of the train? (I). Both the trains are running at the same speed . (II) The first train is y cm long. Ans: (d)

113. The difference between the two digits of a number is 6. What is the number ? (I). The digit at the unit place is bigger than the other digits . (II). The sum of the two digits is 12. Ans: (e)

114. X, Y and Z are integers. Is X an odd number? (I). An odd number is obtained when X is divided by 5. (II) (X + Y) is an odd number. Ans: (a)

115. What is the capacity of a cylindrical tank? (I) Radius of the base is half of its height which is 28 metres. (II) Area of the base is 616 sq. metres and its height is 28 metres. Ans: (c)

The information provided here is only for refrence. It may vary from the original. www.recruitment.guru

Directions (116-120): Study the following table to answer the given questions. Percentage of Marks Obtained by Seven Students in Six Subjects

English History Computers Maths Sciences Economics Student (60) (40) (130) (150) (120) (80) Meera 100 80 50 90 90 60 Subodh 80 70 80 100 80 40 Kunal 90 70 60 90 70 70 Soni 60 60 65 80 80 80 Richu 50 90 62 80 85 95 Irene 40 60 64 70 65 85 Vijay 80 80 35 65 50 75

116. What are the total marks obtained by Meera in all the subjects? (a) 448 (b) 580 (c) 470 (d) 74.67 (e) None of these Ans: (a)

117. What are the average marks obtained by these seven students in History ? ( rounded off to two digits) (a) 72.86 (b) 27.32 (c) 24.86 (d) 29.14 (e) None of these Ans: (d)

118.How many students have got 60% or more marks in all the subjects? (a) One (b) Two (c) Three (d) None (e) None of these Ans: (b)

119.What is the overall percentage of Kunal? (a) 64 (b) 65 (c) 75 (d) 64.24 (e) None of these Ans: (c)

120. In which subject is the overall percentage the best? (a) Maths (b) Economics (c) History (d) Science (e) None of these. Ans: (a)

Directions (121-125): Find out the approximate value which should replace the question mark (?) in the following questions, (You are not expected to find out the exact value)

121.95 3.7 ÷95 0.9989 =95? (a) 1.9 (b) 3 (c) 2.99 (d) 3.6 (e) 2.7 Ans: (e)

122.√1000+ 3.001 of 1891.992 =? 4.987

The information provided here is only for refrence. It may vary from the original. www.recruitment.guru

(a) 2500 (b) 1230 (c) 1640 (d) 1525 (e) 2130 Ans: (b)

123. 0.0004÷0.0001×36.000009=? (a) 0.10 (b) 1.45 (c) 145 (d) 14.5 (e) 1450 Ans: (e)

124. 137% of 12345 = ? (a) 17000 (b) 15000 (c) 1500 (d) 14300 (e) 900 Ans: (a)

125. 3739 + 164 ×27 = ? a) 105400 (b) 4000 (c) 8200 (d) 690 (e) 6300 Ans: (c)

Section 3: English Language

English Language

Directions(126- 135) Read the following passage to answer the given questions. Some words have been printed in bold to help you to locate them while answering some of the questions.

We tend to be harsh on our bureaucracy, but nowhere do citizens enjoy dealing with their government. They do it because they have to. But that doesn't mean that the experience has to be dismal. Now there is a new wind blowing through government departments around the world, which could take some of this pain away. In the next five years it may well transform not only the way public services are delivered but also the fundamental relationship between government and citizens. Not surprisingly, it is the Internet that is behind it. After e-commerce and e-business, the next revolution may be e-governance. Examples abound. The municipality of phoenix, Arizona, allows its citizens to renew their car registrations, pay traffic fines, replace lost identity cards etc. Online without having to stand in endless queues in a grubby municipal office. The municipality is happy because it saves $5 a transaction-it costs only $1.60 to process an online transaction versus $6.60 to do it across the counter. In Chile, people routinely submit their income tax returns over the Internet, which has increased transparency, drastically reduced the time taken and the number of errors and litigation with the tax department. Both tax payers and the revenue department are happier. The furthest ahead, not surprisingly, is the small, rich and entrepreneurial civil service of Singapore, which allows citizens to do more functions online than any other. As in many private companies, the purchasing and buying of Singapore's government departments is now on the web, and cost benefits come through more competitive bidding, easy access to global suppliers and time saved by online processing of orders. They can post their catalogues on their site, bid or contracts submit in voices and check their payment status over the Net. The most useful idea for Indian municipalities is Gov works a private sector- run site that collects local taxes, fines, and utility bills for 3,600 municipalities

The information provided here is only for refrence. It may vary from the original. www.recruitment.guru

across the United States. It is a citizen's site, which also provides information on Government jobs, tenders, etc. The most ambitious is the British Government, which has targeted to convert 100 percent of its transactions with its citizens to the internet by 2005. Cynics in India will say, 'Oh, e-government will never work in India. We are so poor and we don't have computers.' But they are wrong! There are many experiments afoot in India as well. Citizens in Andhra Pradesh can download government forms and submit applications on the Net without having to bribe clerks. In many districts, land records are online and this has created transparency. Similarly, in Dhar district to Madhya Pradesh, villagers have begun to file applications for land transfers and follow their progress on the Net. In seventy village in the Kolhapur and Sangli district in Maharashtra, internet booths have come up where farmers can daily check the market and rates of agricultural commodities in Marathi, along with data on agricultural schemes, information on crop technology, when to spray and plant their crops and bus and railway time tables. They also find vocational guidance in jobs, application for ration cards, kerosene/ gas burners and a land record extracts with details of land ownership. Sam Pitroda's World Tel, Reliance Industries and the Tamil Nadu Government are jointly laying 3000km of optic fibre cables to create a Tamil network which will offer ration cards, school College and hospital admission forms, land records, and pension records. If successful, World Tel will expand the network to Gujarat, Karnataka and West Bengal. In Kerala, all the villages are getting linked online to the district headquarters, allowing citizens to compare the development priorities of their village with other villagers in the state. Many are still skeptical of the real impact because so few Indians have computers. The answer lies in interactive cables. TV and in Internet kiosks. Although India has only five million computers and thirty-eight million telephones, it has thirty-four million homes with cable TV and these are growing eight percent a year. By 2005 most cable homes will have access to the internet from many of the 700000 local STD/PCO Booths. Internet usage may be low today, but it is bound to grow rapidly in the future, and e-governance in India may not be a dream.

126. According to the passage, which country has the most ambitious plan for e- governance ? (a) USA (b) Chile (c) Singapore (c) India (e) UK Ans: (c)

127. Gov Works in which of the following countries ? (a) India (b) UK (c) Chile (d) Singapore (e) None of these Ans: (e)

128. Choose the word that is opposite in meaning of the word "dismal" as used in the passage. (a) grim (b) approve (c) pleasing (d) better

The information provided here is only for refrence. It may vary from the original. www.recruitment.guru

(e) enrich Ans: (c)

129. How can India overcome low penetration of computers for e-governance ? (a) By manufacturing more computers (b) Through cable TV and Internet Kiosks (c) By opening more STD/ PCO booths (d) By making the Internet free (e) By putting more services on internet Ans: (e)

130. Which of the following has not been one of the effects of submitting income tax return over internet in Chile ? (a) Reduction of legal cases (b) Reduction in errors (c) Increase in transparency (d) Increase in no. of returns (e) Reduction in time taken Ans: (d)

131. Choose the word that is same in meaning as the word "abound" as used in the passage. (a) around (b) proliferate (c) flourish (d) plentiful (e) few Ans: (d)

132.Choose the word that is same in the meaning as the word "post" as used in the passage (a) deliver (b) send (c) put up (d) drop out (e) later Ans: (b)

133. According to the passage which country is at present the most advanced in e- governance ? (a) Singapore (b) Chile (c) India (d) USA (e) UK Ans: (a)

134. In which direction is the new wind blowing ? (a) More and more interaction of citizens with government through internet (d) Outsourcing the work of infrastructure creation for internet (c) Increasing the penetration of computers in rural areas (d) Integrating e-commerce, e- business and e-governance (e) Introducing e-governance programmes in schools and colleges. Ans: (a)

The information provided here is only for refrence. It may vary from the original. www.recruitment.guru

135. According to the passage, what is the annual growth rate of computer in India ? (a) 8% (b) 5% (c) 0.5% (d) Not mentioned (e) None of these Ans: (d)

Directions (136-140): In each of the following questions four words are given of which two words are most nearly the same or opposite in meaning. Find the two words which are most nearly the same or opposite in meaning and indicate the number of the correct letter combination.

136. (A) Proximate (B) Elevated (C) Nimble (D) Agile (a) A-B (b) B-C (c) A-C (d) B-D (e) C-D Ans: (e)

137. (A) Notion (B) Symbol (C) Concept (D) Message (a) A-B (b) B-C (c) A-C (d) C-D (e) B-D Ans: (e)

138. (A) Limpid (B) Luscious (C) Acrid (D) Benign (a) A-B (b) B-C (c) C-D (d) B-D (e) A-C Ans: (b)

139. (A) Asinine (B) Furious (C) Fortunate (D) Ridiculous (a) A-D (b) A-B (c) A-C (d) B-C (e) B-D Ans: (a)

140.(A) Companion (B) Amateur (C) Adept (D) Adherent (a) B-C (b) B-D (c) C-D (d) A-B (e) A-D Ans: (e)

141. (A) Squander (B) Disunite (C) Flicker (D) Preserve (a) A-C (b) B-C (c) A-D (d) C-B (e) B-D Ans: (c)

142.(A) Mitigate (B) Acquiesce (C) Relinquish (D) Duplicate (a) A-C (b) B-D (c) A-B (d) C-D (e) B-C Ans: (e)

143.(A) Fable (B) Legend (C) Portrayal (D) Contract (a) A-C (b) B-C (c) B-D (d) A-B (e) C-D Ans: (d)

144.(A) Occurrence (B) Pretence (C) Profusion (D) Extravagance (a) B-D (b) C-D (c) B-C (d) A-C (e) A-B Ans: (b)

The information provided here is only for refrence. It may vary from the original. www.recruitment.guru

145.(A) Excellent (B) Passionate (C) Apathetic (D) Discrepant (a) A-B (b) A-C (c) A-D (d) B-C (e) C-D Ans: (d)

Directions(146-150): Rearrange the following eight sentences (A), (B), (C), (D), (E), (F), (G) and (H) in the proper sequence to form a meaningful paragraph; then answer the questions given below them.

(A) But I always felt somewhere in my mind that I loved acting (B) He never wanted me to be an actor, as he didn't look upon theatre or acting as respectable vocation (C) Firstly there was no tradition of theatre in my family (D) I am talking specifically of acting, not theatre in general. (E) My parents were old-fashioned (F) I will answer all your queries a little elaborately (G) Let alone theatre, arts in general had no place of respect in my family (H) My father was a government servant

146. Which of the following will be the LAST sentence? (a) G (b) C (c) H (d) D (e) E Ans. (d)

147. Which of the following will be the FIRST sentence? (a) A (b) B (c) D (d) C (e) F Ans. (e)

148. Which of the following will be the SIXTH sentence? (a) B (b) C (c) D (d) D (e) E Ans. (a)

149. Which of the following will be the FOURTH sentence? (a) D (b) E (c) A (d) B (e) G Ans. (b)

150. Which of the following will be the THIRD sentence? (a) H (b) E (c) G (d) B (e) A Ans. (c)

Directions (151-155) : Read each sentences to find out whether there is any error in it. The error, if any, will be in one part of the sentence. The number of the part is the answer. If there is no error, the answer is 'e'. (Ignore errors of punctuation, if any)

151. Many animals and plants live in water (a)/ but not in the same kind of water (b)/ because not all water is the same. (c)/ Sea water, for instance, contains a lot of salt, fresh water contains very little. (d)/ No error (e) Ans. (a)

152. A sparrow has made a nest in Kesho's house (a)/ and had laid eggs. Both Kesho and his sister Shyama (b)/ watched the nest for hours every day. (c)/ Even

The information provided here is only for refrence. It may vary from the original. www.recruitment.guru

meal times were forgotten. (d)/ No error (e) Ans. (b)

153. A skilful advertiser may be able to create (a)/ practically a monopoly for himself. (b)/ not because his product is superior to (c)/ but because he has succeeded in inducing people to believe that it is (d)/ No error (e) Ans. (b)

154. Whatever may be the origin of speech, (a)/ we can be certain that man did not began (b)/ to feel the need to speak (c)/ until he began to live in communities. (d)/ No error (e) Ans. (b)

155. Each animal of the same species (a)/ looks for the same sort of food. (b)/ Also, there may be other animals of different species (c)/ competing with the same food. (d)/ No error (e) Ans. (e)

Directions (156-160) : Pick out the most effective pair of words from the given pair of words to make the sentence/s meaningfully complete.

156. Of all the problems that have ...... human beings since the beginning of recorded history, perhaps the most significant has been the ...... of their own nature (a) encountered...... importance (b) perplexed...... value (c) questioned...... scope (d) confronted...... riddle (e) directed...... issue Ans. (d)

157. Self-concept and self-esteem are crucial to personal and professional effectiveness because if they are not fully ...... I may act in mystifying and sometimes...... ways (a) positive...... destructive (b) developed...... proactive (c) nurtured...... reactive (d) nourished...... listless (e) devised...... vulnerable Ans. (c)

158. A key factor in...... an effective and fulfilling life in our complex society is the ability to...... control over our actions. (a) achieving...... exert (b) obtaining...... energize (c) projecting...... restrain (d) providing...... admonish (e) expressing...... withhold Ans. (a)

159. A theory is...... evaluated to the degree that its concepts...... to themselves a verification by independent investigators (a) critically...... pertain (b) positively...... lend (c) obviously...... yield

The information provided here is only for refrence. It may vary from the original. www.recruitment.guru

(d) alarmingly...... introduce (e) delightfully...... submit Ans. (a)

160. It is often the ...... that everyone agrees about objectives; the disagreement...... about exactly how they are to be achieved (a) view...... follows (b) situation...... talks (c) matter...... projects (d) happening...... matters (e) case...... arises Ans. (e)

Directions (161-165) : Which of the phrases (a), (b), (c) and (d) given below should replace the phrase given in bold in the following sentence grammatically correct.. If the sentence is correct as it is & no correction is required, mark (e) as the answer.

161. He went to the blast site to save his colleague who was stranding there (a) had been stranded (b) had to strand (c) was being stranding (d) was been stranding (e) No correction required Ans. (a)

162. When I receive the letter, the date for the interview was already over (a) I received the letter (b) the letter I had received (c) I had to receive letter (d) I was receiving (e) No Correction required Ans. (a)

163. Everyone is requested to attend the dinner party hosted by the president (a) is requesting to (b) is requested to (c) will request to (d) is to requested to (e) No correction required Ans. (e)

164. The chairman approved the recommendations to the committee with partial modifications (a) by the committee for (b) of the committee with (c) to the committee (d) at the committee in (e) No correction required Ans. (b)

165. Had the opportunity been given to him he will have proved successful (a) will have to prove (b) will be proving (c) would have proved (d) would have proven (e) No correction required Ans. (c)

(Directions 166-170) : In the following passage there are blanks each of which has been numbered. These numbers are printed below the passage and against each five words are suggested, one of which fits the blank appropriately. Find out, the appropriate word in each case.

In an upper class drawing room in Mumbai, a dozen or so (166) the city's intelligentsia is (167) on the dregs of what was a live-wire party an hour ago.

The information provided here is only for refrence. It may vary from the original. www.recruitment.guru

Poets, editors, writers, filmmakers, all card-carrying members of the rich and powerful, not at this late hour, on their ninth drink, are (168) in what Indians are best known to do: Media-bashing, armchair criticism and name-calling.

Inevitably the talk turns to Tehelka, and its managing director and editor-in-chief, Tarun Tejpal, and the party quickly degenerates into an orgy of guilt and shame and that peculiar habit of the Indian intelligentsia : passing the buck, with every one present attacking his neighbour for not supporting Tejpal more. It is a trait Tejpal is (169) with and has been telling me about in the weeks preceding this interview.

"Often at parties when someone (170) me 'great job, carry on what you are doing', and showers with me staggering praise, staggering love, staggering (171) just smile and let it pass as i don't want to score a brownie point and say, 'and what about your boss, what will you do?"

This lightly-tossed sentence with the slightest hint of a dark chuckie is the only time I have been Tejpal allow for bitterness, though I have been (172) him for weeks for sings of it. Because for a man who has been the victim of the most (173) government witch-hunts in recent years, Tejpal is astonishingly devoid of (174) . You expect him to rage, to spew fire, to heap scorn and anger at his enemies, and instead, what you encounter, is a man in whose eyes you see only compassion and a weary understanding of the nature of the beast.

The abiding irony in all this is that unlike his armchair supporters in the middle class, who rave and rant on his behalf, but do nothing else, Tejpal displays forbearance. " The fact that I am essentially a literary animal and that my entire sensibility has been shaped by literature has helped. I'm not a black-and-white person. I'm a (175) who understand the greys", he says to me at a coffee shop, over tea.

166. (a) of (b) in (c) into (d) off (e) from

167. (a) feed (b) feeds (c) fed (d) feeding (e) feeded

168. (a) mixing (b) lending (c) scaring (d) indulging (e) pushing

169. (a) ordinary (b) familiar (c) strange (d) free (e) routine

170. (a) bold (b) said (c) ask (d) say (e) tells

171. (a) delight (b) contempt (c) worship (d) affection (e) admiration

172. (a) lure (b) teasing (c) baiting (d) harassing (e) matching

173. (a) gentle (b) good (c) various (d) vicious (e) wicked

174. (a) forbearance (b) anger (c) wrath (d) forgiveness (e) gale

The information provided here is only for refrence. It may vary from the original. www.recruitment.guru

175. (a) boy (b) guy (c) youth (d) human being (e) follow

Section 4: General Awareness

176. On which of the following dates was the National Centre for Promotion of Employment for Disabled People-Shell Helen Keller Awards-2007 presented? (a) December 1, 2007 (b) December 2, 2007 (c) December 3, 2007 (d) December 4, 2007 (e) None of these Ans. (b)

177. The nuclear power plant of 220 MW which was recommissioned on December 2, 2007, after 'en-ass coolant channel replacement (EMCCR)' is______(a) Kalpakam (b) Kota (c) Narora (d) Rawatbhatta (e) None of these Ans. (c)

178. The former South African cricketer who, on December 4, 2007 signed two- year contract with Board of Control for Cricket in India (BCCI) coach the Indian cricket team is ______(a) Boeta Dippenar (b) Jack Russel (c) Allan Donald (d) GaryKirsten (e) None of these Ans. (d)

179. Which of the following Indian cities hosed the East Asia Gender Equality Ministerial Meeting, in December 2007, attended by ministerial level representatives from several countries and various United Nations agencies? (a) New Delhi (b) Chennai (c) Kolkata (d) Mumbai (e) None of these Ans. (a)

180. South Africa's former Deputy President who, on December 19, 2007 defeated Thabo Mbeki, to become the new President on African National Congress, at a party conference the northern town of Polokwane is (a) Susan Booysen (b) Nelson Zulahe (c) Jacob Zuma (d) Momir Williams Junior (e) None of these Ans. (c)

181. Which of the following countries, for the first time, attended a summit the six Arab countries belonging the Gulf Cooperation Council (GCC) in Doha, Qatar, in December 2007? (a) Egypt (b) Syria (c) Iran (d) Iraq (e) None of these Ans. (c)

182. United Russia which won the Russian Parliamentary elections in December 2007, captured______seats the 450-seat Duma, the Lower House of Russia's Parliament.

The information provided here is only for refrence. It may vary from the original. www.recruitment.guru

(a) 290 (b) 315 (c) 367 (d) 410 (e) None of these Ans. (b)

183. The 4300-year-old Chinese oil capital of China's oldest Liangzhu Culture, which was dug up by archaeologists in December 2007 is located in eastern China's ______province (a) Fujian (b) Zhejiang (c) Anhui (d) Jiangsu (e) None of these Ans. (b)

184. With which of the following companies did Reliance Industries, on December 4, 2007 sign a Memorandum of Understanding (MoU) for joint-co-operation in Petrochemicals? (a) Oil India Ltd (b) Indian Oil Corporation (c) GAIL (India) Ltd (d) Hindustan Petroleum Corporation Ltd (e) None of these Ans. (c)

185. The Central Government, in December 2007, allowed ______based Dolce & Gabbana and India's realty firm DLF to set up a joint venture in single brand retailing of lifestyle products (a) Italy (b) France (c) Britain (d) USA (e) None of these Ans. (a)

186. Which of the following companies, on December 4, 2007 signed a 'large Account Reseller Agreement' with Microsoft Corporation India to provide integrated technology solution to Indian enterprises? (a) Dell (b) Lenovo (c) Compaq (d) Toshiba (e) None of these Ans. (a)

187. Which of the following Indian Companies, in December 2007, opened 160 stores to its specialty beauty, health and wellness retail business subsidiary, H & B Stores, under the new brand name "new-u"? (a) Reliance (b) Dabur (c) Unilever (d) Subhiksha (e) None of these Ans. (b)

188. Whom did Neha Aggarwal of Delhi beat in the finals to win the youth girls' crown in the national youth and junior table tennis championship in Indore on December 21, 2007? (a) Madhurika Patkar (b) Ashelsha Bodas (c) Priti Mokashi (d) Pooja Sahasrabudhe (e) None of these Ans. (d)

189. The tennis player from Andhra Pradesh who, on December 22, 2007, beat Divij Sharan of Delhi, to win the men's singles title in the 2007 National grasscourt championship in Kolkata is : (a) J. Vishnu Vardhan (b) Vinod Sridhar (c) V. M. Ranjeeth (d) N. Sriram

The information provided here is only for refrence. It may vary from the original. www.recruitment.guru

Balaji (e) None of these Ans. (a)

190. The 30-year-old Armyman form Maharashtra who, on December 23, 2007 won the men's event in the Mawana Sugars Indian Open Marathon in New Delhi is ______(a) Angad Kumar (b) Rashpal Singh (c) Satya Prakesh (d) Birender Singh Chahar (e) None of these Ans. (c)

191. Which of the following countries in December 2007, won the Davis Cup (tennis) in Portland (U. S.) after a long gap of twelve years? (a) Russia (b) United States of America (c) Sweden (d) Germany (e) None of these Ans. (b)

192. Miss Angola who came first runner-up in the Miss World 2007 pageant held in Sanya, China, in December 2007 is (a) Moran Gordillo (b) Micaela Reis (c) Sandy Dudette (d) Adyshe Barbara (e) None of these Ans. (b)

193. Which of the following South American Countries on December 1, 2007 threatened to halt the OPEC nation's oil sales to the United States is (a) Venezuela (b) Peru (c) Chile (d) Brazil (e) None of these Ans. (a)

194. The renowned sitar maestro of Dharwad, contemporary of the legendry Hindustani vocalist the late Ustad Abdul Kareem Khan Saheb, who passed away in Belgaum on December 2, 2007 is______(a) Maznoo Mia (b) Allah Rakha Khan (c) Bale Khan (d) Aftab Rasool (e) None of these Ans. (c)

195. Who was elected President of the Ramakrishna Math and Mission at a meeting of the Board of Trustees of the Match and the governing body of Mission in Kolkata on December 3, 2007? (a) Atmasthananda (b) Ramananda (c) Parmananda (d) Vijnananada (e) None of these Ans. (a)

196. The Executive Director of the Global Fund to Fight AIDS, Tuberculosis and Malaria, who on December 20, 2007 renewed its grant to India, worth more than $ 100 million for the next three years till 2010 is ______(a) Michael Kazatchleine (b) Francois Dippennar (c) Alex Mulromy (d) Deniel Mozzart (e) None of these Ans. (a)

The information provided here is only for refrence. It may vary from the original. www.recruitment.guru

197. "Hand-in-Hand 2007" the first-ever joint Sino-Indian anti-terrorism training exercise was held at the Kuming Military academy in China in December 2007. Kuming is the capital of south-western ____ province (a) Hunan (b) Yunnan (c) Shandong (d) Zheziang (e) None of these Ans. (b)

198. The ten-year old Indian boy who, in December 2007, emerged as the youngest champion in the history of 'Junior Mastermind; a BBC Quiz show is ______(a) Swapnil Shankar (b) Madhav Prasoon (c) David Varghese (d) Krishna Routrey (e) None of these Ans. (c)

199. Bharatiya Janata Party came back to power for the consecutive ______time in Gujarat on December 23, 2007 the day when the result of the Assembly elections were declared? (a) second (b) third (c) fourth (d) fifth (e) None of these Ans. (c)

200. Whom did Nick Clegg replace to become the new leader of Great Britain's Liberal Democratic Party in London on December 18, 2007? (a) Richard Keen (b) Marshall Daniel (c) Menzies Champbell (d) Alistaire Harrod (e) None of these Ans. (c)

201. Right-wing South Korean businessman who, on December 19, 2007 defeated left-of-centre Chung Dong-young to become the new President of South Korea is (a) Kim Yoon-Ok (b) Lee Myung-bak (c) Ahn Sang-sao (d) Mile Jen- foo (e) None of these Ans. (b)

202. An earthquake of 6.8 magnitude on the Richter scale struck New Zealand's North Island on December 20, 2007 causing massive damage in the city of ______(a) Auckland (b) Hamilton (c) Napier (d) Gisborne (e) None of these Ans. (d)

203. The passport-for Schengen Area of Europe saw its biggest-ever enlargement on December 21, 2007 when______new European Union (E. U.) members began to implement the Schengen Agreement (a) five (b) seven (c) nine (d) eleven (e) None of these Ans. (c)

204.GE Energy, on December 21, 2007 bagged its largest order for supply of Jenbacher gas engines to support a major rural electrification initiative in (a) Kenya (b) Bangladesh (c) Somalia (d) Ecquador (e) None of these Ans. (b)

The information provided here is only for refrence. It may vary from the original. www.recruitment.guru

205. In December 2007, Nasdaq opened an office in the Asian capital-city of ______aimed at stepping up efforts at attracting more firms to list on the exchange and forging deeper ties with bourses and regulators (a) Seoul (b) Tokyo (c) Beijing (d) Manila (e) None of these Ans.(c)

206. In which of the following capital cities, of Gulf Cooperation Council countries, did Union Bank of India open its representative office in December 2007? (a) Riyadh (b) Doha (c) Kuwait City (d) Abu Dhabi (e) None of these Ans. (d)

207. The ______based Prudential Finance (INC (PFI), on December 4, 2007 announced an asset management joint venture with real estate giant DLF with which it already has a life insurance partnership (a) Canada (b) U. S. A. (c) South Africa (d) Singapore (e) None of these Ans. (b)

208. Which of the following teams was defeated by Chennai Superstars to win the ICL () Twenty-20 final in Panchkula on December 16, 2007? (a) Chandigarh Lions (b) Kolkata Tigers (c) Hyderabad Heroes (d) Delhi Jets (e) None of these Ans. (a)

209. Who, among the following golfers, won the Target World Challenge Cup in Thousand Oaks on December 16, 2007? (a) Zach Johnson (b) Lee Westwood (c) Tiger Woods (d) Henrik Stenson (e) None of these Ans. (c)

210. The American cyclist, stripped of his 2006 Tour de France title for doping, who was banned from all competitions in France by the country's anti-doping agency AFLD, on December 19, 2007 is ______(a) Fabio Lefebore (b) John Carragher (c) Desmond Capello (d) Floyd Landis (e) None of these Ans. (d)

211. Whom did Zubin Kumar beat in the finals to win the youth boys' crown in the National youth and junior table tennis championship in Indore on December 21, 2007? (a) Devesh Karia (b) G. Sathiyan (c) Raj Mondal (d) Satyajit Prasanna (e) None of these Ans. (a)

212. The French Minister for Foreign and European Affairs who visited India in December 2007 to prepare the ground for French President Nicolas Sarkozy's visit

The information provided here is only for refrence. It may vary from the original. www.recruitment.guru

in January 2008 is (a) Bernard Kouchner (b) Andrei Strahler (c) William Tarbuck (d) Philip Randell (e) None of these Ans. (a)

213. Hollywood star in box-office hits like 'Monster-in-Law', fitness guru and activist who turned seventy years old on December 21, 2007 is ______(a) Olivia Silverstone (b) Lisa Marigold (c) Jane Fonda (d) Elizabeth Turner (e) None of these Ans. (c)

214. Which of the following countries on December 23, 2007 conclude the annual central rural work conference with a pledge to improve rural infrastructure, promote stable development of agriculture and facilitate a sustained income growth for farmers in the year 2008? (a) Brazil (b) China (c) Canada (d) Argentina (e) None of these Ans. (b)

215. The former British Prime Minister who, in December 2007, was welcomed into Britain's Catholic community after his/ her conversion to the Catholic Church is______(a) John Major (b) Margaret Thatcher (c) Rowan Williams (d) Tony Blair (e) None of these Ans. (d)

216. After 16 months of marathon hearing the inquiry into the 1985 Kanishka bombing ended in Toronto on December 13, 2007. The bombing of the Air India flight killed ______people (a) 117 (b) 233 (c) 329 (d) 409 (e) None of these Ans. (c)

217. Which of the following countries, on December 17, 2007, expressed its interest in investments by Indian corporates for construction and promotion of budget hotels in the island nation? (a) Sri Lanka (b) Mauritius (c) Maldives (d) Tuvalu (e) None of these Ans. (a)

218. The former managing editor of SAGE India and longtime Doordarshan news reader who died in Landour (above Mussorie) on December 16, 2007 is (a) Pankaj Singh (b) Rajesh Sharma (c) Mohanlal Chaturvedi (d) Tajeshwar Singh (e) None of these Ans. (d)

219. The editor of Jaipur-based monthly journal, "Sharad Krishi" who was on December 18, 2007 presented the prestigious Atmaram Prize for developing scientific and technical literature in Hindi is (a) Mahendra Madhup (b) Anupam Shrivastava (c) Dhirendra Singh

The information provided here is only for refrence. It may vary from the original. www.recruitment.guru

Parmar (d) Gaurav Bainsala (e) None of these Ans. (a)

220. The Supreme Court of ______sentenced the former President, Alberto . K. Fujimori, on December 11, 2007 sentenced the former President K. Fujimori to six years in prison for ordering an illegal search as his government was collapsing in 2000 (a) Chile (b) Peru (c) Colombia (d) Mexico (e) None of these Ans. (b)

221. The United States and China, in December 2007, held the Sino-U. S. Strategic Economic Dialogue (SED) in (a) Beijing (b) New York (c) Oslo (d) Seattle (e) None of these Ans. (a)

222. Thousands of workers in which of the following countries, on December 15, 2007 march through the national capital to protest against the pressures on life and fuel and food prices due to the failure of the political leaders to form a government after the June 2007 general elections? (a) Slovakia (b) Czech Republic (c) Romania (d) Belgium (e) None of these Ans. (d)

223. The well-known British born science fiction writer and author of the book "2001 : A Space Odyssey" who was honoured by the Sri Lankan government, on December 16, 2007 for his contribution to the world of space exploration is______(a) Daniel Bald (b) Arthur Clarke (c) Anthony Cordesman (d) Boris Schlesinger (e) None of these Ans. (b)

224. Which of the following car companies in India announced a special savings scheme______'Happy Customer Offer' - in December 2007? (a) Maruti Suzuki (b) Tata (c) Ford (d) Hyundai (e) None of these Ans. (a)

225. The Eleventh Plan Document approved by the National Development Council (NDC) on December 19, 2007 envisages a growth of _____ percent during 2001- 12 the terminal year of plan period. (a) Eight (b) Nine (c) Ten (d) Eleven (e) None of these Ans. (c)

The information provided here is only for refrence. It may vary from the original. www.recruitment.guru

Explanations

(1-5): @ = > * = > # = =

$ = < + = <

1. Statements D + T, E $ V, F * T, E @ D After conversion D < T, E < V, F > T, E > D Or V > E > D < T < F Conclusion I . D$ V = D , < V : Not true D is similar than V. II. D + F + = D < F : True 2. Statements B + D, E $ T, T * P, P @ B After conversion B < D, E < T, T > P, P > B Or, E < T > P > B < D Conclusion I . P $ D = P < D : Not true II. P @ D = P > D : Not true While considering the relation between two entities three possibilities exist: greater than, equal to or smaller than. Therefore either I or II is true. 3. Statements T * U, U $ W, V @ L, W + V After conversion T> U, U < W, V > L, W< V Or T > U < W < V > L Conclusion I. V @ T = V > T : Not true II. L # W = L > W : Not true 4. Statements P $ Q, N # M, M @ R, R * P After conversion P < Q, N = M, M > R, R > P Or N = M > R > P > Q Conclusion I. P + N = P < N : Not true II. Q $ W = Q < M : Not true 5. Statements E * F, G $ H, H # E,G @ K After conversion E > F, G < H, H = E, G > K Or , K < G F Conclusion I. H @ K = H > K : Not true II. H $ F = H > K : Not true

The information provided here is only for refrence. It may vary from the original. www.recruitment.guru

(6-11):

(i). There are altogether 27 elements in the above sequence.

(ii). There are only 14 letter in the above sequence.

(iii) There are only 9 digits in the above sequence.

(iv) There are only 4 symbols in the above sequence.

(v) The middle terms of the sequence is P.

6. 5 +6 M +6 T +6 1 E +6 J +6 U +6 H $ +6 P +6 7 +6 Q Therefore, ? = TU7 7. 11th from left and =1 7th from right end = 1 Remaining elements between I and 1: K * P T @ U 9 A 7

Middle Term 8. 6th to the right of20th from the right end means 20-6 =14th from the right end. There are altogether 27 elements in the above sequence and the middle term, i.e., Therefore, our required answer is option. 9. Digit Digit Digit

There is no such combination.

10. 8th to the left of 13th element from right end means 13 + 8 =21st from the right end. 21st element from the right end is equivalent to 28 – 21 = 7th elements from the left and vice-versa. 1st 15th 2nd 14th 3rd 13th 4th 12th 5th 11th 6th 10th 7th 9th Therefore, required element would be 9th from the left in the original sequence. 9th from left --> M. 11. According to question, A B C D E F G H I J K L M N O

5 2 4 6 5 8 10 12 5 14 16 18 20 22 5

P Q R S T U V W X Y Z

24 26 28 30 32 5 35 36 38 40 42

Now,

The information provided here is only for refrence. It may vary from the original. www.recruitment.guru

C U S T O M

4 + 5 + 30 +32 + 5+20 = 96 12. a + b < x + y 5

Such combinations are :

2 + 4 < 5 + 4 4+ 5 < 4 + 8 5 5

8+ 7 < 9 + 8 5

13. Option (1) P x Q ÷ R + S – T P x Q --> P is the wife of Q Q ÷ R --> Q is the father of R R + S * R is the sister of S S – T --> S is the brother of T Note. The sex of T is not known Deductions (i) Q is the husband of P (ii) P is the mother of R, S and T (iii) Q is the father of R,S and T (iv) R is the sister of S and T (v) S is the brother of R and T

Option (2) P x Q --> P is the wife of Q Q ÷ R --> Q is the father of R R- T --> R is the brother of T T + S --> T is the sister of S Note: The sex of S is not known Deductions (i) Q is the husband of R (ii) P is the mother of R,S and T (iii) Q is the father of R, S and T (iv) T is the daughter of P and Q 14. According to questions, the new sequence would be: N O P Q R S T U V W X Y Z A B C D E F G H I J K L M 9th to the right of 17th letter from the right means (17-9) =8th letter from the right. 8th letter from right = F

(15-18): On the basis of given information and conclusion as well as sub-conclusion drawn from them we can construct the following chart: P Field Destination P Finance Lucknow

The information provided here is only for refrence. It may vary from the original. www.recruitment.guru

Q HRM _____ R ____ Calcutta Bangalore S ____ Calcutta Bangalore T Administration Chennai U Marketing ____ W System Delhi

19. N U R S I N G

There are three such pairs. (20-26): On careful analysis of the given passcodes it is evident that the last word of the previous code becomes the first word for the next batch and the first and the second words shifted to the second and the third positions respectively. Again the seventh and the sixth words occupy the fourth and fifth positions respectively and then the third, the fourth and the fifth words of the previous code are written in the same order in the passcode for the next batch.

Passcode for Batch I

1 2 3 4 5 6 7 8

Cloths neat and clean liked are all by

Passcode for Batch II

8 1 2 7 6 3 4 5

By cloths neat all are and clean liked

1 2 3 4 5 6 7 8

Passcode for Batch III.

8 1 2 7 6 3 4 5

Liked by cloths clean and neat all are

1 2 3 4 5 6 7 8

Passcode for Batch IV

8 1 2 7 6 3 4 5

Are liked by all neat cloths clean and

1 2 3 4 5 6 7 8

The information provided here is only for refrence. It may vary from the original. www.recruitment.guru

Passcode for Batch V

1 2 3 4 5 6 7 8

And are liked clean cloths by all neat

1 2 3 4 5 6 7 8

Passcode for Batch VI

1 2 3 4 5 6 7 8

Neat and are all by liked clean cloths

Now on the basis of Above analysis we can summarise the procedure for getting passcode for each subsequent batch with reference to the passcode fro batch I in the following manner.

Passcode for Batch I

1 2 3 4 5 6 7 8

Clothes neat and clean liked are all by

Passcode for Batch II

8 1 2 7 6 3 4 5

By cloths neat all are and clean liked

Passcode for Batch III

5 8 1 4 3 2 7 6

Liked by cloths clean and neat all are

Passcode for Batch IV

6 5 8 7 2 1 4 3

Are liked by all neat cloths clean and

Passcode for Batch V

3 6 5 4 1 8 7 2

And are liked clean cloths by all neat

The information provided here is only for refrence. It may vary from the original. www.recruitment.guru

Passcode for Batch VI

2 3 6 7 8 5 4 1

Neat and are all by liked clean cloths

Thus, we can write any required step i.e., passcode for any batch from the given passcode directly.

20. Passcode for Batch III

5 8 1 4 3 2 7 6

Night succeed day and hard work to for

Passcode for Batch VI

2 3 6 7 8 5 4 1

Work hard for to succeed night and day

21. Passcode for Batch V

3 6 5 4 1 8 7 2

Visit in zoo should the we time day

Given Passcode

5 8 1 4 3 2 7 6

Zoo we the should and day time in clearly, this is the passcode for Batch III

22. Passcode for Batch IV

6 5 8 7 2 1 4 3

To fats rush avoid not do very run

Passcode for Batch II

8 1 2 7 6 3 4 5

Rush do not avoid to run very fast.

23. Passcode for Batch II

8 1 2 7 6 3 4 5

Length the day equal of and night are

Passcode for Batch V

3 6 5 4 1 8 7 2

The information provided here is only for refrence. It may vary from the original. www.recruitment.guru

And of are night the length equal day

24. Passcode for Batch II

8 1 2 7 6 3 4 5

To confidence hard you leads work and success

Passcode for Batch IV

6 5 8 7 2 1 4 3

Leads success to you hard confidence and work

25. Passcode for Batch VI

2 3 6 7 8 5 4 1

And pencil are used are pen

Passcode for Batch I

1 2 3 4 5 6 7 8

Pen and pencil are used by all boys

26. Passcode for Batch IV

6 5 8 7 2 1 4 3

Not go the way to of out do

Passcode for Batch III

5 8 1 4 3 2 7 6

Of to do out go not way the

27. First and third premises are particular Affirmative, i.e., I-type.

Second premise is Universal Affirmative (A-type).

1. Some books are pens

2. All pens are chairs.

We know that, I + A = I-type conclusion Therefore, our derived conclusion would be: “Some books are chairs” This is the conclusion I. Conclusion II is the conversion of our derived conclusion.

The information provided here is only for refrence. It may vary from the original. www.recruitment.guru

Two out of three premises are Particular and hence, Universal conclusion is invalid. That is, conclusion III does not follow. Conclusion IV is the conversion of the third premise. Therefore, only conclusion I, II and IV follow.

28. All the three premises are Universal affirmative (A-type).

1. All cars are jeeps.

2. All jeeps are buses.

We know that A + A = A-type conclusion Therefore, our derived conclusion would be: “All cars buses’. There is no such conclusion Now, All cars are buses.

All buses are trucks We know that, A + A = A-type conclusion Thus, our derived conclusion would be: “All cars are trucks”: This is the conclusion IV. Again, All jeeps are buses.

All buses are trucks. We know that, A + A = A-type conclusion Thus, our derived conclusion would be: “All jeeps are trucks” Therefore, only conclusion IV follows. Thus, our required answer is option (d) VENN -DLAGRAM

B

us Je C Tr e ar

u p ck

The information provided here is only for refrence. It may vary from the original. www.recruitment.guru

29. All the three premises are Particular Affirmative (I-type). Therefore, no conclusion can be derived from these premises.

Now look for any conversion and/or implication: There is no such conclusion.

30. First premises is Universal Affirmative (A-Q type).

Second premises is Particular Affirmative (I-type).

Third premises is Universal Affirmative (A-type).

Some bricks are ropes.

All ropes are doors.

We know that,

I +A = I-type conclusion.

Conclusion: Some bricks are doors. It is conversion of conclusion II.

Conclusion I and II from complementary pair. Therefore either conclusion I or II follows.

Therefore, our required answer is option(c).

31. First and second premises are Particular Affirmative i.e., I-type. Third premise is Universal Affirmative (A-type).

Conclusion I is the conversion of the third premise.

Second and third premises are relevant for the conclusion II. Thus,

Some pens are watches.

All watches are radios.

We know that,

I + A =I type conclusion.

Conclusion : Some pens are radios .

Conclusion II is the conversion of this conclusion.

Therefore, only conclusion I and II follow.

VEENN DLAGRAM W atc Ra Book Pen he dio

s s

The information provided here is only for refrence. It may vary from the original. www.recruitment.guru

32. First premise Universal Affirmative (A-type)

Second premise Universal Negative (E-type)

Third premise Particular Affirmative (I-type)

All towns are villages.

No village is forest.

We know that,

A + E = E type conclusion

Conclusion: No town is forest

There is no such conclusion

All towns are villages.

No village is forest.

We know that,

E + I= O I-type conclusion

Thus, our derived conclusion would be :

“Some rivers are not villages.”

This is the conclusion III.

Again.

No town is forest.

Some forests are rivers.

We know that,

E+I = O1- type conclusion

Conclusion: Some rivers are not towns.

There is no such conclusion.

Conclusion II is the conversion of the second premise.

Therefore Conclusion II and III follow.

33. 11 Boys 12th

After shifting his place

The information provided here is only for refrence. It may vary from the original. www.recruitment.guru

11 Boys 12 13 14 15 16

18th 17 Boys

Total Number of boys in the row

= (16 + 18) -1 = 33

34. 1 2 3 4 5 6 7

P R O B L E M

It has been coded as

7 1 6 2 5 3 4

M P E R L O B

Similarly

1 2 3 4 5 6 7

N U M B E R S

It code would be:

7 1 6 2 5 3 4

S N R U E M B

35. Both the assumptions are implicit in the statements. It is clearly mentioned in the statements that the Government has hiked the prices of diesel and petrol to reduce the oil pool deficit. Whenever the prices of commodities are hiked, generally people raised the voice against such measure.

36. The price any product is lowered assuming that its demand will increase. Therefore, assumption I is implicit in the statement.

37. From the content of the statement it is clear that both the assumptions are implicit in the statement.

38. Only assumption II is implicit in the statement. If Aswin’s mother asked his son to return home by train if it rains heavily, it implies that the trains would ply even if it rains heavily.

39. Both the assumptions are implicit in the statement.

40. Both the assumptions are implicit in the statements.

41. Only argument II is strong.

42. Neither of the argument is strong.

43. Only argument I is strong. Now a days television is an, essential means to provide useful academic information. Therefore, it is not desirable to prevent children from watching selected programmes on television. For the same reason, argument II is invalid.

44. Only argument I is strong. It is true that employees put their genuine demands before the management through the trade unions. Therefore, it is not judicious to ban the trade unions

The information provided here is only for refrence. It may vary from the original. www.recruitment.guru

completely. It is true that employees compel the management in some instances to concede their some illegal demands through the unions but the solution suggested is not appropriate.

45. Only argument I is strong. Women are equally capable and they should be given equal opportunity in matter of employment. Argument II makes no point.

46. Both the conclusions logically follows from the information given in the statement. The term milk glut clearly indicates that the milk production of State ‘X’ is more than its requirement. Again, it is clearly mentioned that the Government and co-operative dairies in State ‘X’ failed to use the available milk it implies that such dairies State ‘X’ are not equipped properly.

47. Neither conclusion I nor conclusion II follows. The subsidy on any item cannot be 100 per cent. It is mentioned in the statement the Government has decided to withdraw 3 per cent of “the subsidy” on cooking gas and not 33 per cent of the actual price of the cooking gas. Therefore, it is erroneous to assume that the price of the cooking gas will increase at least by 33 per cent. Any policy of the Government affects all sections of the society and it cannot be assumed that poor people do not need subsidy. Thus, neither of the conclusion follows.

48. From the statement it is clear that the Government will either increase the price of diesel or will not increase the price of diesel. The deficit on this count can be adjusted by some other means. Therefore, after the review of the present policy of the diesel price in view of further spurt in the international oil prices, the Government is left with only two options to increase or not to increase the price of diesel. Therefore, either conclusion I or conclusion II follows.

49. Neither of the conclusions follows. It is clear from the statement that the City ‘Z’ requires beautification but this does not imply that the people of that city are unaware about the state of ugliness of their city. Therefore, conclusion I does not follow. The new Municipal Commissioner asserted that the task of beautification of City ‘Z’ could also be accomplished if such has happened in City ‘X’ and City ‘Y’ . From this it cannot be derived that the Municipal Commissioner has served in City ‘X’ and City ‘Y’ while asserting something one may quote the works of another person. Hence conclusion II is also not valid.

50. Neither conclusion I nor conclusion follows. In the absence of harsh punitive measures the cases of sexual harassment may take place anywhere. Therefore, it is erroneous to assume that sexual harassment of women at work place is more prevalent in India as compared to other developed countries. The statement exhorts that amendment in the Act is meant to curb sexual harassment at the work place and it cannot be taken as the imposition of restriction on recruitment of women, In order to avoid pain in arms we cannot chop our arms.

51. It is not given in the statement I That Ashok or Ravi is facing towards which direction. Therefore, it is not possible to determine the direction of right side of Ravi. From statement II Samir is sitting opposite of Ashok facing north. It implies that Ashok is facing toward south. From both the statements Ashok is facing toward south and is sitting to the right of Ravi. It implies that Ravi is facing toward south or north. Therefore, required answer is (d).

52. From statement I The mother of N is the daughter of M. The mother of N is the sister of K. Therefore, the mother of N and K are the children or M. But it is not clear whether M is the father or mother of K. From statement II P is the wife of M. Therefore, M is the husband of P. There is no information about K in statement II. From both the statements it is clear that M is the father of K.

The information provided here is only for refrence. It may vary from the original. www.recruitment.guru

53. From statement I T is the sister of M and K. There is no information about D in the statement I. It is not clear whether M and K are males or females. Thus, the data in statement I alone are not sufficient to answer the question. From statement II K is the brother of D. There is no information about T in the statement U. We can’t determine the sex of D even with the help of data given in the statement I and II. Therefore, our required answer is option(d).

54. There is no information about P in the statement. Therefore, the data in statement I alone are not sufficient to answer the question. But it is clear that M,S and T are brothers and or sister to one another. From statement II P is the mother of T and she has only one daughter. It implies that T is the daughter of P. From statement I and II P has two sons S and T,M is the daughter of P. Therefore the data given in both the statement I and II are necessary to answer the question

55. From statement I R > Q, T There is no information P and S in the statement I. Thus, the data given in the statements I alone are not sufficient to answer the question. From statement II T > S, P S > Q, R T > S > Q, R

P It is clear that T is the tallest among them. Thus, the data in the statement II along are sufficient to answer the question, while the data in the statement I alone are not sufficient to answer the question.

61. The movement and changes of designs from Problem Figure (a) to (b) can be shown as:

• • •

• • • New design From Problem Figure (d) to (e)

• • • New design

• • •

From Problem Figure (b) to (c) similar changes would occur as that have been occurred from Problem Figure (d) to (e) and new design (p) would appear at the lower middle position because starting from Problem Figure (b) a new design appears in the clockwise direction in the subsequent figures.

62. From Problem Figure (d) to (e) the lower design is reversed laterally while the other design moves to the opposite side. Similar changes would occur from Problem Figure(b) to (c) .

The information provided here is only for refrence. It may vary from the original. www.recruitment.guru

63. In each subsequent figure all the designs move one step in clockwise direction and one of the designs gets paired.

64. In each, subsequent figure the plane of designs rotates through 45° anticlockwise. From Problem Figure (d) to (e) the fourth design from right is shifted to the first position while all other designs move one step and the last design is replaced by a new design. Similar changes would occur from Problem Figure (b) to (c) .

65. Each of the arcs rotates 90° clockwise or anticlockwise and 180° alternately.

66. The movement of designs from unnumbered figure to figure (a) can be shown as:

• • •

• • • New design

The movement of design from figure (a) to figure (b) can be shown as:

New design • • •

• • •

These two steps are repeated alternately . In figure. There should be (P) instead of (S).

67. The last figure is inverted form of the first figure. Figure (d) is the inverted form of figure (b). Therefore, figure (e) should be inverted form of figure (a).

68. In the first step one line segment is deleted from the upper figure while the lower figure is inverted. In the next step one line segment is deleted from the lower figure while the upper figure is inverted. These two steps are repeated alternately. In the figure (c) both the figures are inverted.

69. The triangle is rotating respectively 45°, 90°, 135°, 180°, 225°,.. in anticlockwise direction while the arc is inverted in the next figure. In figure (a) triangle has been rotated through 90° anticlockwise instead of 90° clockwise. 70. The line segments are added in the following manner: 0 1 1 2 2 3 3 4 4 5 5 6 Upper 1 1 0 1 1 1 Side Lower 1 2 1 1 2 0 Side Therefore, figure (d) is the wrong in the series.

71. In the subsequent figure respectively one, two, three… design (s) is/are inverted.

72. In the subsequent figure respectively two, three, two, four… line segments are deleted.

73. In each subsequent figure all the designs move half step in anticlockwise direction, two designs interchange positions and one of these two, designs is replaced with a new design.

74. The complete design is inverted in the next figure and one design is added in every figure.

75. From Problem Figure (a) to (b) the following changes occur:

• • • • •

The information provided here is only for refrence. It may vary from the original. www.recruitment.guru

Similar changes would occur from Problem Figure (5) to Answer Figure.

76. Let the unit’s digit be x and ten’s digit be y.

Also let y > x Number = 10y + x And number obtained by interchanging the digits. = 10x + y :. 10y + x - 10x – y = 36 = 9y -9x =36 = 9(y –x) =36 = y-x =4

77. Required difference = 2 or 200 – 3 of 125 5 5 = 80- 75 = 5

78. Let the breadth of rectangular field be x metres. :. Length = x x 115 100 = 23x metres 20 Now, Length x Breadth = Area = 23x x x = 460 20 = x2 = 640 x 20 23 2 = x =20x 20

x √ 20 x 20 = 20metres

79. Let the listed price be Rs. x :. Discount = 30% of x = 30 x = Rs. 3x 100 10 According to the question, 3x = 82.5 10 = x = 82.5 x 10 = Rs. 275 3 :. Required cost price of calculator = 70% of 275 = Rs 70 x 275 = Rs .192.50 100

80. There are 8 letters in the word “SOFTWARE”, including 3 vowels (O, A, E) and 5 consonants ( S, F,T,W,R). Considering three vowels as one letter, we have six letters which can be arranged in 6P6 = 6! Ways . But corresponding, the vowels can be put together in 3! Ways. :. Required number of words = 6! x 3! = 4320

81. Initial speed of motor =71 kmph.

The information provided here is only for refrence. It may vary from the original. www.recruitment.guru

Distance covered in first 2 hours = 2 x 70 = 140kms. For next two hours speed of motor = 80kmph :. Distance covered in first 4 hours = 140 + 160 =300 Remaining distance =345 – 300 = 45km This distance will be covered will be covered at the speed of 90 kmph. :. Time taken = 45 = 1 hour 90 2

:. Total time = 4 + ½ = 4 ½ hours

82. Total area of rectangular plot to be gardened = 340 x 1 + 4 x 1 = 344 sq. m :. Total cost = 344 x 10 = Rs. 3440

83. 4 men out of 7 men and 4 women out of 8 women can be chosen in 7C4 x 8C4 ways =7 x 6 x 5 x 4 x 8 x 7x 6 x 5 1 x 2 x 3 x4 1 x 2 x 3 x4 = 35 x 70= 2450

84. Let the principal be Rs x and rate of interest be r% Case I: x x r x 7 =1750 100 = xr = 1750 x 100 7 = Rs. 25000 Case II: S.I. = x x (r+2)x 7 100 Which cannot be determined with the help of given information.

85. Let the initial seats for Maths, Physics and Biology be 5x, 7 x and 8x respectively. Now, new seats for Maths, = 5x x 140 100 For Physics .= 7x x 150 100 And for Biology = 8x x 175 100 :. Required ratio = 5x x 140 : 7x x 150 : 8x x 175 100 100 100 = 5 x 140 : 7 x 150: 8 x 175

The information provided here is only for refrence. It may vary from the original. www.recruitment.guru

= 2 : 3 : 4

87. Amount of fat in : 200 gms of A and 300 gms of C = 200 x 28 + 300 x 13 100 100 = 56 + 39 = 95 gms 200 gms of B and 200 gms of A = 40 + 56 =96 gms 100 gms of C and 300 gms of A = 13 + 84 = 97 gms 300gms of C and 100gms each of A and B =39 + 28 +20 =87 gms 150gms each of A, B and C = 28 x 3 + 20 x 3 + 13 x 3 2 2 2 = 42 +30 + 19.5 =91.5 gms Hence option (c) is our answer.

88. A =y% of x = xy 100 B =y% of x = xy 100 :. A = B

89. Number of literate males =35% of 2,50,000 Number of literal females =33% of 2,50,000 :. Required difference = (35 -33)% of 2,50,000 = 2% of 2,50,000 = 5,000

90. We have, x – y = a - 1 a- 1 a – 1 = a – 1 =1 a - 1 Clearly x > y

91. (I) p2 - 7p = -12 = p2 – 4p +12 = 0 = p2 -4p 3p + 12=0 = p(p-4) -3(p-4)=0 = (p -4) (p -3) =0 = p=3 or 4 (II) q2 -3q + 2=0 = q2 -2q –q +2=0 = q(q-2) -1(q -2) =0 = (q-2) (q-1)=0 = q=1 or 2

The information provided here is only for refrence. It may vary from the original. www.recruitment.guru

Obviously p >q

92. (I) 12p2 - 7p = -1 = 12p2 - 7p + 1 = 0 = 12p2 – 4p - 3p + 1 = 0 = 4p(3p – 1) – 1 (3p -1) =0 = (3p -1) (4p -1) =0 = p = 1 or 1 4 3 (II) 6q2 -7q + 2 = 0 = 6q2 - 4q – 3q + 2 =0 = 2q(3q -2) – 1 (3q -2)=0 = (3q -2) (2q -1) =0 = q = 2 or 1 3 2 Obviously p < q

93. (I) p2 + 12p + 35 = 0 = p2 + 7p + 5p + 35 =0 = p (p +7) + 5(p + 7) =0 = p(p+7) +5 (p + 7)=0 = p(p +7) (p + 5) =0 = p = -5 or -7 (II) 2q2 + 22q + 56 = 0 = 2q2 + 14q + 8q + 56 = 0 = 2q(q + 7) +8(q +7) =0 = (q + 7) (2q +8) =0 = q = - 7 or - 4 Obviously p < q

94. (I) p2 + 8p + 15 = 0 = p2 – 3p - 5p + 15 =0 = p(p -3) -5 (p-3) =0 = (p -3) (p -5) =0 = p = 3 or 5 (II) q2 – 5q + 6q +6=0 = q2 - 3q – 5q +6 =0 = q(q – 3) -2(q -3) =0 = q(q-3) (q – 2) =0 = q = 3 or 2 Obviously p < q

95. (I) 2p2 + 20p + 50 = 0 = p2 + 10p + 25 =0 = (p +5)2 =0 = p +5 =0 = p = -5 (II) q2 = 25 = q = + 5 Obviously p < q

96. Production of Company A In 2000 =4 lakh units In 2001 = 3 lakh units Decrease = 4 -3 = 1 lakh units :. % decrease = 1 x 100 = 25%

The information provided here is only for refrence. It may vary from the original. www.recruitment.guru

4

97. Production of Company B In 2006 = 7 lakh units In 2007 = 11 lakh units :. Required percentage = 11 x 100 7 =157.14 =157%

98. It is obvious from the graph.

99. Total production of Company A =(4 + 3 + 8 + 8 + 8 + 7 + 8 + 12) Lakh units =5800000 Total production of Company B = (1 + 1 + 1 + 2 + 3 + 5 + 7 +11) =31 lakh units = 3100000 :. Required difference = (58-31) x100000 =2700000

100. Required average production of Company B = 31 lakh units 8 =3.875= 3.9llakh units

101. For Course D Number of girls = 30% of 800 = 30 x 800 = 240 100 Number of students = 35 x 1 x 1200 100 = 420 :. Number of boy = 420 – 240 = 180 :. Required ratio = 180 : 240 = 3 : 4

102. Number of boys In Couse E = 12% of 1200 – 14% of 800 = 144 – 112 = 32 In course F = 13% of 1200 – 14% of 800 156 – 112 = 44 In course A = 20% of 1200 – 30% of 800 = 240 – 240 =0 In course D = 35% of 1200 – 30% of 800 = 420 – 240 =180 In course C = 5% of 1200 -2% of 800 = 60 – 16 = 44 Obviously pair C and F is our answer. 103. For Course E, Number of girls = 14% of 800 = 112 Number of boys = 32 :. Required percentage

The information provided here is only for refrence. It may vary from the original. www.recruitment.guru

= 80 x 100 = 250 32

105. Number of girls in course C = 2% of 800 = 16

106. The given series is based on following pattern 15 – 10 = 5 24 – 15 = 9 37 – 24 = 13 54 – 37 = 17 75 – 57 = 21 100- 75 = 25 Obviously 35 is wrong number

107. Here the middle number = difference of succeeding number and preceding number i.e. 4 – 1 = 3 7 – 3 = 4 11- 4 = 7 18 – 7 =11 27– 11 = 16 Here the sequence gets disturbed 29 - 11 = 18 47- 18 =29 Hence 27 is the wrong number

108. The sequence is based on following pattern 3 x 0.5 + 0.5 =2 2 x 1 +1 = 3 3 x 1.5 + 1.5 = 6 6 x 2 +2 =14 14 x 2.5 + 2.5= 37.5 37.5 x 3 + 3 =37.5

109. 32431 = 7 x 4626 + 72 4626 = 6 x 765 + 62

765 = 5 x 148 + 52 = 148141 x 32 + 42 But 148 = 4 x 33 + 42 33 = 3 x + 32 8 = 2 x 2 +22 Obviously 32 is the wrong number. 110. The sequence is based on following pattern: 3 – 2 = 13 11 – 3 = 8 = 23 38 – 11 = 27 = 33 102 – 38 = 64 = 43 But 229 – 102 = 127 = 53 227 – 102 = 125 = 53 442 – 227 = 125 = 63 obviously 229 is the wrong number.

111. S.P. of the product = Rs. 100 From statement I C. P. = 100 x 90= Rs. 75 120 :. Actual profit = 100 -75 = Rs. 25

The information provided here is only for refrence. It may vary from the original. www.recruitment.guru

From statement II Let the C.P. be Rs.x Then profit = x 3 :. x + x = 100 3 = 4x = 300 = x =Rs. 75 :. Profit = Rs. 25

112. Time taken in seconds to cross = Sum of length of both trains in metres Relative speed in m/sec. x + 0. 0y + z 24 Where z m is the length of other train To know u,z is required.

113. Let the digits at unit’s and ten’s places be x and y respectively. When x > y then x – y = 6 Then x –y = 6 And x + y = 12 Gives x = 9 And y = 3 Again, where y > x y- x = 6 Hence y – x = 6 And x + y = 12 Gives y = 9 And x = 3 Hence, the numbers are 93 or 39 Obviously when statement I is not taken into consideration, two numbers are possible.

114. X = odd number. It is possible 5 5 Only when X is odd From statement II X + Y = an odd number But, X = 2 and Y =5 gives X + Y =7, an odd number Hence only statement I is sufficient to answer the question. 115. From statement I Capacity of tank = p x 14 x14 x 28 = 17248 m3 From Statement II. Capacity of tank = 616 x 28 = 17248 m3 Obviously, either statement I or statement II is required. 116. Marks obtained by Meera in: English = 60 History = 40 x 80 = 32 100 Computers = 50 x 130 = 65 100 Maths = 90 x 150 = 135 100 Science = 90 x 120 = 108 100 Economics = 60 x 80 = 48

The information provided here is only for refrence. It may vary from the original. www.recruitment.guru

100 :. Total marks = 60 +32 +65 + 135 +108 + 48 = 448 117. Marks obtained in History by: Meera = 40 x 80 =32 100 Subodh = 70 x40 =28 100 Kunal = 70 x40 =28 100 Soni = 60 x 40 100 Richu = 90 x 40 = 36 100 Irene = 60 x 40 = 24 100 Vijay = 80 x40 = 32 100 Required average = 32 + 28 + 28 + 24 + 36 + 24 +32 7 = 204 = 29.14 7 118. Total marks obtained by Kunal = 54 + 28 + 78+ 135+ 84 + 56 = 435 Total marks = 60 + 40 +130 +150 +120 +80 = 580 :. Required percentage = 435 x 100 = 75 580

121. 95? = 953.7 ÷ 951.0 = 95? = 953.7 – 1 =952.7 = ? = 2.7

122. ? = √1000 + 3 x 1892 5 = 100 + 1135.2 = 1235.2 = 1230

123. = 0.0004 x 36 0.0001 = 4 x 36 = 144 = 145

124. ? = 140% of 12300 = 140 x 12300 100 = 17220 = 17000

125. ? = 3739 + 160 x 30 = 3739 + 4800 = 8539 = 8200

136. (A) Proximate – nearest or next, without anything between a cause and it effect.

The information provided here is only for refrence. It may vary from the original. www.recruitment.guru

(B) Elevated –dignified, elated, grand, high lofty, noble, raised, sublime (C) Nimble – active, agile, alert, brisk, deft, light-footed, lively, ready. (D) Agile-active, adroit, brisk, clever, quick, sharp, swift, spry Hence C-D bear same meaning. 137. (A) Notion-apprehension, belief, concept, fancy, image, judgement, knowledge (B) Symbol-badge, emblem, image, logo, representation sign, token, type (C) Concept-abstraction conception, construct, idea, image, type, view (D) Massage-rubbing, kneading, manipulation, rub-down Hence, B_D bear same meaning

138. (A) Limpid-bright, clear, comprehensible, crystal-clear, glassy, intelligible, pure (B) Luscious-appetising, delicious, desirable, juicy, savoury, sweet, tasty (C) Acrid –acid, acrimonious, bitter, sharp, burning, pungent, sarcastic (D) Benign-amiable, benevolent, friendly, genial, gentle, good, gracious Hence, B-C bear opposite meanings

139(A) Asinine-of or like an ass, idiotic (B) Furious + angry, boiling, enraged, fierce, fuming, givid, mad, raling, violent, stormy (C) Fortunate Advantageous, blessed, bright, luck, happy, successful, felicitous, opportunate (D) ridiculous absurd, comical, derisory, farcical, foolish, funny, stupid, ludicrous Hence , A-D bear same meaning 140. (A) Companion – accomplice, aide, ally assistant, associate fellow, mate, partner (B) Amateur-buff, dabbler, dilettante, fanier, ham, layman, non-professional (C) Adept- able, accomplished, adroit versed, skilled, proficient, practised nimble. (D) Adgerent-admirer, advocate, devotee, disciple, fan, follower, supporter Hence, A-D bear same meanings.

141. (A) Squander- blow, consume, dissipate, expend, fritter, waste, scatter, misuse (B) Disunite-to separate from union, detach, divide, to severe or surrender (C) Flicker-flare, flash, flutter, waver, vibrate, twinkle, glimmer, gutter (D) Preserve care for, conserve, continue, defend, guard, keep, uphold, hitter Hence, A-D bear opposite meanings

142. (A) Mitigate-to mollify, oppease, to make more easily born, to temper (B) Acquiesce-accede, accept, agree, allow, approve, assent, comply, concur (C) Relinquish-abandon, cede desert, discard, forgo, resign, vacate (D) Duplicate-corresponding, identical, method, twofold, twin Hence, B-C bear opposite meanings

144. (A) Occurrence-aution, adventure, affair, instance, incident, episode (B) Pretence-acting, affection, aim, allegation, appearance, charade, grab (C) Profusions-abundance, copiousness, glut, multitude, wealth, extravagance (D) Extravagance-abundance, excess, folly, squandering, waste, profusion Hence, C-D bear same meaning

145. (A) Excellent-admirable, commendable, unequalled, superb, wonderful, worthy (B) Passionate-ardent, erotic, stormy, violent, wild, zealous, fiery, inflamed, fervent (C) Apathetic-Cold, Cool, impassive, passive, unemotional, indifferent (D) Discrepant-contrary, disagreeing Hence, B-C bear opposite meanings.

(146-150) First Sentence –(F) I will answer all your queries a little elaborately. Second Sentence-(C) Firstly, there was no tradition of theatre in my family Third Sentence-(G) Let alone theatre, arts in general had none place of respect in my family. Fourth Sentence-(E) My parents were old fashioned. Fifth Sentence-(B) My father was a government servant.

The information provided here is only for refrence. It may vary from the original. www.recruitment.guru

Sixth Sentence-(B) He never wanted me to be an actor, as he didn’t took upon theatre or acting as respectable vocation. Seventh Sentence-(A) But I always felt somewhere in my mind that I loved acting. Eighth Sentence-(D) I am talking specifically of acting not theatre in general.

151. The group of words, ”sea water for instance, contains a lot of salt, fresh water contains very little” should be replaced by ‘sea water for instance, contains a lot of salt fresh water contains a very little’ Look at the sentences: He has little milk to give you (x) He has a little milk to give you (√)

152. The group of words, a sparrow has made a nest in Kesho’s house’ should be replaced by ‘a sparrow had made nest in Kesho’s house’. Look at the sentences : Heena said that he is wrong (x) Heena said that he was wrong (√)

153. The group of words, ‘monopoly for himself should be replaced by ‘monopoly of himself. 154.The group of words, ‘we can be certain that man did not began’ should be replaced by ‘man did not begin’ Because Do/Does/Did takes First Form (V1). Look at the sentences: He do goes to market. (x) He do go to market. (√) Or, he goes to market.( √) He did not went to market. (x)

V2 V2

He did not go to market. (√)

161. The group of words ‘was Stranding’ should be replaced by ‘had been stranded’. The word strand means- to cause something to be along, to make a thing or person to be deserted. Look at the sentence: The dead body of the man who hanged

active was found. (x) The dead body of the man who was hanged

Passive Was found. (√)

162. The group of words, ‘I receive the letter 1 should be replaced by ‘I received the letter’. The principal clause of the sentence is in Past Tense, hence the sub ordinate clause should also be used in Past Tense. Look at the sentence: Yesterday when I go there, he had gone

Present tense Past tense Out. (x) Yesterday when I went there, he had gone

Past tense Past tense Out.(√)

164. The group of words, ‘to the committee’ should be replaced by of the committee. To express possession between two Nouns we use-Noun + of + Noun structure.

The information provided here is only for refrence. It may vary from the original. www.recruitment.guru

Look at the sentence: The colour to this pen is green(x)

Noun possessive noun The colour this pen is green(x)

Noun possessive noun

The information provided here is only for refrence. It may vary from the original. www.gradeup.co

1 www.gradeup.co

1. Recently, RBI revised PCA framework for 9. Which of the following country has topped in banks. Here PCA stands for- the World Economic Freedom (WEF) Index A. Prompt Clear Action 2016? B. Primary Corrective Action A. Belgium B. Algeria C. Primary Corporate action C. Hong Kong D. Switzerland D. Prompt Corrective Action E. None of these E. None of the above 10. Reserve Bank of India (RBI) tightened the 2. Which bank has launched ‘Mingle’? rules around making the JLF more effective, A. SBI B. ICICI directing banks not to break any rules and to C. PNB D. Axis meet all deadlines. Here JLF stands for- E. BOB A. Joint Lending Firms 3. RBI has said in a notification that all existing B. Joint Lending Forum asset reconstructions companies (ARC) must C. Joint Lender’s Forum have minimum net owned corpus of how D. Joint liability Forum much amount by March 2019? E. Joint litigation Forum A. Rs. 500 crores B. Rs. 100 crores 11. US-based think tank GFI in its report titled C. Rs. 200 crores D. Rs. 400 crores ‘Illicit Financial Flows to and from Developing E. None of these Countries: 2005-2014’, has estimated that 4. In the year 2017, the Indian government has $770 billion worth of black money entered revised base year of which of the following India during 2005-2014. Here GFI stands for- indices? A. Global Financial Institution A. CPI B. WPI B. Global Financial Investigator C. IIP D. b and c only C. General Financial Institution E. All the above D. Global Financial Integrity 5. How much fiscal deficit target in terms of GDP E. Global Fair Institution percent, fixed by government for financial 12. Working capital means year 2018? A. Current assets – current liabilities A. 4.2 Percent B. 3.7 percent B. Fixed assets – current assets C. 3.2 Percent D. 2.9 Percent C. Fixed assets – fixed liabilities E. 2.7 percent D. Fixed liabilities – current liabilities 6. Insolvency and Bankruptcy Board of India 13. What do we call the risk of collapse of an (IBBI) constituted Technical Committee Under entire financial system or entire market? ______to give recommendations for laying down Technical Standards for performance of A. Seismic Risk core services and other services under IBBI B. Systemic Risk (Information Utilities) Regulations 2017. C. Systematic Risk A. Dinesh Sharma B. Asim Dasgupta D. Market Risk C. R B Barman D. Sushil Mitra E. Economic Risk E. None of the above 14. The Union Budget 2017-18 has allocated Rs. 7. Recently IMF revised India’s GDP growth rate ______crore to the flagship MGNREGA forecast from earlier estimated 7.2 Percent to program? ______. A. 48,000 B. 44,000 A. 7 Percent B. 6.9 Percent C. 55,000 D. 42,000 C. 6.8 Percent D. 6.7 Percent E. 58,000 E. 6.5 Percent 15. Reverse Repo Rate is a tool used by RBI to 8. Which of the following country hosted the 14th Edition of United Nations International A. Inject Liquidity Day of Vesak 2017? B. Absorb Liquidity C. Increase the liquidity with banking system A. Greece B. France D. To keep the liquidity at one level C. Belgium D. Sri Lanka E. None of the above E. None of these

2 www.gradeup.co

16. What is a ante-dated cheque? A. 12 months B. 24 months A. A cheque issued without drawer's signature C. 48 months D. 60 months B. A cheque with only signature of the drawer E. None of these C. A cheque which has completed three 23. Consider the following statements regarding months from its date of issue Payment Banks: D. A cheque which has completed six months A) Payment Banks can't advance loans or from its date of issue issue credit cards. E. Cheque is written by drawer and dated at B) Payment Banks can accept demand some date in past but not expired deposits up to Rs 5 lakh. 17. Which bank has launched 'Aadhaar Payment C) Payment Banks offer remittance services, Bridge System' (APBS) for small ticket micro- mobile payments/transfers/purchases. finance loan disbursements? D) Payment Banks offer banking services like A. SBI B. ICICI Bank ATM/debit cards, net banking and third party C. HDFC D. Federal Bank fund transfers. E. RBL Bank Which of the above options are correct: 18. ______is the rate at which banks borrow A. A, C and D only B. B, C and D only funds overnight from the Reserve Bank of C. A and D only D. All of the above India (RBI) against approved government E. Only A & B securities. 24. Recently NITI Aayog has constituted an A. CRR B. SLR ‘Expert Task Force’ to provide a major thrust C. Call Money D. MSF to job creation by enhancing India’s exports. E. Repo Rate Who will head the task force? 19. The National Financial Switch (NFS) facilitates A. Dr. Rajesh Kumar routing of ATM transactions through inter- B. Rahul Ranjan Sinha connectivity between the Bank’s Switches, C. Dr. Rajiv Kumar thereby enabling the citizens of the country to D. R. B. Barman utilize any ATM of a connected bank. The E. None of these National Financial Switch (NFS) is run by ___. 25. Unified Payments Interface (UPI) launched by A. National Payments Corporation of India the National Payments Corporation of India (NPCI) (NPCI) would do the following function- B. Industrial Finance Corporation of India 1) It would eliminate the need to exchange (IFCI) sensitive information about bank account C. Institute for Development and Research in numbers during a financial transaction. Banking Technology (IDRBT) 2) It allows sharing of a bill among peers. D. Reserve Bank of India (RBI) 3) It allows individual to even request the E. None of these money from its debtors. 20. Name the world’s highest wicket-taker in Select the correct answer- women ODI cricket? A. 1 and 2 only B. 2 and 3 only A. Jhualan Goswami B. Mithila Raj C. 1 only D. 1, 2 and 3 C. Poonam Raut D. Anjum Chopra E. 3 only E. Veda Krishnamoorthy 26. Kavinder Singh is related to which game? 21. Who among the following has been elected as A. Boxing B. Archer the chairman of ‘FIFA governance committee’ C. Hockey D. Shooting at the world football governing body's? E. Wrestling A. Altamas Kabir B. R C Lahoti 27. The base year for calculating key economic C. Mukul Mudgal D. P Sathasivam growth from 2004-05 changed to? E. None of these A. 2010-11 B. 2011-12 22. Non-Banking Financial Company (NBFC) are C. 2008-09 D. 2007-08 allowed to accept/renew public deposits for a E. 2005-06 maximum period of ___.

3 www.gradeup.co

28. After the merger of five associates Banks in 36. A class of derivative whose value is at least State Bank of India (SBI), now the total partly derived from one or more underlying number of ATMs reached across the country is stock exchange securities? (Trade, _____. Commodity, Credit, Interest Rate, Equity) A. 60,000 ATMs B. 69,000 ATMs A. Derivative C. 59,000 ATMs D. 72,000 ATMs B. Stock Derivative E. 45,000 ATMs C. Future Derivative 29. The target of digital transactions through UPI, D. Equity Derivative USSD, Aadhar, IMPS for 2017-18 is ____. E. Other than the given options 37. The government has decided to make tax A. 22,00 crore B. 1600 crore evasion of over ______under the C. 2000 crore D. 2,500 crore proposed Goods and Services Tax (GST) the E. 19,00 crore regime as a non-bailable offence. 30. Finland has become the first country in A. Rs 6 crore B. Rs 3 crore Europe to pay its unemployed citizens a basic C. Rs 5 crore D. Rs 8 crore monthly income, amounting to___. E. Rs 10 crore A. 226 euros B. 324 euros 38. As per report of Stockholm International C. 560 euros D. 421 euros Peace Research Institute (SIPRI) India’s E. 611 euros military expenditure in 2016 grew by- 31. Government has approved Rs 681 crore as A. 7 per cent B. 8.5 per cent seed capital for building a total corpus of ____ C. 8 per cent D. 7.5 per cent under the electronic development fund meant E. 6.5 per cent to support entrepreneurship and innovation in 39. India Licensing Expo (ILE), India's first and electronics and IT. most influential brand licensing show will take A. Rs 5,125 crores B. Rs 6,355 crores place on 20 August 2017 in ___. C. Rs 5,732 crores D. Rs 6,831 crores A. Hyderabad B. Bengaluru E. Rs 7,233 crores C. Chennai D. Mumbai 32. Government of India has changed Cost E. New Delhi Inflation Index (CII) base year from 1981 to 40. Foreign direct investment (FDI) in India grew ___. 18 per cent during 2016 to touch ______data released by the Department of Industrial A. 2000 B. 2002 Policy and Promotion (DIPP). C. 2001 D. 2005 A. $42 billion B. $46 billion E. 2004 C. $50 billion D. $39 billion 33. Institute for Development & Research in E. $49 billion Banking Technology (IDRBT) is in? Direction (41-45): Read the following A. Hyderabad B. Bengaluru passage carefully and answer the questions C. Chennai D. Mumbai that follow. E. New Delhi At first sight, it looks as though Panchayati 34. Amendment to Section ______of Negotiable raj, the lower layer of federalism in our polity, Instruments Acts to include the electronic is as firmly entrenched in our system as is the image of a truncated cheque and a cheque in older and higher layer comprising the Union electronic form. Government and the States. Like the A. 7 B. 8 democratic institutions at the higher level, C. 2 D. 5 those at the panchayat level, the Panchayati E. 6 raj institutions (PRIs), are written into and 35. The target for agricultural credit in 2017-18 protected by the Constitution. All the essential has been fixed at a record level of _____. features, which distinguish a unitary system A. Rs. 5 lakh crores B. Rs. 10 lakh crores from a federal one, are as much enshrined at C. Rs. 20 lakh crores D. Rs. 25 lakh crores the lower as at the upper level of our federal E. Rs. 15 lakh crores system. But look closely and you will discover a fatal flaw. The letter of the Constitution as

4 www.gradeup.co

well as the spirit of the present polity have had been occupied by the Centre in the first exposed the intra-State level of our federal 30 years of Independence, and most of what system to a dilemma of which the inter-State was still left after that was occupied by the and Union-State layers are free. The flaw has States in the next 20. PRIs might have hoped many causes. But all of them are rooted in an to wrest some space from their immediate historical anomaly, that while the dynamics of neighbour, the States, just as the States had federalism and democracy have given added wrested some from the Centre. But having at strength to the rights given to the States in last managed to checkmate the Centre's the Constitution, they have worked against encroachments on their rights, the States the rights of panchayats. were not about to allow the PRIs to do some At both levels of our federal system there is encroaching of their own. the same tussle between those who have By the 1980s and early 1990s, the only certain rights and those who try to encroach national party left, the Congress, had gone upon them if they believe they can. Thus, the deeper into a siege mentality. Finding itself Union Government was able to encroach upon surrounded by State-level parties, it had built certain rights given to the States by the walls against them instead of winning them Constitution. It got away with that because over. Next, the States retaliated by blocking the single dominant party system, which Congress proposals for Panchayati raj in characterised Centre-State relations for close Parliament, suspecting that the Centre would upon two decades, gave the party in power at try to use panchayats to bypass State the Union level many extra-constitutional Governments. The suspicion fed on the fact political levers. Second, the Supreme Court that the powers proposed by the Congress for had not yet begun to extend the limits of its panchayats were very similar to many of the power. But all that has changed in recent more lucrative powers of State Governments. times. The spurt given to a multi-party State-level leaders also feared, perhaps, that democracy by the overthrow of the if panchayat-level leaders captured some of Emergency in 1977 became a long-term trend the larger PRIs, such as district-level later on because of the ways in which a panchayats, they would exert pressure on vigorously democratic multi-party system State-level leaders through intra-State multi- works in a political society which is as party federalism. assertively pluralistic as Indian society is. It It soon became obvious to Congress leaders gives political clout to all the various that there was no way the Panchayati raj segments which constitute that society. amendments they wanted to write into the Secondly, because of the linguistic Constitution would pass muster unless State- reorganisation of States in the 1950s, many level parties were given their pound of flesh. of the most assertive segments have found The amendments were allowed only after it their most assertive expression as States. was agreed that the powers of panchayats Thirdly, with single-party dominance could be listed in the Constitution. becoming a thing of the past at the Union Illustratively, they would be defined and level, governments can be formed at that endowed on PRIs by the State Legislature level only by multi-party coalitions in which acting at its discretion. State-level parties are major players. This has made it impossible for the Union Government This left the door wide open for the States to to do much about anything unless it also exert the power of the new political fact that carries a sufficient number of State-level while the Union and State Governments could parties with it. Indian federalism is now more afford to ignore panchayats as long as the real than it used to be, but an unfortunate MLAs were happy, the Union Government had side-effect is that India's Panchayati raj to be sensitive to the demands of State-level system, inaugurated with such fanfare in the parties. This has given State-level actors early 1980s, has become less real. strong beachheads on the shores of both By the time the PRIs came on the scene, most inter-State and intra-State federalism. By of the political space in our federal system using various administrative devices and non-

5 www.gradeup.co

elected parallel structures, State A. Should the state governments wrest more Governments have subordinated their PRIs to space from the Union, before considering the the State administration and given the upper Panchayati system? hand to State Government officials against B. Should rights similar to those that the the elected heads of PRIs. Panchayats have States managed to get be extended to become local agencies for implementing panchayats as well? schemes drawn up in distant State capitals. C. Should the single party system which has And their own volition has been further withered away be brought back at the level of circumscribed by a plethora of “Centrally- the States? sponsored schemes”. These are drawn up by D. Should the States get "their pound of even more distant Central authorities but at flesh" before allowing the Union government the same time tie up local staff and resources to pass any more laws? on pain of the schemes being switched off in E. None of these the absence of matching local contribution. 44. Which of the following most closely describes The "foreign aid" syndrome can be clearly the 'fatal flaw' that the passage refers to? seen at work behind this kind of "grass roots A. The ways in which the democratic multi- development". party system works in an assertively 41. The central theme of the passage can be best pluralistic society like India's are flawed. summarized as B. The mechanisms that our federal system A. Our grassroots development at the uses at the Union government level to deal panchayat level is now driven by the "foreign with States are imperfect. aid" syndrome. C. The instruments that have ensured B. Panchayati raj is firmly entrenched at the federalism at one level, have been used to lower level of our federal system of achieve the opposite at another. governance. D. The Indian Constitution and the spirit of C. A truly federal polity has not developed the Indian polity are fatally flawed. since PRIs have not been allowed the E. None of these necessary political space. 45. Which of the following best captures the D. The Union government and State-level current state of Indian federalism as parties are engaged in a struggle for the described in the passage? protection of their respective rights. A. The Supreme Court has not begun to E. None of these extend the limits of its power. 42. The sentence in the last paragraph, “And their B. The multi-party system has replaced the own volition has been further single party system. circumscribed...”, refers to: C. The Union, state and Panchayati raj levels A. The weakening of the local institutions' have become real. ability to plan according to their needs. D. There is real distribution of power between the Union and State level parties. B. The increasing demands made on elected local leaders to match central grants with E. None of these local contributions. C. The empowering of the panchayat system Direction (46-50): Read the given passage as implementers of schemes from State carefully and answer the questions that follow. capitals. D. The process by which the prescribed The world is leaning on its biggest economy to Central schemes are reformulated by local sustain the global recovery, according to the elected leaders. International Monetary Fund. The fund left its E. None of these forecast for global growth unchanged in the 43. What is the "dilemma" at the intra-State level latest quarterly update to its World Economic mentioned in the first paragraph of the Outlook, released Monday in Kuala Lumpur. passage? The world economy will expand 3.4 percent

6 www.gradeup.co

this year, up from 3.2 percent in 2016, and by outlook. For 2018 the fund sees Chinese 3.6 percent next year, the IMF said. The growth at 6.4 percent, an increase of 0.2 forecasts for this year and next are points from three months ago. In the report, unchanged from the fund’s projections in the IMF looked for average annual growth of April. Beneath the headline figures, though, 6.4 percent in China during 2018 through the drivers of the recovery are shifting, with 2020. “Rich market valuations and very low the world relying less than expected on the volatility in an environment of high policy US and the UK and more on China, Japan, the uncertainty raise the likelihood of a market euro zone and Canada, according to the correction, which could dampen growth and Washington-based IMF. confidence, said the fund, which also cited The dollar fell to its lowest in 14 months last China’s credit growth and protectionist week as investors discounted the ability of policies as threats. President Donald Trump’s administration to While risks to the global outlook are “broadly deliver on its economic agenda after efforts by balanced” in the near term, medium term the Republican Senate to overhaul healthcare risks are titled to the downside, the IMF said. collapsed. The IMF estimated US growth at IMF urged advanced countries with weak 2.1 percent this year and again in 2018, demand and low inflation to continue consistent with what the fund said June 27 in supporting growth through monetary and its annual assessment of the US 2.3 percent fiscal policy while cautioning central banks and 2.5 percent, respectively, in 2017 and against raising borrowing costs too quickly. 2018. The economy expanded by 1.6 percent The fund said widespread protectionism or a in 2016. “US growth projections are lower “race to the bottom” on financial and than in April, primarily reflecting the regulatory oversight would leave all countries assumption that fiscal policy will be less worse off. expansionary going forward than previously 46. Which of the following statements can anticipated,” the IMF said in the latest report. definitely be concluded from the given In June, the IMF said it had dropped passage? assumptions of a boost to growth from (i) IMF does not perceive any considerable Trump’s plans to cut taxes and increase long term risk to global market in the short infrastructure spending. Trump’s budget term director, Mick Mulvaney, wrote in July that the (ii) IMF views China’s credit and growth policy administration’s goal is “sustained 3 percent as a risk to global outlook economic growth,” and he named the (iii) The US can perform well in global outlook program “MAGAnomies” after Trump’s by abandoning its protectionist policies campaign slogan, “Make America Great A. Both (i) and (ii) Again”. B. Both (i) and (iii) Meanwhile, as the UK works through its Brexit C. Only (i) negotiations, the IMF also chopped its D. Only (ii) forecast for UK growth this year by 0.3 E. Only (iii) percentage point to 1.7 percent on weaker- 47. Which of the following is TRUE in the context than-expected activity in the first quarter. of IMF’s growth forecast of China? “This forecast underscores exactly why our (i) An annual average growth of 6.4 percent plans to increase productivity and ensure we during 2018 through 2020 get the very best deal with the EU, are vitally (ii) No change in its forecast for 2017 from its important,” the UK Treasury said in an Annual Staff report of June 14th emailed statement. “The fundamentals of our (iii) The forecast for 2018 has grown a economy are strong.” meager 0.2 percent from previous quarter Other countries are picking up the slack. The A. Only (ii) and (iii) IMF’s projection for growth in China is 6.7 B. Only (i) and (ii) percent for 2017 – the same as its estimate C. Only (i) and (iii) made June 14 in an annual staff report, and D. None of these up 0.1 point from April’s world economic E. All of the above

7 www.gradeup.co

48. IMF has cut growth forecast of UK by 0.3 52. I. Cyber-crime is at ______proportions and percent. What does it imply for the UK? the chance that your website, your name and A. It should increase infrastructure spending your business can be ______affected are and cut taxes more of a reality than ever before. B. It made a mistake to withdraw itself from II. An/a ______of smallpox resulted in the Europe death of a substantial portion of the native C. It should work on its fundamental for faster population of this country and has ______economic recovery affected the population of the nation. D. It should ensure best deal out of Brexit A. Epidemic, severely negotiations B. Severely, destroyed C. Splutter, severely E. None of these D. Sagacious, Juxtaposed 49. According to the IMF, the advanced countries E. None of these with weak demand and low inflation should- 53. I. Wood windows will require maintenance (i) discard their protectionist policies and re-painting to ______appearance and (ii) support growth through monetary and ______of the wood against rot and insects. fiscal policy II. Even though they were tired mentally and (iii) keep a check on rapid increase in rate of physically, the soldiers continued to ______interest by central banks their duty to serve their country with ______A. Both (i) and (ii) B. Both (ii) and (iii) and honour. C. Both (i) and (iii) D. Only (i) A. Lurk, germane B. Maintain, Integrity E. Only (ii) C. Garnish, Integrity D. Profess, Heft 50. Which of the following factors can be E. None of these attributed to the fall of dollar to its lowest in 54. I. The Black Student ______is holding a the 14 weeks period? cultural _____ workshop in the media centre A. The US’ diminishing role in global recovery on Saturday. B. The emergence of Japan, China and Euro II. We realized that it was a _____ of people Zone as the driver of global recovery who were against religious ____ in the C. The protectionist policies of Trump society. D. The inability of Trump to keep his A. Jocular, Adept B. Convulsion, Niggle economic promises C. Huff, Chasm D. Caucus, diversity E. None of these E. Less expansionary fiscal policy of the US than expected Direction (55-58): A sentence is divided into three parts I, II and III. For each part a Direction (51-54): In the given question, correction statement is given. Determine the two sentences with two blanks in each are part which requires correction and mark it as followed by five options with two words in your answer. each. Select that option as your answer which 55. Rapid economic ascendance has brought can fill both the blanks of both the sentences I many challenges, / including high inequality; and II. rapid urbanization; challenges of, / 51. I. When deciding the term of the loan, base environmental sustainability; and external your _____ on the loan’s total cost and not imbalances. _____ payments. I. Rapid economic ascendance has brought on II. The ______to pay the workers on a many challenges monthly basis is very beneficial for small firms II. high inequality; rapid urbanization; to keep a check on their ______profit. challenges to A. Monthly, advance III. environmental sustainability; and external B. Decision, monthly imbalance C. Determine, yearly A. Only III B. Both I and II D. Mention, pay C. Both I and III D. Only II E. None of these E. Only I

8 www.gradeup.co

56. The academic literature provides mixed Direction (59-61): In the given findings, / for the relationship among question, a statement with one blank, immigration and crime worldwide, / but finds followed by three alternatives, is given. Out of for the United States that immigration has no the given three alternatives, more than one impact on the crime rate. can fill the given blank. Choose the set of I. The academic literature provided mixed words from the given options which can be findings used to fill the blank. II. for the relationship between immigration 59. Despite having a border dispute India and and crime worldwide, China have ______thereby creating a ‘managed relationship’. III. but finds for the United States that i. Never exchanged a bullet on the border in immigration is in no impact on the crime rate. the last 40 years A. Only II B. Only III ii. Preserved accord C. Both I and II D. Both II and III iii. Not been belligerent towards each other E. Both I and III A. Only i B. Only ii 57. Fascism has many faces, but the most C. Both i and iii D. Both ii and iii constant is that the supremacist delusion that E. All the above the West is the carrier of “universal 60. The need for rural communities to approach values”/and that, as exclusive interpreter and development from a wider perspective has custodian of these values; /the West is ______rather than merely creating incentive obligated to act upon watchdog of democracy for agricultural or resource-based businesses. and human rights throughout the globe. i. Made things difficult and more complex I. Fascism has many faces, but the most ii. Been a burdensome and strenuous task constant is that of the supremacist delusion iii. created more focus on a broad range of that the West is the carrier of “universal development goals values” A. Only i B. Only ii II. and that, as an exclusive interpreter and C. Only iii D. Both i and ii curator of these values; E. All the above III. the West is obligated to act as a watchdog 61. The best way to end violence against women of democracy and human rights throughout and girls is to prevent it from happening in the globe the first place by ______. i. addressing its root and structural causes. A. Both I and II B. Both II and III ii. redressing the wounds caused by it. C. All I, II and III D. Only I iii. Curbing it wherever visible. E. Both I and III A. Only i B. Only ii 58. Several times we sat with the new people and C. Only iii D. Both i and ii tried to put together agreements / that would E. Both ii and iii accommodate the needs of both of our peoples, / each time it was us who gave Direction (62-64): In the given question, privileges to the new people. four statements are given which are I. Several times we sat with the new people connected in different ways in the options and tried to put forward agreements given below. Determine the option in which the four statements have been the most II. That would accommodate the needs for appropriately expressed. both of our peoples, 62. The weather was stormy. The fishermen had III. each time it was we who gave privileges been warned the previous night. The to the new people. fishermen were in danger. Anyone could have A. Only II got drowned in such weather. B. Only III A. The weather was so stormy that anyone C. Both II and III could have got drowned and the fishermen D. Both I and II were in danger, so they had been warned the E. None of these. previous night.

9 www.gradeup.co

B. The weather was stormy enough to have 65. Direction: In the given question, there are the fishermen in danger as they had been four statements A, B, C and D that have to be warned the previous night and anyone could arranged in a logical order to make a have got drowned in such weather. paragraph between 1 and 6. C. The fishermen had been warned the 1) In a move that will encourage banks to previous night as they were in danger in the lend more for housing and possibly make weather was stormy and anyone could have high-value home loans cheaper, the Reserve got drowned in such weather. Bank of India (RBI) on Wednesday reduced D. The fishermen were in danger as the the capital requirement for home loans. weather was stormy as they had been waned A) The repo rate is the rate at which it lends the previous night and anyone could have got to banks, while the reverse repo is the facility drowned in such weather. through which it borrows from banks. E. None of these B) It also retained the cash reserve 63. Timothy was with Philip since he was a child. requirement (CRR) at 4%. Timothy was a playful dog. Philip considered C) The RBI has also improved the lendable Timothy to be a special dog. Philip thought resources of banks by Rs 50,000 crore by Timothy could always protect him. reducing the proportion of deposits that banks A. Timothy was with Philip since he was a have to invest in government bonds. child and was considered to be special by him D) In its monetary policy review, the RBI who was a playful dog and could always retained the repo rate at 6.25% and the protect him. reverse repo rate at 6%. B. Timothy was with Philip who considered 6) Significantly, however, the RBI cut its him to be a special dog and could always projection for consumer inflation to 2-3.5% in protect him as he was with him since he was the April to September period, down from a child and was a playful dog. 4.5%, and to 3.5-4.5% in October to March, C. Timothy was a playful dog who was with down from 5%. Philip since he was a child and was considered A. CDAB B. DABC a special dog who could always protect him. C. DBAC D. CBDA D. Timothy was with Philip who was a playful E. BACD child and considered him to be a special dog 66. Direction: In the given question, there are and could always protect him. four statements A, B, C and D that have to be E. None of these arranged in a logical order to make a 64. The situation was difficult. The girl was only paragraph between 1 and 6. seven years old. The girl had no relative to 1) Quaint is not an obvious word to use look after her. The financial condition too was about America: a country built on revolution, grave. restless expansion and unabashed pursuit of A. The situation was difficult and grave as the profits. financial condition was bad and the girl was A) Often physical settings added to this sense only seven years old had no relative to look of quaintness. after her. B) From Boston to Philadelphia, or to the B. The situation was difficult as the girl who lovingly-restored Georgian streets of was only seven years old had no relative to colonial Williamsburg in Virginia many New look after her and the financial condition was World candles of liberty looked strikingly like grave as well. the Old all red-brick mansions cobbled lanes C. The girl was only seven years old and the and candle-lit inns, haunted by the host in financial condition was too grave as she had tricorne hats. no relative to look after because the situation C) Museums and historic sites depicted the was difficult. birth of the United States as a morality tale D. The situation was grave and difficult and an Anglo-Saxon family dispute, pitting however the girl was only seven years old and tyrannical King George and his redcoats had no relative to look after her and the against freedom-loving colonial subjects. financial condition was grave. D) Yet for years a cloud of quaintness hung E. None of these about many of the country's founding places.

10 www.gradeup.co

6) This can also be found in several other B. and at that time there was only one regions of the great America. national channel A. DACB B. DCAB C. and at that time there was alone one C. BACD D. ABCD national channel E. DBAC D. and of that time there was only one

Direction (67-70): In the following question, national channel a part of the sentence is underlined. Below E. and at there time there was only one the sentence alternatives to the underlined national channel part are given at (A), (B), (C), (D) and (E) 70. India has some of the most biodiverse regions which may help improve the sentence. Choose in the world and hosts four of the world’s 35 the correct alternative. biodiversity hotspots – or treasure-houses – that is the Western Ghats, the Eastern 67. The Sanskrit language is of a wonderful Himalayas, Indo-Burma and Nicobar Islands. structure; more perfect then the Greek, more copious than the Latin, and more exquisitely A. India had some of the most biodiverse refined than either, yet bearing to both of regions of the world and them a stronger affinity. B. India has some of a most biodiverse A. is of a wonderful structure; more perfect regions of the world and than the Greek, most copious than the C. India has some of the most biodiverse B. is of a wonderful structure; more perfect regions of the world and then the Greek, more copious then the D. India has some of the most biodiverse C. is of a wonderful structure; more perfect region of the world and than the Greek, more copious than the E. India has some of the most biodiverse D. is of a wonderful structure; more perfect regions of the world or than the Greek, more copious than a 71. Direction: In the following question, a part of E. are of a wonderful structure; more perfect the sentence is printed in bold. Below the than the Greek, more copious than the sentence alternatives to the bold part are 68. Field hockey was considered to be the given at (A), (B), (C) and (D) which may help national game of India, but this had been improve the sentence. Choose the correct recently denied by the Government of India, alternative. In case the given sentence is clarifying on a Right to Information Act (RTI) correct, your answer is (E) i.e. No correction filed that India has not declared any sport as required. the national game. The need for conservation of wildlife in India A. is considered to be the national game of is often questioned because of the India, but this has been recently denied apparently incorrect priority on the face of B. was considered to be the national game of direct poverty of the people. India, but this has been recently deny A. incorrect priority of the fact of direct C. was considerable to be the national game poverty of the people of India, but this has been recently denied D. was considered to be a national game of B. incorrect priority in the face in direct India, but this has been recently denied poverty of the people E. was considered to be the national game of C. incorrect priority in the mouth of direct India, but this has been recently denied poverty of the people 69. Indian small screen programming started off D. incorrect priority in the face of direct in the mid-1970s where at that time there poverty of the people was only one national channel Doordarshan, E. No correction required which was government owned but 1982 saw a revolution in TV programming in India, with Direction (72-75): In the given question, a the New Delhi Asian games, India saw the theme followed by three passages is given. colored version of TV, that year. Determine which passage is based on the A. and at that time their was only one given theme and mark it as your option. More national channel than one passage can be based on the given theme.

11 www.gradeup.co

72. Women empowerment A. Only B B. Only C A) With the slogan of women empowerment, C. Both A and B D. All A, B, C the question arises that “are women become E. None of the above really strong” and “is long-term struggle has 74. Money versus peace of mind ended”. Many programmes have been A) Peace of mind is the most important thing implemented and run by the government such in life, nothing can replace it not even heaps as International Women’s day, mother’s day, of money. Money is just a commodity, it can etc in order to bring awareness in the society be earned as well as spent in exchange of about the true rights and value of the women worldly pleasures, but peace of mind is a in the development of the nation. Women state. need to be progressed in the number of B) Since we have done away with the barter spheres. system, we’ve developed various forms of B) There is a high level of gender inequality in currency and coins. Money is one of the most India where women are ill-treated by their important factors determining one’s success family members and outsiders. Women in and life in today’s world. It is true that 40% of India are always subjected to the honour our happiness depends on our actions. Out of killings and they never given their basic rights the remaining 60%, 50% is via the genes and for the proper education and freedom. They 10% via uncontrollable factors. You can’t lose are the victims who have faced violence and the important 40% determiner of your abuse in the male-dominated country. happiness for something called money, can C) Giving priority to the gender equality you? facilitates women empowerment all over the C) Today, everyone is running: some are country. To get the high-level goal of women running after money, some are running after empowerment, it should be promoted from the people they love, some are running after the childhood in each and every family. It the situations they desire. No one is at a needs women to be strong physically, stable position i.e. no one is at peace. Inner mentally and socially. peace reflects the peace of mind. In today’s A. Only C B. Both A and B era, everyone is extremely busy in making C. Both A and C D. All A, B, C their lives perfect. E. None of the above A. Only B B. Only C 73. Competition in the e-commerce sector C. Only A D. Both B and C A) All of us want to come out on top. We want E. All A, B, C to see others subordinated to our superiority. 75. Impact of Artificial Intelligence Our goal is to dominate. If you do not A) Artificial intelligence today is properly secretly, or overtly, harbor this desire, then known as narrow AI (or weak AI), in that it is you should not be in e-commerce. If you see designed to perform a narrow task (e.g. only some good ideas, note them, and implement facial recognition or only internet searches or them on your own site. only driving a car). However, the long-term B) If you want to beat your competition, you goal of many researchers is to create general are going to have to establish the best supply AI (AGI or strong AI). While narrow AI may chain and maintain the best wholesaler outperform humans at whatever its specific relationships. This does not always mean the task is, like playing chess or solving cheapest source. If you choose the cheapest equations, AGI would outperform humans at wholesaler or distributor, you may end up nearly every cognitive task. compromising on quality or reliability. B) Autonomous weapons are artificial C) The market for e-commerce solutions is intelligence systems that are programmed to extremely competitive and we may find kill. In the hands of the wrong person, these ourselves unable to compete effectively. weapons could easily cause mass casualties. Because there are relatively low barriers to Moreover, an AI arms race could inadvertently entry in the e-commerce market, we expect lead to an AI war that also results in mass continued intense competition as current casualties. competitors expand their product offerings and new competitors enter the market.

12 www.gradeup.co

C) One of AI’s greatest impacts could be in A. 27 kmph B. 25kmph food production — an industry challenged by a C. 26 kmph D. 28 kmph rapidly growing world population, competition E. Can’t determine for natural resources and plateauing 77. If the time taken by boat to travel upstream agricultural productivity. The Food and Agriculture Organization of the United Nations on Monday is hrs more than the time (FAO) estimates that the earth’s population taken by it to travel downstream on the same will balloon to 9.7 billion people on by 2050 day, then find the speed of boat in still stream A. Only A B. Only B on Monday if speed of boat in still water is 10 C. Both A and B D. Both A, B, C kmph E. All A, B, C A. 2.2 kmph B. 1.8 kmph

C. 2.0 kmph D. 2.5 kmph Direction (76-80): Study the data given E. Can’t be determined below and answer the following question. The pie charts shown below shows the distance 78. If the speed of boat in still water on Friday covered by a boat moving upstream and was 22 km/hr and the speed of boat in still downstream in different days of a week. And water on Tuesday was 10/11 times of Friday and time taken to travel downstream on the table shows the speed of stream in km/hr. in different days of a week. Friday is 20/11 hrs more than the time taken by it to travel downstream on Tuesday, then find the speed of stream (in kmph) on Wednesday? A. 1.5 B. 2 C. 4 D. None of the above E. Can’t be determined 79. The speed of boat in still water on Saturday was 20 km/hr. if the time taken by boat to travel downstream on Saturday is 25/32 times taken to travel upstream on Sunday, then find the time taken by the boat to cover a distance of 21.25 km upstream when the speed of stream is same as that of Sunday.

A. 3 hrs. B. 2.5 hrs.

C. 4 hrs. D. None of these E. Can’t be determined 80. If the time taken by boat to travel upstream on Sunday is 6 hours more than the time taken by it to travel downstream on Thursday and the speed of boat in still water on Thursday is 13 kmph, then find the upstream speed of boat on Sunday? A. 17 kmph B. 20 kmph C. 12 kmph D. 15 kmph E. Can’t be determined

Direction (81-84): There are three bags A, 76. If the time taken by boat to travel upstream B and C. In each bag there are three types of on Sunday is equal to the time taken by it to coloured balls Yellow, Green and Black. travel downstream on Thursday and the speed In bag A, no. of yellow coloured balls are y of boat in still water on Thursday is 13 kmph and no. of green coloured balls are g. Number then find the speed of boat in still water on of green coloured balls are 4 more than the Thursday? number of yellow coloured balls. When one ball is picked at random then the probability

13 www.gradeup.co

Directions (85-86): Each of A, B, C and D of getting black colour ball is , The value of need a unique time to do a certain work. A can do the work in x days and B can do the y is less then g. In bag B, number of work in 2x days. A started the work and do it for days then he is replaced by B and B yellow coloured balls is more than completed remaining work in same time as C that of bag A. If two balls are picked at and D together can complete the whole work. random from bag B then the probability of The ratio of the efficiency of C and D is 4 : 5. 85. If C and D work for alternative days starting getting both green colour ball is . Total from C then they can do the total work number of balls in bag B is 75. In bag C, the ratio of number of green in days. coloured balls and number of black coloured Then find x value? balls is 7 : 5. Total number of green and black A. 33.33 B. 66.66 coloured balls is 36. If one ball is picked at C. 30 D. 70 random then the probability of getting one E. 80 86. If E and F together work for 24 days then yellow ball is they are replaced by A and B respectively 81. If x number of yellow balls from bag B are then they can do the remaining work in 20 taken and placed into bag A and 20% of black days. If the efficiency of E and F is 5 : 4, If E balls from bag A are taken and placed into in and F together complete the whole work then bag B. If we pick one ball from bag B then the find the difference between the work done by E alone and the total work done by F alone ? probability that the ball is of black colour is A. 1/7 B. 1/9 . Then find the value of x? C. 1/6 D. 3/7 A. 2 B. 4 E. 1/3 C. 1 D. 1/2 87. ABCD is a trapezoid. PQRS and MLKJ are two E. 3/2 rhombus. Diagonal of PQRS are 6 cm and 8 82. If one ball picked at random from each of the cm. One of the angle of MLKJ is 120 degree bag A and bag B then find the probability that and the diagonal bisecting that angle both of the balls are of the same colour? measures 15 cm. Side of PQRS = AB, side of MLKJ = CD. Find MN (median of trapezoid) A. B.

C. D. None of the above E. Can’t be determined 83. Difference between the number of green balls in bag A and bag C is how much percent more/less than the sum of the number of A. 15 B. 20 black balls in bag A and bag C together? C. 10 D. 12 A. 97.5% B. 91% E. Can’t determine C. 75% D. 79.5% 88. A vessel contains 2.5 litres of water and 10 E. None of the above litres of milk. 20% of the contents of the 84. Ram counted balls in Bag C and Bag A but vessel are removed. To the remaining mistakenly he counted green balls in bag B contents, x litres of water is added to reverse instead of C. What is error percentage the ratio of water and milk. Then y litre of (approx.)? milk is added again to reverse the ratio of A. 3% B. 2% water and milk. Find y. C. 5% D. 4% A. 128 B. 120 E. 1% C. 60 D. 30 E. 130

14 www.gradeup.co

Directions (89-92): A,B,C,D and E are five Directions (93-96): Find the correct persons to complete jobs X and Y. Line graph relationship between the given quantities: shows the data regarding the number of days taken by these persons to complete the job X 93. Quantity 1: Find A value where S is an acute and Y. angle and PQ||RS

Quantity 2: A=25

A. Quantity 1 < Quantity 2

B. Quantity 2 < Quantity 1 89. A and B working on job Y alternatively for 10 C. Quantity 1 = Quantity 2 days. C and D worked alternatively for n days. D. Can’t establish a relation If 1/15 of the job is remained then how many E. None of the above days c alone can finish the job? 94. There are 63 cards in a box numbered from A. 14 01 to 63. Every card is numbered with only 1 B. 12 number. C. 13 D. None of the above Quantity 1: Probability of picking up a card E. Can’t be determined whose digits, if interchanged, result in a 90. If A worked on job X for 5 days each and C number which is 36 more than the number worked for 3 days. the remaining job was picked up. done by E and D. If the ratio of the number of Quantity 2: Probability of picking up a card, days for which D and E worked is in ratio 1:1, the number printed on which is a multiple of 8 then find the number of days for which C,D but not that of 16. and E worked? A. Quantity 1 < Quantity 2 A. 6 B. 3 B. Quantity 2 < Quantity 1 C. 9 D. 5 C. Quantity 1 = Quantity 2 E. Can’t be determined D. Can’t establish a relation

91. If the ratio of the number of days for which D E. None of the above and C worked on job Y is 12:5 and completed 95. Quantity 1: Number of hours in which A, B the work then find the difference between and C can complete the work if A, B and C number of days for which C and D worked. alone can finish it in 25, 20 and 15 hours A. 7 B. 14 respectively. C. 21 D. Can’t be determined Quantity 2: Number of hours in which B can E. All of above finish the work if A working alone finishes it in 92. If B worked on job Y with 4/9 times his work 20 hours while working together they finish it efficiency and assisted by D every 3rd day in 5 hours. then find the number of days for which B and A. quantity 1 > quantity 2 D worked to complete the job. B. quantity 1 ≥ quantity 2 A. 18 B. 13 C. quantity 1 ≤ quantity 2 C. 16 D. 12 D. quantity 1 < quantity 2 E. None of the above E. quantity 1 = quantity 2

15 www.gradeup.co

96. Quantity 1: In an examination, Ankita scored A. I and II B. I and III 35 marks less than Puneeta. Puneeta scored C. Only III D. I and either II or III 65 more marks than Meenakshi. Rakhi scored E. Any two of the three 115 marks, which is 20 marks more than 99. Directions: The question given below is Meenakshi's. Simpy scored 108 marks less followed by two statements. You have to than the maximum marks of the test. What determine whether the data given in the approximate percentage of marks did Simpy statements are sufficient for answering the score in the examination, if she got 67 marks question. You should use the data and your more than Ankita? knowledge of Mathematics to choose between Quantity 2: The length of a rectangle is the possible answers. increased by 60%. By what percent would the What is the rate of interest Percent per width have to be decreased to maintain the annum? same area? I. An amount doubles itself in 5 yr on simple A. Quantity1 < Quantity2 interest. B. Quantity1 ≤ Quantity2 C. Quantity1 ≥ Quantity2 II. Difference between the compound interest D. Quantity1 > Quantity2 and the simple interest earned on a certain E. None of these amount in two years is ₹ 400. 97. Directions: The question given below is III. Simple interest earned per annum is ₹ followed by two statements. You have to 2000 determine whether the data given in the A. Only I statements are sufficient for answering the B. II and III question. You should use the data and your C. Any two of three knowledge of Mathematics to choose between D. Either I alone or II and III together the possible answers. E. Either I and II or I and III How many students from institute 'A' got the 100. Direction: The question given below is placement? followed by some statements. Read the I. Number of students studying in institutes A question carefully and determine which of the and B are in the ratio of 3:4 respectively. given statements is/are necessary/required to II. Number of students who got placement answer the question. from institute B is 120% of the number of Find the two-digit number? students who got placement from institute A I. The difference between the number and the III. 80% of the students studying in institute number formed by interchanging the digit is B got placement. 27 A. I and II both B. Only I C. Only II D. Any one of the three II. The difference between two digits is 3. E. Question cannot be answered even with the III. The digit at unit's place is less than that at information in all three statements ten's place by 3 98. Direction: The question given below is A. I and III B. I and either II or III followed by some statements. Read the C. II and III D. All of these question carefully and determine which of the E. None of these given statements is/are necessary/required to 101. Directions: The following question is followed answer the question. by few statements. You have to study the What is the cost of flooring the rectangular question and all the statements given and hall? decide whether the information provided in I. Length and the breadth of the hall are in the statement(s) is sufficient/required to the ratio of 3:2 answer the question or not: II. Length of the hall is 48 m and cost of If k is an integer and x(x – k) = k + 1, what is flooring is ₹ 850 Per sq.m. the value of x? III. Perimeter of the hall is 160 m and cost of I. x < k flooring is ₹ 850 per sq m II. x = k + 1 III. x = 3 – k

16 www.gradeup.co

A. Only statement I is sufficient. A. 15% B. 12% B. Only statement II is sufficient C. 20% D. 16% C. Any of the two statements out of I, II and E. 25% III are sufficient 104. Company F marked item IV at 25% higher D. All three are together sufficient than its cost price. Selling price of this item is E. None of these Rs. 4140. Find the marked price of item IV, if ‘d’ is discount% and profit% made by selling Direction (102-106): Study the given table the item is (3d – 9)%. carefully and answer the questions that A. Rs. 4400 follow. B. Rs. 4500

Given below is a table which tells us about the C. Rs. 4680 profit or loss% made on 4 items sold by 6 D. Rs. 5140 companies, A, B, C, D, E, and F. E. None of these Some of the data is missing in the table which 105. The selling price of the items sold by one has to calculate using the given data. Company E was Rs. 2400, Rs. 2100, Rs. 1870 and Rs. 2030 in the order from I to IV. They made a profit on items I and IV while they lost money on item II. Find the profit or loss made on item III if the company made an overall profit of 5% by selling all the items. A. Rs. 200 B. Rs. 400 C. Rs. 500 D. Rs. 420 E. Rs. 560 106. The average selling price of item I sold by company C and company A is Rs. 3493. The cost price of the item I for company A and company C is in the ratio of 5 : 4. Find the % 102. The ratio of the selling price of item I and II by which the selling price of this item sold by for company A is 4 : 3. Find the profit or Company C is greater or less than the selling loss% made on item II if the ratio of their cost price of this item sold by company A. Both price was 8 : 9. It is known that company A companies made profit on the item. made profit on item I. A. 17.2% B. 18.54% C. 19.25% D. 20.6% A. 12.25% profit B. 12.25% loss E. 21.36%

C. D. Directions (107-108): The following E. None of these information should be used to solve the following 2 questions. The information given 103. The marked price of items are same for each in a question or the answer may be used in company. The marked price of items are: the other question as the 2 questions are Item I – Rs. 2500, item II – Rs. 1750, item III linked. Please solve carefully before – Rs. 2400, item IV – Rs. 3200. For company proceeding: C, a discount% of 10% is given on item III while two successive discounts of the equal % 3 partners – A, B, and C – invested in a was given on item I by company D. Find the business through different means – cash successive discount% given by company D if capital, land assets and equity certificates. the cost price paid by company C for item III Cash invested by the 3 were in the proportion was equal to the cost price paid by company 2:3:7 respectively; land assets were invested D for item I. Company C made profit on item in the ratio 4:3:5 respectively; and equity III while company D suffered loss on item I. certificates were invested in proportion 1:5:4 respectively.

17 www.gradeup.co

107. If total cash invested was Rs. 42 lakhs, total Step 1- Diagonally interchange second letter, land worth invested was Rs. 93 lakhs, total if box contain only one letter then consider it value of equity certificates was Rs. 65 lakhs, as a first letter and no interchange process and all invested for the same time, then what takes place. will be profit share of ‘A’ if the total profit at the end of 1st cycle is Rs. 1.5 crores? A. Rs. 45.65 lakhs B. Rs. 37.755 lakhs C. Rs. 33.375 lakhs D. Rs. 29.895 lakhs E. None of the above 108. In the next cycle the investments made were Step 2- doubled but in the same proportion as in the I) if box contain even number previous cycle. However, the period they were a) If one letter is consonant and other one is invested changed for this cycle. A invested for vowel then vowel increase by 4 values and 10 months, B invested for 2.5 years and C opposite it and consonant letter first opposite invested for 20 months. What is C’s then increases by 5 values and interchange approximate profit share if the total profit this both positions. time was Rs.3 crores? b) If both letters are vowel then increases A. Rs. 2.25 crores B. Rs. 2.50 crores both letters position by 5 values and C. Rs. 1.63 crores D. Rs. 1.27 crores interchange their positions and opposite both E. Rs. 2.22 crores letters.

Direction (109-110): Study the following c) If both letters are consonant then information carefully and answer the question decreases both letters positions by 6 values given billow: and interchange their positions and opposite Two trains A & B are travelling in the same both letters. direction at a certain speed. Length of the d) If there is only one letter either vowel or train A is 300 metres and the speed of the consonant, if letter is vowel then vowel letter train B is 30km/hr. The train A passes a man decreases by 7 values and opposite this vowel who is walking at a speed of 4km/hr along the letter and then increases by 6 values. If this line in the same direction as ‘A’ in 30 letter is consonant then increase letter value seconds. by 7 and opposite this consonant letter and 109. If the train A reached the station in 15 then decrease by 6 values. minutes after it had passed the man. In what II) If box contain odd number time the man reached the station? a) If one letter is consonant and other one is A. 1 hour B. 1.5 hours vowel then vowel decrease by 4 values and C. 2 hours D. 2.5 hours opposite it and consonant letter first opposite E. Data insufficient then decreases by 5 values and interchange 110. If the train A passes the train B completely in both positions. 5.4 minutes find the length of the train B in b) If both letters are vowel then decreases metres? both letters position by 5 values and A. 500m B. 750m interchange their positions and opposite both C. 822m D. 850m letters. E. 600m c) If both letters are consonant then increases Directions (11-112): Study the following both letters positions by 6 values and diagram and convert it into other diagrams by interchange their positions and opposite both implementing the instructions which is given letters. in each step to get next step. d) If there is only one letter either vowel or consonant, if letter is vowel then vowel letter increases by 7 values and opposite this vowel letter and then decreases by 6 values. If this letter is consonant then decrease letter value by 7 and opposite this consonant letter and

then increase by 6 values.

18 www.gradeup.co

A. If the inference is definitely true, i.e., it properly follows from the statement of facts given. B. In the inference is probably true though not definitely true in the light of the facts Step 3-is coded in some special pattern given. C. If the data are inadequate i.e., from the facts given you cannot say whether the inference is likely to be true or false. D. If the inference is probably false though not definitely false in the light of the facts

given. As per the rules followed in the above step, E. If the inference is definitely false i.e., it find out the appropriate steps for the given cannot possibly be drawn from the facts given input and answer the following questions. or it contradicts the given facts. 114. The fiscal position of railways in the earlier plan period was better than the current plan period. A. If the inference is definitely true, i.e., it

properly follows from the statement of facts 111. Which element comes in step-3 in the second given. column of third row? B. In the inference is probably true though A. SS8 B. TE6 not definitely true in the light of the facts C. NR2 D. H7 given. E. None of these C. If the data are inadequate i.e., from the 112. Which element replaces IE6 in step-3? facts given you cannot say whether the A. NR2 B. H7 inference is likely to be true or false. C. TE6 D. SS8 D. If the inference is probably false though E. None of these not definitely false in the light of the facts

given. Direction (113-114): In the form of E. If the inference is definitely false i.e., it inference/conclusions are based on the cannot possibly be drawn from the facts given passages given below. Each passage is or it contradicts the given facts. followed by five inferences. You are required to examine each inference separately in the Direction (115-117): Study the information context of the passage and decide upon its given below and answer the questions based degree of truth or falsity. Give answer on it. More than a decade of abrasion in budgetary Eight persons A, B, C, D, E, F, G and H are assistance and support from the Union sitting in a straight line and facing north. No government, has seriously affected Indian two successive persons are sitting together Railways’ capacity to finance its plan according to the alphabetical order. (I.e. A expenditures. The situation has come to a cannot sit with B, Similarly B cannot sit with pass where the railways must now think of either A or C). They like different colours i.e. novel and radical ways to get longer mileage Red, Blue, Black, Yellow, Brown, Pink, Silver from its investments. Significantly the and White, not necessarily in the same order. resource crunch has had grievous impact on B is 3rd to the left of C who likes Blue. One the railways. As a result, it will not be in a person sits between B and the one who likes position to acquire necessary equipment and Black. E is 2nd to the right of C. The one who this will seriously affect the railways’ capacity likes Brown is immediate left of E. One person to serve the needs of the economy in future. sits between the one who likes Brown and the 113. Railways had so far believed in conventional one who likes Pink. D likes Red and sits 2nd to ways in inducing income. the right of F. G is not neighbour of C. A

19 www.gradeup.co

doesn’t like Brown. The one who likes Silver is 120. A train is scheduled to leave the station at immediate left of the one who likes White. ‘@@’. A person has reached the station 20 115. Who among the following likes Yellow? minutes before the train’s scheduled time. At A. B B. A what time the person has reached the station? C. E D. H A. @# B. @% E. G C. %@ D. %# 116. Who among the following is 3rd to the right of E. $+ the one who likes Blue? A. No one B. G Direction (121-125): Study the information C. E D. H given below and answer the questions based E. A on it.

117. How many persons sit between B and the one 6 cars- M, N, O, P, Q and R are parked in a who Red? straight line not necessarily in the same A. 0 B. 1 order. Distance between each car is C. 2 D. 3 successive multiple of 3 but not necessarily in E. 4 the order. The distance between car N and car O is 36m and no car is parked between them. Direction (118-120): Study the following The distance between car M and car O is information carefully and answer the 102m. Car R is 99m to the right of car N. Only questions given below: one car is parked between car O and car R.

‘#’ – Either the hour or minute hand of clock Car M is parked to the immediate left to car P. on 3 The distance between car M and car P is 12m more than the distance between car P and car ‘$’ – Either the hour or minute hand of clock N. The distance between R and M is more on 12 than 60m. The distance between car P and car ‘%’ – Either the hour or minute hand of clock Q is 93m. If car Q moves 20m to the north on 4 then takes a left turn and moves 50m then '@' – Either the hour or minute hand of clock again takes a left turn and moves for 10m on 8 and stops at point Z. Car A is 16m to the west ‘+’ – Either the hour or minute hand of clock of point Z. Car A moves 66m towards west on 5 and stops at point Y. Example: Time ‘#%’ represents 3:20PM All 121. In which of the following direction is point Z the times are in PM. with respect to car N? The first symbol represents hours and second A. South-east B. North-east symbol represents minutes C. North-west D. South-west 118. A trains reaches station at time '% @' If it E. North gets late by 3 hours 35 minutes, then at what 122. What is the distance between car P and car R? time it reaches the station? A. 132m B. 112m A. @+ B. @@ C. 126m D. 144m C. @# D. #+ E. 99m E. %# 123. Point Y is in which of the following direction 119. A person has to catch a train that is scheduled and distance with respect to car M? at ‘@+’. It takes the person 5 hours to reach A. 10m, South B. 10m, North the railway station from his home. At what C. 20m, South D. 20m, North time should he leave from his home for the E. 10m, North west railway station to arrive at the station to 124. What is the distance between point Z and reach exactly 5 minutes before the departure Point Y? of the train? A. 82m B. 88m A. #@ B. @# C. 84m D. 76m C. ## D. #% E. 66m E. %%

20 www.gradeup.co

125. What is the maximum distance between two and reduction in the need for additional work cars? force) annual sales output has only risen by A. 171m B. 175n 10% this year. It appears that Joe’s C. 165m D. 163m innovation has caused the reduction in the E. 172m annual growth rate.

Direction (126-127): Each passage is Which of the following, if true, would most followed by a set of five statements. Answer seriously weaken the conclusion above? according to the directions given for each A. The investment in computerization has a question. provision for depreciation of the cost of the computers Whenever ISIS carries out a new atrocity, B. Increases in selling price did not follow whether it's beheading a group of Egyptian increases in the cost of the inputs. Christians or enslaving Yazidi women in Iraq C. The innovation brought in by the new or burning its victims alive, the big question trainer were intended as long term most people have is: Why on Earth is ISIS investments and not made for short term doing this? What could possibly be the point? profit growth. Adding to your list of enemies is never a D. General demand for the training provided sound strategy, yet ISIS' ferocious campaign by the company has declined. against the Shia, Kurds, Yazidis, Christians, E. None of these and Muslims who don't precisely share its views has united every ethnic and religious Direction (128-130): Study the given group in Syria and Iraq against them. ISIS is information carefully and answer the given questions. even at war with its most natural ally, al Qaeda in Syria. The Nazis and the Khmer An input-output is given in different steps. Rouge went to great lengths to hide their Some mathematical operations are done in crimes against humanity. Instead, ISIS posts each step. No mathematical operation is its many crimes on social media for global repeated in next step but it can be repeated distribution with seemingly no thoughts for with some other mathematical operation (as the consequences. multiplication can be used with subtraction in 126. Which if the above can be safely inferred from step 1 and same can be used with addition in the above passage? step 2) A. The ISIS is not motivated as much by political gains and perceptions as by the coverage its activities attract. B. The Nazis and Khymer Rouge were large scale terrorist organisations like the ISIS. C. The Nazis and Khymer Rouge had very little access to media that pervaded boundaries since they lived in a world that was much less connected as it is now. D. The ISIS is at loggerheads with its natural ally, the Al Qaeda so that it may establish natural superiority in the region of the Levant. E. The ISIS is a fundamentalist group that

believes that we all live in a post apocalyptic

world where concepts of right and wrong do As per the rules followed in the steps given not matter. above, find out in each of the following 127. Before the arrival of new trainer, the sales questions the appropriate step for the given output in AMS learning system Ltd. had been input rising by 20% per year on the average over the past ten years. However, after new training innovations by the trainer (which included computerization of training processes

21 www.gradeup.co

128. What is the result of addition of numbers Statement 1: Q is sitting second to the left of obtained in step 3? the person U. K and J are immediate A. 96 B. 97 neighbours of each other. M is at one of the C. 98 D. 108 extreme ends. E. None of these Statement 2: U is sitting opposite to the 129. What is the result of multiplication of person who is second to the left of the person individual numbers in block obtained in last K. I is sitting second to the right of the person step? J. Q and T are immediate neighbours. A. 14 B. 16 Statement 3: K faces Q. T is second to the left C. 18 D. 20 of the person S. M is second to the left of E. None of these person T. S is sitting immediate right of Q 130. What is the result of division of 2nd block and A. Statement I alone is sufficient to answer 1st block values in step 2? the question. A. 03 B. 04 B. Statement II alone is sufficient to answer C. 02 D. 06 the question. E. None of these C. All the statements I, II and III taken 131. Direction: In the given question, a statement together are sufficient to answer the is given with some assumptions as options. question. An assumption is something supposed or D. Neither of the I, II, III statements are taken for granted. You have to consider the sufficient to answer the question. statement and the following assumptions and E. Any of the two statements taken together decide which of the assumptions is implicit in are sufficient to answer the question. the statement. 133. Directions: In each of the following Statement: questions, a question is followed by The school, which used to provide free information given in three statements. You breakfast to ensure that the students receive have to study the question alone with the at least one proper meal a day, stopped the statements and decide the information given facility from January as the Charity Society in which statement is/are sufficient to answer which used to fund the breakfast withdrew the the question. support due to drop in donations. Eight boxes A, B, C, D, E, F, G and H are kept Which of the following is an assumption one above another. Which of the following box implicit in the above statement? is at the bottom position? A. The students may not receive any proper I. Two boxes are between D and E. D is above meal in a day from January E. Three boxes are between E and B. B. In future the donation to the Charity II. Box B is just above D. Two boxes are Society may further decrease. between D and E. Box F is just below box E. C. The donations received by the Charity III. Two boxes are between E and G. Box C is Society are used for social welfare. above box F. D. The families either do not have the ability A. I and II to provide proper meal to their children or B. II and III ignorant of what a proper meal is. C. Either I and III or II and III E. None of these. D. Any of two 132. Direction: Each of the following question E. All of them given below consists of a question and three 134. Read the paragraph and answer the questions statements. You have to decide whether the given below: data provided in the statements are sufficient The building owners wants the rent control by to answer the question. government to be removed. They say that the There are eight persons sitting in the two rent will increase but it will be for a short rows facing each other. In one row, person I, time. The increased rent will lead to more J, K, M are sitting and in other row, persons investment in real estate and lead to more Q, S, T, U are sitting. Who faces the person number of residential buildings. Thus, the U? increased supply will lower the rent rapidly.

22 www.gradeup.co

So, removing the rent control will lower the 136. Who faces opposite to the person who is rent. sitting second to the left of Q in smaller Which of the assumption if true will seriously square? damage the argument? A. T B. L A. The increase in rent is necessary due to C. S D. H inflation and economic crisis. E. J B. The probability that income from rent will Direction (137-138): Study the following be fully invested in more real estate and information carefully and answer the given constructions. questions. C. Government introduced rent control to prevent exploitation due to lack of proper There are equal number of males and females housing especially to people migrating to in a family. Family members are A, B, C, D, E, cities. F, G & H. There are also three married D. The demand will remain the same but the couples in the family and two of them are supply will increase. children. F is the sister-in-law of G. G is the E. None of the above daughter of B. A is the son of H and brother of G. F is the mother of C. C is the niece of G. D Direction (135-136): Study the following is the brother-in-law of A. A is the father of E. information carefully and answer the H is the grandfather of E. questions given below: 137. What is the relation of D with respect to B? A. Son of B B. Grandson of B Twelve persons A, C, F, H, I, J, K, L, Q, S, T C. Son-in-Law of B D. Brother-in-law of B and W are sitting in two squares, such that E. None of these the larger square have middle and corners seats. The smaller square has only middle 138. Who is granddaughter of H? seats. The persons sitting on larger square A. G B. C are facing towards the centre and person C. E D. F sitting on smaller square are facing outside E. None of these the centre. The smaller square is embedded in the larger one. A, who is sitting at one of 139. Direction: Each passage is followed by a set corners, is sitting second to the right of F. L of five statements. Answer according to the and S have only one member between them. directions given for each question.

The one, who is sitting opposite to T, is The average American voter does not care second to right of J. C and K are immediate about Libya. That may sound harsh, but it’s neighbors, such that one of them is sitting at generally true. The average American does one of the middle seat. I is sitting third to the however care a great deal about gas prices. left of Q, who is sitting at one of the middle So perhaps it is no surprise that as the regime seat. J is not immediate member of either of of Moammar Gadhafi falls, American news K or A. W faces K. H has same members outlets are trying to explain the news out of between him and J as between person A and Libya within the frame of prices at the pump. I. W is sitting second to the right of T, who is Heck, it was the first idea that came to my immediate left of S, who is sitting in smaller mind as I thumbed through the morning square. (Note: The person sitting on Middle news. seat means that the person is in between two corner seats). Adding insult to injury is the amount of fuel 135. Four of the following five are alike in a certain being used to conduct these fuel-based way based on the given seating arrangement military operations in the Middle East. A and thus form a group. Which one of the CNN.com article pointed out recently that, following does not belong to group? “One out of eight U.S. Army casualties in Iraq A. L B. S was the result of protecting fuel convoys. A C. I D. J post on Scaling Green contained a video of E. None of them FTI Consulting’s Adam Siegel recalling a chat with Gen. Richard Zilmer. In that

23 www.gradeup.co

conversation, the former commander of There are ten exams to be conducted from 9 troops in the Anbar province of Iraq told to 13 of a month. There are two time slots for Seigel, “I need renewable energy because every day, morning slot and evening slot. The getting fuel to my base is putting people's maximum marks of these exams are from 70 lives at risk.” to 160. The maximum marks of every subject Which of the above statements can be a just are unique. The time durations of the exams explanation for the way American news is from the range of 1hr to 5hr. The time agencies are portraying the Libyan news? duration is also unique for each subject. The subjects are Math, English, Physics, Bio, A. The general American public has several Chemistry, Economics, History, Home issues closer home related to everyday Science, Geography and Accounts. politics to worry about and does not want to The first exam is of math and of 2hrs. The spend precious time worrying about the fall of time duration of last exam is 15 minutes more an African country far away. than first exam. The exam of English is of 80 B. American news media, as is the tendency marks and on the same day of the exam of of the rest of the Western media indulges in a math. Total marks of the exams of first day misguided portrayal of Middle Eastern are 180 marks. There are two exams between monarchies and dictatorships. history and accounts. The last exam is of C. Renewable energy will not require convoys accounts and of 110 marks. There are three to transport it and hence save American lives. exams between English and the exam which is D. With the dearth of oil in the United States, of 90 marks. The exam of physics is in it is but natural that the USA should look out morning and it is on an even date. The exam for its interests and use the prospect of felling of 3hr and 5hr are on the same day. The sum a dictatorial regime for gains in oil. of timings of math and physics is 5hr. The E. American news outlets are pandering to the exam of 130 marks and 90 marks are on the general public and putting the Libyan crisis in same day. The exam of chemistry is not of 90 1/2 a context that the average American can marks. The exam of geography is of 4 hrs. understand. and on the last day. The exam of home science is on an even date and of least marks. 140. Study the following paragraph and answer the The exam of chemistry is before exam of given question. economics. The exam of 150 marks is of BSNL has launched its Customer Acquisition maximum time duration. The exam of and Customer Communication Management economics is of economics is of least time (CACMS) project that digitalizes mission duration. The sum of time duration of critical, customer-centric business process to economics and history is of 5hrs.The exam of provide customer experience across all Chemistry is of 31/2 hrs. duration. The channels throughout the customer lifecycle. maximum marks of physics is 30 marks less Which of the following will be an impact of the than that of biology. The maximum marks of CACMS launched by BSNL? 41/2 hrs. exam is 30 marks more than marks A. To centralize the business processes to of accounts. History is of maximum marks. ensure faster service, accurate customer The maximum marks of home-science are responses and customer delight. least of all. B. Customer onboarding is now being increased from hours to days 141. Which two exams are scheduled for 10th of the C. Customer onboarding is now being month? decreased from days to hours A. Physics and Biology D. Both 1 and 2 E. Both 1 and 3 B. Chemistry and History

Direction (141-145): Study the following C. Geography and Accounts information carefully and answer the following D. History and Home Science questions: E. Math and English

24 www.gradeup.co

142. Find the odd one out? 147. “G” is written against which of the following A. Math B. Physics number? C. Chemistry D. English A. Number 7 B. Number 9 E. History C. Number 10 D. Number 5 143. Which of the following is an incorrect E. Number 1 statement? 148. Direction: Read the short passage given below A. Math is of 100 marks exam and answer the question that follows it. B. Chemistry is scheduled on 11th of the Cash is the mother's milk of crime. Its appeal month to criminals is clear. Unlike cars or paintings, C. History is of 4 hrs it can be concealed immediately after being D. Accounts is of 140 marks pinched. It has no security features to prevent E. Biology is of 5 hrs its being easily and anonymously spent on 144. Which statement is correct for Economics legal or illegal goods. Unlike nearly any other exam? object that can be stolen, it needs no fence. A. Economics is of 90 marks But a new paper from the National Bureau of B. Economics exam is on the same day on the Economic Research found that electronic exam of chemistry payments led to a drop of 9.8% in the overall C. Economics exam is of 1 hr crime rate and caused the rates of burglary, D. The exam of economics is on 11th of the assault, and larceny to fall by 7.9%, 12.5%, month and 9.6% respectively. The introduction of E. All are correct electronic payments was also associated with a lower number of arrests, an indication that 145. Which is the correct combination of the Exam, the crime rates decline did not stem from time, day and Maximum marks? more aggressive policing. The paper said that th A. Math – 3hr – 9 – 130 marks the shift from cash to cards since 1990 debit- th B. Math – 2hr – 9 – 100 marks card transactions have risen 27-fold, whereas th C. Physics – 3hr – 11 – 120 marks cash volume has grown by just 4% a year th D. Geography – 4.5 hr – 12 – 140 marks may have also contributed to the decline in E. All are correct crime. Direction (146-147): Study the information 148. Which of the following conclusions can given below and answer the questions based most properly be drawn from the on it. information above?

First 10 numbers starting from 1 is written A. Criminals' need for cash motivates much from top to bottom. The word “COVER” is predatory street crime. written in alphabetical order against each B. The more payment transactions are even number from top to bottom. Two letters digitized, the less opportunity for crime are between C and B. The number of letter is C. If cash motivates crime, an absence of written between M and E is same as O and G. cash can reduce crime M is written above E. Two letters are written D. A cashless economy leads to a safer between V and S. (No letter is repeated society against any number) E. Instead of cash barter system should be 146. If there are 2 alphabets in English reintroduced. alphabetical series between alphabets written Direction (149-153): Study the information against numbers 2 and 4. Then how many given below and answer the questions based alphabets in English alphabetical series are on it. there between the alphabets written against number 2 and 1? There are 10 shelves numbered 1, 2… 10. They are arranged in two rows one above the A. Four B. Five other. The shelves 1, 2, ….5 are in row 1 and C. Six D. Seven rest in row 2, which is above row 1. The E. Eight shelves are arranged in increasing order of

25 www.gradeup.co

number given to them. Like the shelf number 152. What is the colour of the shelf which is just 1 is placed on extreme left of row 1, then below the Blue shelf? shelf number 2 and so on. Similarly, the shelf A. Violet B. Green number 6 is placed on extreme left of row 2, C. Red D. Yellow and so on. Each shelf contains a certain E. Brown number of glass slabs and photo frames. 153. What is the total length of row 2? There is at least one glass slab in each shelf. A. 258cm B. 242cm The length of each glass slab is 16 cm and C. 240cm D. 252cm that of each photo frame is 8 cm. The shelf 2 E. 262cm has length 24cm. There is one shelf between 154. Direction: Read the short passage given shelf 2 and Green shelf. The Green shelf below and answer the question that follows it.

contains 1 glass slab and 4 photo frames Grilling meat gives it great flavour. This taste, more than shelf 2. The Black shelf is just though comes at a price, since the process above the Green shelf. The Black shelf creates free radicals which damage DNA and contains same glass slab as Green Shelf and 2 thus increase the eater's chances of photo frames less than Green shelf. No shelf developing colon cancer. But recent research is between the Blue shelf and the Black shelf. has suggested that to find a way around this There is one shelf between Blue shelf and Pink problem, one should add beer when shelf. The Pink shelf is not at the extreme barbecuing meat. Free radicals which form right end. The length of Blue shelf and Black from fat and protein in the intense heat of this shelf is same but Black shelf has 1 glass slab type of cooking can be stopped by applying less than Blue shelf. The length of Pink shelf is chemicals called antioxidants which mop them 16cm less than the Black shelf. The Pink shelf up. has same photo frames as Yellow shelf. The 154. The conclusion above would be more Yellow shelf is even number shelf in row 1. reasonably drawn if which of the following Total length of shelf 7 and shelf 8 is 88cm. were inserted into the argument as an Shelf 7 is white colour which has length more additional premise? than Pink shelf. The white shelf has same A. Beer reduces the health risk of barbecuing glass slab as Blue shelf. The Brown shelf is meat just below the Orange shelf. B. Steaks should be left unmarinated There is one shelf between Brown shelf and C. Beer is rich in antioxidants Red shelf. The Orange shelf has same number D. Unmarinated steaks have more free of glass slabs and photo frames as Pink shelf. radicals than those steeped in beer The Brown shelf has same glass slab as Yellow E. Steaks can be marinated under proper shelf and same photo frames as Green shelf. climatic conditions. The Violet shelf is immediate next in number 155. Prabir Sengupta was born on 8th May 1963. to Green shelf. The Violet and Red shelf have He has seemed 65 per cent marks in the same length, and both have 2 glass slabs selection process. He has been working for each. The total length of row 1 is 248cm. the past fifteen years in an organization out of 149. Which of the following is the colour of shelf 6? which twelve years as Senior Manager-HR A. Green B. Blue after completing his post-graduation in HR C. Pink D. Orange with 55 per cent marks. He has successfully E. White implemented HR system in his organization 150. Which of the following is the length of the during the last two years. Green shelf? A. The candidate is to be selected. A. 48cm B. 72cm B. The candidate is Not to be selected C. 40cm D. 36cm C. The data provided are Not adequate to E. 56cm take a decision. 151. What is the total length of shelf 1 and shelf 2? D. The case is to be referred to Executive A. 70 cm B. 90 cm Director C. 80 cm D. 100 cm E. The case is to be referred to Chairman. E. 88 cm

26 www.gradeup.co

27

www.recruitment.guru

State Bank of India

Probationary Officer’s Examination

Held on 26.11.2006

REASONING

Directions (1-5): Study the following information to answer these question.

I . Seven persons P, Q, R, S, T, V, X belonging to different cities viz. Delhi, Mumbai, Chennai, Kolkata, Bangalore, Hyderabad and Trivandrum, not necessarily in the same order, went to USA for attending a Conference. Each one had a different specialisation viz. Literature, Physics, Economics, Marketing, Computers, Textile Engineering and Information Technology.

II. ‘S’ is from Chennai and ‘Q’ does not have specialisation in Textile or Economics. R is a man of Marketing and comes from Mumbai. The person from Kolkata has specialisation in Computers. ‘P’ who is specialised in Literature does not belong to Delhi. V having specialisation in Physics is from Trivandrum. Information Technology is the specialisation of X who comes from Bangalore.

1. Which of the following persons s from Delhi? (a) P (b) S (c) T (d) S or T (e) None of these 2. Who is a Textile Engineer? (a) S (b) T (c) V (d) S or T (e) None of these 3. Person with specialisation in Literature comes from which of the following cities? (a) Bangalore (b) Kolkata (c) Delhi (d) Hyderabad (e) None of these 4. Which of the following combinations of person, city and specialisation is correct? (a) S-Chennai-Economics (b) S-Chennai-Textile (c) Q-Kolkata-Computers (d) T-Delhi-Economics (e) None of these 5. Who is specialised in Computers ? (a) Q (b) S (c) T (d) Data inadequate (e) None of these

The information provided here is only for refrence. It may vary from the original. www.recruitment.guru

Directions (6-10): Study the following information carefully and answer the questions given below:

XYZ Limited Company organised an exhibition of machine tools. The exhibition was opened on all the weekdays for public. Certain passcode were issued to the visitors as entry card. The passcode of entry card was changed every hour according to a certain rule as shown below. The entry time of the first batch of the visitors was 9 AM and that for the last batch was 7 PM. Each batch was allowed only one hour. The lunch time was from 1 PM to 2 PM.

Batch I (9 AM to 10AM)

Passcode: course easy set for each year was

Batch III (10 AM to 11 AM)

Passcode: easy each course for was set year

Batch III (11 AM to 12 Noon)

Passcode: each was easy for year course set and so on.

6. If the passcode for the batch entering at 12 Noon is “she the girl is clever very good”, then what will be the passcode for the batch entering at 3 PM? (a) Clever good is the very she girl (b) Clever good the is she very girl (c) Clever good the very is she girl (d) Clever good very is the she girl (e) None of these 7. The passcode of which of the following batches will be similar to the passcode for the batch III? (a) VI (b) VII (c) VIII (d) IX (e) None of these 8. If the passcode for the batch III is “pin to the point is sharp not”, then what will be the passcode for the batch V ? (a) Is not to sharp point pin the (b) Is not to point sharp pin the (c) Not is to sharp point pin the (d) Not is to point sharp pin the (e) None of these. 9. If the passcode for the second batch is “for the life is good change got” then the passcode for which of the following batches is “got change good is life the for” ? (a) IV (b) III (c) V (d) VI (e) None of these

The information provided here is only for refrence. It may vary from the original. www.recruitment.guru

10. If the passcode for batch IV is “do how will the you job now”, then what will be the passcode for batch II ? (a) Job will now the do you how (b) Job now will the do you how (c) Job will how the do you now (d) Job will the now do you how (e) None of these

Directions (11-15): in each question below is given a statements followed by two assumptions numbered I and II. An assumption is something supposed or taken for granted. You have to consider the statement and the following assumptions and decide which of the assumptions is implicit in the statement.

(a) If only assumption I is implicit (b) If only assumption II is implicit (c) If either I or II is implicit (d) If neither I nor II is implicit and (e) If both I and II are implicit. 11. Statement: ‘Country ‘A’ would explore all channels to diffuse current tensions with country ‘B’ and bring peace on its borders’ – Statement of spokesperson of country ‘A’. Assumptions: I. Country ‘A’ is desirous to diffuse current tension and restore peace with country ‘B’. II. It is desirable to use more than one channel when complex issues are to be settled amicably. 12. Statement: Two months ago, it was announced that central government pensioners would get dearness relief with immediate effect but till date, bank have not credited the arrears’-A statement from a Pensioner’s Forum. Assumptions: I . Most of the banks normally take care of the pensioners. II . Two months’ time is sufficient for the government machinery to move and give effect to pensioners 13. Statements: The bridge was built at the cost of Rs. 128 crores and even civil bus service is not utilising it, what a pity to see it grocely underutilised’ A citizen’s view on a new flyover linking east and west sides of a suburb. Assumptions: I. The building of such bridges does not serve any public objective. II. There has to be some accountability and utility of money spent on public projects. 14. Statement: Use our product to improve memory of your child, it is based on natural herbs and has no harmful side effects’-Advertisement of Pharmaceutical Company. Assumptions: I. People generally opt for a medical product which is useful and has no harmful side effects. II. Improving memory of child is considered as important by many parents. 15. Statements: The traders of State ‘K’ would observe a state-wide bandh as the state has failed to meet their demand to resolve sales tax and other issues. Assumptions: I. The traders of State ‘K’ have earlier tried other usual procedures to get their problem solved.

The information provided here is only for refrence. It may vary from the original. www.recruitment.guru

II. The State ‘K’ is not keen to solve the problem of traders.

Directions (16-20) : In the following questions the symbols +, x, ?, @ and $ are used with the following meaning”

A + B means A is neither equal to nor smaller than B.

A x B means A is neither equal to nor smaller than B.

A ? B means A is neither greater nor equal than B.

A @ B means A is neither greater nor equal to B.

A $ B means A is equal to B.

Now in each of the following questions assuming the given statements to be true find out which of the conclusions I and II given below them is/ are definitely True?

(a) If only conclusion I is true (b) If only conclusion II is true. (c) If either I or II is true (d) If neither I nor II is true (e) If both I and II are true. 16. Statements : C + D, D $ M, M ? C Conclusion: I. M $ C II. D + M 17. Statements : K $ M, M x B, K + B Conclusions: I. M x K II. K ? M 18. Statements : F @ G, G ? P, P x H Conclusions: I. F @ H II. H x G 19. Statements : M @ T, T x R, R $ Q Conclusions : I. M ? R II. M ? Q 20. Statements : D x G, G @ K , K + M Conclusions: I. M + G II. D x K

The information provided here is only for refrence. It may vary from the original. www.recruitment.guru

Directions (21-25) : Each of the followings below consists of a questions and two statements numbered I and II given below it. You have to decide whether the data provided in the statements are sufficient to answer the questions. Read both the statements and

(a) If the data in statement I alone are sufficient to answer the question, while the data in statements II alone are not sufficient to answer the question. (b) If the data in statements II alone are sufficient to answer the question while the data in statement I alone are not sufficient to answer the question. (c) If the data either in statement I alone or in statement II alone are sufficient to answer the question. (d) If the data even in both statements I and II together are not sufficient to answer the question. (e) If the data in both statements I and II together are necessary to answer the question. 21. Which village is to the North-East of village ‘A’? I. Village ‘B’ is to the North of village A’ villages C and D are to the East and West of village B, respectively. II. Village ‘P’ is to the South of village ‘A’ and village ‘E’ is to the East of village ‘P’ village ‘K’ is to the North of village ‘P’. 22. Cab Rohan retire from office ‘X’ in January 2006, with full pension benefits? I. Rohan will complete 30 year of service in office ‘X’ in April 2006 and desires to retire. II. As per office ‘X’ rules an employee has to complete minimum 30 years of service and attain age of 60. Rohan has 3 years to complete age of 60. 23. Among five friends P, Q, R, S and T, who ranks third in terms of salary obtained by them? I . T’s salary is more than P and Q but not more than S. II. R’ salary is lowest among them. 24. How is P related to Q? I . J has two daughters, one of them ‘R’ is marred to ‘P’. II. Q is the mother of ‘S’, the younger sister of ‘R’. 25. Which word in the code language means ‘flower’? I . ‘de fu ia pane’ means ‘rose flower is beautiful’ and ‘la quiz’ means ‘beautiful tree’. II. ‘de la chin’ means ‘red rose flower’ and ‘pa chin’ means ‘red tea’.

Directions (26-30) : Below is given a passage followed by several passible inferences which can be drawn from the facts stated in the passage. You have to examine each inference separately in the context of the passage and decide upon its degree of truth or falsity.

(a) If the inference is “definitely true”, i.e. it properly follows from the statement of facts given. (b) If the inference is “probably true” though not “definitely true” in the light of the facts given. (c) If the “data are inadequate” i.e., from the facts given you cannot say whether the inference is likely to be true of false. (d) If the inference is “probably false” though not “ definitely false” in the light of the facts given”. (e) If the inference is “definitely false”, i.e., it cannot possibly be drawn from the facts given or it contradicts the given facts.

In India the asbestos industry is growing and employs more than 15,000 people in 75 units which are spread over several states like Gujarat. Madhya Pradesh, Maharashtra, Andhra

The information provided here is only for refrence. It may vary from the original. www.recruitment.guru

Pradesh etc. Surprisingly advanced countries are banning cancer causing asbestos products, multinational companies are from those countries which are setting up units in developing countries like India. One reason being lack of awareness in the society and indifference of the government machinery of these countries. Prolonged exposure to asbestos dust and fibres can cause lung cancer but most workers in India are too afraid to protect for fear of losing jobs. Some of these factories are operating in Mumbai. Quite a few of the factories in India are not known to take adequate precautions to protect workers from asbestos dust.

The Government is taking several steps to provide medical inspection of workers. In fact it has amended factories act to extend the provision to even those factories employing less than 10 workers.

26. The asbestos industry is one of the largest industries in India. 27. The asbestos industries in India are located in few metropolitan pockets only. 28. The advanced countries are concerned and careful to protect health hazards of its people. 29. The demand for asbestos products appears to be growing in India 30. The Governments of developing countries appear to be not taking appropriate measures while granting permission to set up production units of multinational companies in their countries.

Directions (31-35) : Given below are pairs of events A and B. You have to read both the events A and B and decide their nature of relationship. You have to assume that the information given in A and B are true and you will not assume anything beyond the given information in deciding the answer.

(a) If ‘A’ is the effect and ‘B’ is its immediate and principal cause (b) If ‘A’ I the immediate and principal and ‘B’ is its effect. (c) If ‘A’ is an effect but ‘B’ is not its immediate and principal cause. (d) If ‘B’ is an effect but ‘A’ is not its immediate and principal cause. (e) None of these 31. Event (A) : We can get anything with money. Event (B) : Today money is the most important. 32. Event (A) : Due to mechanisation the life of human beings is becoming more comfortable in unban areas. Event (B): Life is becoming insecure in urban areas. 33. Event (A) : The Government has decided recently to provide additional dearness allowance to its employees. Event (B) : Consumer Price Index is increasing for the last five months. 34. Event (A) : The children of younger generation do better in their study. Event (B) : The parents of children now realise the importance of education very well. 35. Event (A) : There is considerable increase in the number of people having computers. Event (B) : Computer education is being made compulsory in schools.

Directions (36-40): Read the following in formation and answer the questions based on it.

(a) The length breadth and height of a rectangular piece of wood are 4 cm, 3 cm, and 5 cm. respectively.

The information provided here is only for refrence. It may vary from the original. www.recruitment.guru

(b) Opposite sides of 5 cm, x 4 cm. piece are coloured in red colour. (c) Opposite sides of 4 cm x 3 cm are coloured in blue (d) Rest 5 cm x 3 cm. are coloured in green in both sides. (e) Now the piece is cut in such way that a cuboid of 1 cm. x 1 cm. will be made. 36. How many cuboids shall have all the three colours ? (a) 8 (b) 10 (c) 12 (d) 14 (e) None of these 37. How many cuboids shall have not any colour? (a) No any (b) 2 (c) 4 (d) 6 (e) None of these 38. How many cuboids shall have only two colours red and green in their two sides? (a) 8 (b) 12 (c) 16 (d) 20 (e) None of these 39. Anita was doing a survey in different companies by working on all days of the week. She started with company ‘A’ and on completing her work she went to company ‘B’ on the third day. She was required to work there for three days and then went to company ‘C’ on the fourth day. She worked in company ‘C’ on the fourth day. Thereafter she worked in company ‘D’ and completed her work on fourth day after working for three days which was a Saturday. On which day did she start her work in company A? (a) Thursday (b) Saturday (c) Sunday (d) Monday (e) None of these 40. How many pairs of letters are there in the word ATMOSPHERE which have as many letters between them as in the English alphabet ? (a) Two (b) Three (c) Four (d) Five (e) None of these

Directions (41-45) : In each question below are given some statements followed by four conclusions numbered I, II, III and IV. You have to take the given statements to be true even if they seem at variance from commonly know facts. Read all the conclusions and then decide which of the given conclusions logically follows from the two given statements disregarding commonly known facts.

41. Statements :

The information provided here is only for refrence. It may vary from the original. www.recruitment.guru

Some ice are rings. No ring is paint. Some rings are gold. Conclusions : (I) No any gold is paint (II) No any ice is gold (III) Some rings are paints (a) Only I and III follows (b) Only I and II follow (c) Only III and IV follow (d) Only II and III follow (e) None of these 42. Statements : All gates are flowers. Some gates are fruits. Some flowers are clips. Conclusions : (I) Some flowers are fruits. (II) Some clips are fruits. (III) Some clips are gates. (IV) No any flower is fruit. (a) Only I follows (b) Only I and IV follows (c) Only II and IV follows (d) Only I and III follows (e) None of these 43. Statements : No candle is bell. Some shoes are bells. All tables are shoes. Conclusions: (I) Some tables are bells (II) No table is bell. (III) Some shoe are candles (IV) No shoes are candle. (a) Only I and IV follow (b) Only I and II follow (c) Only III and IV follow (d) Only I and III follow (e) None of these 44. Statements : Some films are clouds. All rate are clods. Some clouds are chairs. Conclusions : (I) No film is chair. (II) Some rats are films (III) Some clouds are rats.

The information provided here is only for refrence. It may vary from the original. www.recruitment.guru

(IV) Some chairs are rats. (a) Only I and III follow (b) Either II or IV follow (c) No any conclusion follows (d) Only IV follows (e) None of these 45. Statements : Some lice are slates All slates are apples. No apple is car Conclusions : (I) Some cars are slates. (II) Some lice are cars. (III) Some apples are lice (IV) No car is lice. (a) No any conclusion (b) Only II follows (c) Only III follows (d) Either II or IV and III follow (e) None of these

Directions (46-50): Read the following information to answer the questions Symbolise the given number and symbol.

Number : 1 2 3 4 5 6 7 8 9

Symbol : x * ? $ + ! D

(i) If any number by odd number then odd number then the odd number symbolised by @. (ii) If any number ends by even number then the even number symbolised as ©. 46. Which of the following will be the symbol of 673258? (a) + ? * $ ! (b) @ + ? * $! (c) © ? + $ * ! (d) + ? * $ © (e) None of these 47. Which of the following will be the symbol of 236475 ? (a) ? * x @ (b) © ? +@ (c) x ? +@ (d) © + $ (e) None of these 48. Which of the following will be the symbol of 846721 (a) © + * x (b) ! + * @ (c) ! + * x (d) © + * x (e) None of these 49. Which of the following will be the symbol of 178524 ?

The information provided here is only for refrence. It may vary from the original. www.recruitment.guru

(a) © + ! $ * © (b) @ x + ! $ © (c) © + ! $ * (d) * + ! $ © (e) None of these 50. Which of the following will be the symbol of 25486 ? (a) * $ ! D (b) * $ ! © (c) © $ ! (d) * $ ! (e) None of these 51. If 3 is subtracted from the middle digit of the following numbers and then the position of the digits are reversed, which of the following will be the last digit of the middle number after they are arranged descending order ? 589 362 554 371 442 (a) 5 (b) 4 (c) 2 (d) 1 (e) 3 52. In a certain code language GERMINATION is written as IMGRENNOAIT. How is ESTABLISHED written in that code ? (a) BATESLDEIHS (b) BAETSLDEIHS (c) BAETSLEDIHS (d) BEATSLDEIHS (e) None of these 53. In a certain code language POULTRY is written as PRQXNVTY. How is TREASON written I that code ? (a) TVSGCUQN (b) TVTGCUQN (c) TTVGCUQN (d) TVTHCUQN (e) None of these 54. If in the English alphabet, all the letters at odd numbered positions are written in serial order from left to right followed by the letters at even numbered positions written in reverse order, which letter will be sixth to the left of seventeenth letter from left ? (a) D (b) B (c) V (d) U (e) None of these 55. If it is possible to make a meaningful word from the second, fourth, fifth, eleventh and thirteenth letters of the word ESTABLISHMENT using each letter only once, write second letter of that word as your answer, if more than one such word can be formed, write M as your answer and if no such word can be formed, write ‘X’ as your answer. (a) B (b) A

The information provided here is only for refrence. It may vary from the original. www.recruitment.guru

(c) E (d) M (e) X

Directions (56-60) : Study the following information carefully and answer the questions given below: An institute XYZ provides scholarship to its employees for higher studies in the United States of America. Following are conditions for awarding scholarship to the employees . The candidate must: (i) Not be more than 30 year of age as on April 1, 2006 (ii) Have secured more than 70 per cent marks in Post-graduation and 75 per cent marks in Graduation (iii) Have at least two years work experience in the XYZ institute. (iv) Be ready to sign a bond of two years with the company, (v) Have got A or A + rating for his/her works in the last two years. However, in case of a candidate who fulfils all other criteria EXCEPT (a) (ii) above but has secured minimum 60 per cent marks in Graduation and post Graduation and he/she has got Ph.D. may be referred to the Director. (b) (v) above but has work experience of three years with ratings B + A or A + may be referred to the Chairman (c) (iv) above but he/she has to leave the institute and he is ready to pay Rs. 50,000, may be referred to the President. Based on these criteria and information provided below, decide the course of action in each case. You are not to assume anything. The cases are given to you as on April 1, 2006. If the data provided are not adequate to decide the given course of action your answer will be “data inadequate”. 56. Deepali Mirza working in XYZ institute for the last two and half years. Her performance is good and has secured ratings A +. She has secured 70 per cent and 75 per cent marks in Graduation and Post-Graduation respectively. She is Post Graduate from Mathematics. She is ready to sign the bond of two years. Her date of birth is 31 .03 .1977. she has also done Ph.D. (a) Data inadequate (b) Refer to the Director (c) Refer to the President (d) Refer to the Chairman (e) Not to be selected 57. Ajit D’ souza is a Post-Graduate in Physics with 75 per cent marks and passed Graduation with 77 per cent marks. He is working in the institute since 2002 and has always got A ratings on performance. He was 25 years old at the time of joining the institute. He is keen to seek job in the United States of America. (a) Refer to the Chairman (b) Refer to the President (c) Data inadequate (d) Not to be selected (e) To be selected. 58. Rajendra Bhave is a post-graduate in Psychology. He had secured 72% marks at post graduation and 78 % marks at graduation. He is working in XYZ institute for more than two years and his performance ratings are A + and A respectively in the last two years.

The information provided here is only for refrence. It may vary from the original. www.recruitment.guru

He is not keen to sing the bond but does not mind paying Rs. 50,000/- if required to leave . his date of birth is 14.8.1976. (a) To be selected (b) Refer to the Chairman (c) Refer to the Director (d) Refer to the President (e) Not to be selected 59. Harish Chheda ha a bright academic career throughout. He has done post-graduation with 65% marks and has submitted Ph. D. thesis. His date of birth is 27.9.1979. He is working for last three years in the XYZ institute and has always got A ratings on performance. He is ready to pay Rs. 50,000/- in case if required to leave. (a) To be selected (b) Refer to the President (c) Refer to the Director (d) Data inadequate (e) Not to be selected 60. 28 years old Tarique Anwar is a post-graduate in Chemistry and has registered for Ph. D. He scored 80% and 78 % marks at graduation and post-graduation respectively. He is working in the XYZ institute since 2001. His performance ratings in the last three years are B + , A+ and A respectively. He is ready to sing two years’ bond with the institute (a) To be selected (b) Refer to the President (c) Refer to the Chairman (d) Refer to the Director (e) No to be selected Direction (61-65) : In each of the questions given below which of the five answer figures should come after the problem figures if the sequence were continued ?

61.

62.

The information provided here is only for refrence. It may vary from the original. www.recruitment.guru

63.

64.

65.

The information provided here is only for refrence. It may vary from the original. www.recruitment.guru

Directions (66-70) : In each of the following questions a related of figures is followed by five numbered pars of figures. Select the pair of figure which shows similar relationship as that given in the original pair. You are required to select the best answer from among fairly close alternatives.

66.

67.

68.

s 69.

70.

The information provided here is only for refrence. It may vary from the original. www.recruitment.guru

Directions (71-75) : In each of the following questions is four out of the five figures, element I is related to element II in the same particular way. Find out the figure in which the element I is so related to element II. 71.

72.

73.

74.

75.

ENGLISH LANGUAGE Directions (76-90) : Read the following passage and answer the questions given below it . Certain words/phrases are given in bold lo help you locate them while answering some of the questions.

The information provided here is only for refrence. It may vary from the original. www.recruitment.guru

In a disarmingly frank talk at the Indian Merchants Chamber in Mumbai the Japanese Ambassador in India dwelt at length with issues that exercise the mind of Japanese investors when they consider investment proposals in India. Raising the question “ What comparative advantages does India offer as an investment market?”, he said though labour in India is inexpensive, wage levels are offset by productivity level to a large extent. Acknowledging that vastness of the Indian market is great inducement for investment in manufacturing industry he wondered if it was justifiable to provide that overseas remittance of profit in foreign exchange to be fully covered by exchange earnings as had been done. Significantly, on the eve of the Prime Minister’s visit to Japan, the Government delinked profits repatriation from exports, meeting this demand. The Ambassador said foreign investors needed to be assured of the continuity and consistency of the liberalization policy and the fact that new measures had been put into force by means of administrative notifications without amending Government laws acted as a damper. The Ambassador pleaded for speedy formulation of the exit policy and pointed to the highly restrictive control by the Government on disinvestment by foreign partners in joint ventures in India. While it is all too easy to dismiss critical comment on conditions in India Contemptuously, there can be little doubt that if foreign investment is to be wooed assiduously, we will have to meet exacting international standard and cater at least partially to what we may consider the idiosyncrasies of our foreign collaborators. The Japanese too have passed through a divided as substandard and shoddy. That they have come out of the ordeal of fire to emerge as an economic super power speaks as much of their doggedness to pursue goals against all odds as of their ability to improvise and adapt to internationally acceptable standards. There is no gain-saying that the past record of Japanese investment is a poor benchmark for future expectations.

76. The author has appreciated the Japanese for their (a) Quality of products manufactured in the fifties (b) Passing through an ordeal (c) Perseverance for raising quality of products (d) Future expectations (e) None of these 77. According to the Japanese Ambassador, which of the following motivates the foreign investors to invest in India manufacturing industry? (a) Very large scope of Indian market (b) Overseas remittance of profit in foreign exchange (c) Assurance of continuity of the liberalisation policy (d) High productivity levels (e) None of these 78. The purpose of the author in writing this passage seems to be to (a) Discourage foreign investment in India (b) Critically examine Indian investment environment (c) Paint a rosy picture of India’s trade and commerce (d) Criticise Government’s liberalisation policy (e) Raise the expectations of foreign investors 79. Which of the following suggestions were expected by the Japanese Ambassador? (A) Speedy formulation of the exist policy.

The information provided here is only for refrence. It may vary from the original. www.recruitment.guru

(B) Imposing restrictions of disinvestment by foreign partners in joint ventures in India (C) Continuity and consistency of the liberalisation policy (a) All the three (b) (A) & (B) only (c) (B) & (C) only (d) (A) & (C) only (e) None of these 80. According to the Japanese Ambassador, India offers a comparative advantage of foreign investors in terms of – (a) Inexpensive labour (b) Abysmally low wage levels (c) Higher productivity (d) Skilled workforce (e) None of these 81. For seeking more and more foreign investment the author suggest that we should – (a) Satisfy fully the whims of our foreign collaborators (b) Dismiss all critical comments of Indian conditions (c) Link profit repatriations to exports (d) Raise the quality of product of match international standards (e) None of these 82. From the passage it can be inferred that the author is- (a) A political commentator (b) A secretary of the Japanese Ambassador (c) A Japanese investor (d) An Indian investor (e) None of these 83. The author attributes Japan’s emergence as an economic super power to (A) Their ability to overcome any ordeal. (B) Their tenacity and perseverance despite unfavourable circumstances. (C) Their ability to improvise and adapt to globally acceptable quality levels (a) (A) & (B) only (b) (B) & (C) only (c) (A) & (C) only (d) ALL of three (e) None of these 84. Which of the following statements (s) is/are true about the critical comments on investment conditions in India. (A) These comments are difficult to be countered. (B) These comments are received from various international quarters. (C) These comments are based more on based than on facts. (a) Only (C) (b) Only (B) (c) Only (A) (d) (A) & (B) only (e) (A) & (C) only Directions (82-86) : Choose the word which is most nearly the SAME in meaning as the word printed in bold as used in the passage. 85. Assiduously

The information provided here is only for refrence. It may vary from the original. www.recruitment.guru

(a) Persistently (b) Hastify (c) Feebly (d) Deliberately (e) Innocently 86. Idiosyncrasies (a) Demands (b) Needs (c) Deviations (d) Ideologies (e) Identify 87. Shoddy (a) Extraordinary (b) Shabby (c) Cheap (d) Disadvantageous (e) Unprofitable Directions (88-90) : Choose the word which is most OPPOSITE in meaning of the word printed in bold as used in the passage. 88. Inducement (a) Incentive (b) Motive (c) Breach (d) Temptation (e) Impediment 89. Justifiable (a) Unreasonable (b) Formidable (c) Irrevocable (d) Unscrupulous (e) Inevitable 90. Contemptuously (a) Amicably (b) Reasonably (c) Respectfully (d) Methodically (e) Indecisively Directions (91-95) : Rearrange the following six sentences (A), (B), (C), (D), (E), and (F) in the proper sequence to form a meaningful paragraph; than answer the question given below them. (A) The application of economic. Environmental and consumer pressures have been on an increase in recent years. (B) As a result, our agro-food production and technology are amongst the most advanced in the world. (C) They are thus able to provide expertise and technology to satisfy the needs of agro-food production. (D) In turn, the support industries have developed to an equally advanced state. (E) They have also equipped themselves with the necessary expertise to satisfy the most exacting requirements of the overseas markets.

The information provided here is only for refrence. It may vary from the original. www.recruitment.guru

(F) These have greatly influenced the development of the agriculture and food industries in our country. 91. Which of the following should be the SECOND sentence after rearrangement ? (a) F (b) E (c) D (d) C (e) B 92. Which of the following should be the THIRD sentence after rearrangement ? (a) F (b) E (c) D (d) C (e) B 93. Which of the following should be the FOURTH sentence after rearrangement ? (a) F (b) E (c) D (d) C (e) B 94. Which of the following should be the FIFTH sentence after rearrangement ? (a) F (b) E (c) D (d) C (e) B 95. Which of the following should be the SIXTH (LAST) sentence after rearrangement ? (a) F (b) E (c) D (d) C (e) B Directions (96-100) : Which of the phrases (a), (b), (c) and (d) given below should replace the phrase given in bold in the following sentence to make the sentence grammatically correct ? if the sentence is correct as it is and no correction is required, mark (e) as the answer. 96. They didn’t pay any heed to their superior’s instructions. I did neither. (a) I either did (b) Either I did not (c) Neither didn’t I (d) Nor did I (e) No correction required 97. Every poet gives vice to his anger and his compassion through his poems. (a) Voiced to give (b) Gives voice for (c) Gave voice against (d) Voiced at given (e) No correction required

The information provided here is only for refrence. It may vary from the original. www.recruitment.guru

98. The police nabbed a notorious criminal who had been terrorising builders and extorted money from them for the past two years. (a) Was terrorising builders and extorted (b) Had not been terrorised builder and extorted (c) Had been terrorising builders and extorting (d) Had terrorised builders and extorted (e) No correction required 99. Managers frequently encounter situations where they need to help other solved problems. (a) To helping others solved (b) Help others solved (c) To help others solve (d) Help others solving (e) No correction required 100. From among various alternatives we should choose the one which is viable and consumes less time and energy. (a) Is viable and consuming lesser (b) Is viability and consumes less (c) Being viable and consumed less (d) Has viable and consuming less (e) No correction required 101. We are looking forward for good rains this year. (a) Are looking forward to (b) Have looked forward for (c) Have been looking forward at (d) Should look forward with (e) No correction required 102. There is full justification to close down the units which are neither profitable nor serving any social cause. (a) Which are not profitable or serve (b) Which have neither profitable nor serving (c) Which have neither profitable nor serve (d) Which are neither profitable nor serve (e) No correction required 103. Please take delivery of the material and acknowledgement at the earliest. (a) Send acknowledging at the earliest. (b) Acknowledge at the earliest (c) Acknowledge the earliest (d) Early acknowledgement (e) No correction required 104. The Officer had triggered a controversy by charging his superiors by ignoring his warning. (a) By charging his superiors with ignoring (b) With charging his superiors by ignoring (c) By charging his superiors after ignoring (d) For charging his superiors on ignoring (e) No correction required.

The information provided here is only for refrence. It may vary from the original. www.recruitment.guru

105. Had we been alone we would have contended ourselves with any plain food that give us strength (a) That gives us strength (b) That gave us strength (c) Which give us strength (d) Which give strength to us (e) No correction required Directions (106-110) :In each of the following question an idiomatic expression and its four possible meanings are given. Find out the correct meaning of the idiomatic expression and mark the number of that meaning as you answer 106. To eat humble pie (a) To go to ruins (b) To be earnest (c) To spread rapidly (d) To refuse after consenting (e) To apologize 107. A wild goose chase (a) An absurdly hopeless enterprise (b) A fuss over a trifling matter (c) To be insensitive to criticism (d) To speak boastfully of oneself (e) To protect oneself from wild animals 108. To be hard up (a) To look depressed (b) To keep starvation away (c) To act excitedly (d) To be short of money (e) To behave like a fool 109. To cry wolf (a) To come to what is most important (b) To give false alarm (c) To turn pale (d) To be astonished (e) To run away 110. To take to one’s heels (a) To act against one’s own interest (b) To assault (c) To run away (d) To have concern (e) To run oneself Directions (111-115) : In each of the following sentences there are two blank spaces. Below each sentence there are five pair of words denoted by numbers a, b, c, d and e, . Find out which pair of words can be filled up in the blanks in the sentence in the same sequence to make the sentence meaningfully complete. 111. He objected to the proposal because it was founded on a ____ principle and also was ___ at time. (a) Faulty ___ desirable (b) Imperative ___ reasonable

The information provided here is only for refrence. It may vary from the original. www.recruitment.guru

(c) Wrong ____ inconvenient (d) Sound ___ acceptable (e) Uncomforting ___ deplorable 112. The criterion for ___ a player should be his recent performance; but unfortunately, the journalists are ___ to be carried away by earlier successes. (a) Condemning ___ satisfying (b) Revealing ___ reluctant (c) Judging ___ prone (d) Eager ____ acclaiming (e) Criticising ____clean 113. For the last half century he ____ himself to public affairs ___ taking a holiday. (a) By____ committed (b) After ____ offered (c) Devoted ____ without (d) Sacrificed____after (e) Prepared ____ before 114. You will see signs of everywhere, which speak well for the ___ of these people. (a) Decoration-senses (b) Clear-debris (c) Beauty-careful (d) Industry -prosperity (e) Repairs-extravaganza 115. The police arrested Ramesh on a ___ of theft but for lack of evidence___ him. (a) Crime __ imprisoned (b) Punished __ complaint (c) Left __ condition (d) Tip __ absconding (e) Charge ___ released Directions (116-125) : In the following passage, there are blanks, each of which has been numbered. These numbers are printed below the passage and against each, five words are suggested, one of which fits the blank appropriately. Find out the appropriate word in each case. (116) stringent antipollution laws, mass awareness levels in India about the need to (117) the environment are low. Which is (118)many people insist that mere laws won’t do; what we (119) need are “environment conscious” citizens. It is in this context that the University’s (120) to introduce environment studies as a compulsory paper at the undergraduate level (121) significance. There was some (122) initially about who would teach the paper because financial (123) make it impossible for colleges to (124) approval for new teaching posts. In fact, in August 1999, the University Grants Commission (UGC) imposed a ban on the creation of new teaching posts in colleges. (125) with this problem, authorities at the university have decided that serving teachers belonging to various disciplines will teach the paper.

116. (a). Despite (b) Having (c) Enacting (d) Adopting (e) Although 117. (a) contaminate

The information provided here is only for refrence. It may vary from the original. www.recruitment.guru

(b) Clean (c) Filter (d) Protect (e) Pollute 118. (a) resulting (b) Why (c) Obvious (d) As (e) Because 119. (a) seldom (b) Don’t (c) Hardly (d) Perfectly (e) Actually 120. (a) inability (b) Deferral (c) Decision (d) Failure (e) Reluctance 121. (a) extracts (b) Trivalency (c) Expects (d) Loses (e) Assumes 122. (a) displeasure (b) Antagonism (c) Hurdles (d) Confusion (e) Priority 123. (a) losses (b) Constraints (c) Apathy (d) Soundness (e) Independence 124. (a) receive (b) Establish (c) Emphasize (d) Expect (e) Sanction 125. (a) Down (b) Familiarity (c) Faced (d) Convinced (e) Solution

QUANTITATIVE APTITUDE Directions (126-130) : Each of the questions below consists of a question and three statements denoted A, B and C are given below it. You have to study the question and all the

The information provided here is only for refrence. It may vary from the original. www.recruitment.guru

three statements and decide whether the question can be answered with anyone or two of the statements or all the statements are required to answer the question. 126. How much marks did Arun secure in English? A. The average mark obtained by Arun in four subjects including English is 60. B. The total mark obtained by him in English and Mathematics together is 170. C. The total mark obtained by him in Mathematics and Science together is 180. (a) All three A, B and C together are necessary. (b) Only A and B together are necessary (c) Only B and C together are necessary (d) Only A and C together are necessary (e) None of these 127. How much profit did Mahesh earn on the cost price of an article by selling it ? A. He get 15% discount on the marked price at the time of purchase B. He sold if for Rs. 3060. C. He earned 2% profit on the marked price. (a) Only A and B both together are necessary (b) Only B and C both together are necessary (c) Only A or C and B together are necessary (d) Even A, B and C all together are not sufficient to answer the question. (e) All three A, B and C together are necessary 128. What will be sum of two numbers? A. Among the two numbers, the bigger number is greater than the smaller number by 6. B. 40% of the smaller number is equal to 30% of the bigger number.

C. The ratio between half of the bigger number and rd of the smaller number is 2 : 1.

(a) Only B and C together are necessary (b) Only A and B together are necessary (c) Out of A, B and C any two together are necessary (d) All three A, B and C together are necessary (e) None of these 129. What is the area of a right angled triangle? A. The perimeter of the triangle is 30 cm. B. The ratio between the base and the height of the triangle is 5 : 12 C. The area of the triangle is equal to the area of a rectangle of length 10 cms. (a) Only B and C together are required (b) Only A and B together are required (c) Only either A or B and C together are required (d) Only A and C together are required (e) None of these 130. What is R’s share of profit in a joint venture ? A. Q started business investing Rs. 80,000 B. R joined him after 3 months. C. P joined after 4 months od Rs. 1,20,000 and got 6,000 as his share of profit. (a) Only A and C are required (b) Only B and C are required (c) All A, B and C together are required (d) Even with all A, B and C the answer cannot be arrived at (e) None of these

The information provided here is only for refrence. It may vary from the original. www.recruitment.guru

Directions (131-135) : Study the following table carefully and answer the questions given below it.

The number of candidates appeared, passed and selected in a competitive examination from five states over the years.

A B C D E State A P S A P S A P S A P S A P S Years

2000 850 215 25 1050 245 35 990 195 28 1080 300 36 1120 240 40

2001 880 240 20 980 230 30 650 150 28 1150 320 38 960 180 26

2002 750 180 22 1120 210 28 840 180 25 995 280 42 885 177 32

2003 920 290 36 890 190 32 780 160 32 975 260 39 1040 220 30

2004 960 300 32 950 225 40 1020 220 36 888 240 32 980 280 34

2005 820 250 28 1180 200 38 930 215 35 864 216 30 900 228 24

A = Appeared, P= Passed, S = Selected

131. In the year 2000, which state had the lowest percentage of candidates selected over the candidates appeared ? (a) A (b) B (c) C (d) D (e) E 132. During which of the following years the passing percentage over appeared is the highest from state ‘D’? (a) 2005 (b) 2004 (c) 2003 (d) 2002 (e) None of these 133. Total number of candidates selected from state ‘A; is approximately what percentage of the total number of candidates selected from state ‘B’ ? (a) 72 (b) 88 (c) 85 (d) 75 (e) 80 134. During which of the following years the percentage of candidates selected over passed is the lowest for state ‘B’ ? (a) 2000

The information provided here is only for refrence. It may vary from the original. www.recruitment.guru

(b) 2001 (c) 2003 (d) 2004 (e) None of these 135. What is the ratio between number of candidates passed from state A in 2001 to that from state E in 2004 ? (a) 6 : 7 (b) 14 : 15 (c) 13 : 16 (d) 12 : 16 (e) None of these

Directions (136-140) : Study the following graph carefully to answer the question given below it. Production of paper by 3 different companies A, B and C over the years. 70 Quan 60 tity of 50 pape r in 40 A lakh B 30 tonn C es 20

10

0 1999 2000 2001 2002 2003 2004

136. What is the difference between the production of company C in 1991 and the production of company A in 2004 ? (a) 50,000 tonnes (b) 5,00,00000 tonnes (c) 50,00,000 tonnes (d) 5,00,000 tonnes (e) None of these 137. What is the percentage increase in production of company ‘A’ from 2000 to 2001 ? (a) 37.5 (b) 38.25 (c) 35 (d) 36 (e) None of these 138. For which of the following years the percentage of rise/fall in production from the previous year is the maximum for company B (a) 2000 (b) 2001 (c) 2002 (d) 2003 (e) 2004

The information provided here is only for refrence. It may vary from the original. www.recruitment.guru

139. The total production of company C in 2001 and 2004 is what percentage of the total production of company A in 1999 and 2000? (a) 95 (b) 90 (c) 110 (d) 115 (e) 133.33 140. What is the difference between the average production per year of the company with highest average production and the company with lowest average production in lakh tonnes? (a) 3.17 (b) 4.33 (c) 4.17 (d) 3.33 (e) None of these Direction (141-145) : Study the following graph carefully and answer the question given below it. The number of students who joined and left the school in the beginning of years for six years, from 2000 to 2005. Initial strength of the school in 1999 = 1500 4 3.5 3 2.5 2 1.5 1 0.5 0 2000 2001 2002 2003 2004 2005

Number of students left

Number of students joined

141. What was the increase/decrease in strength of the school from 2001 to 2002? (a) Increase by 100 (b) Decrease by 100 (c) Increase by 200 (d) Decrease by 200 (e) None of these 142. For which of the following years, the percentage rise/fall in number of students left from the previous years is the highest ? (a) 2001 (b) 2002 (c) 2003 (d) 2004 (e) 2005 143. How many students were there in the school during the year 2003?

The information provided here is only for refrence. It may vary from the original. www.recruitment.guru

(a) 1495 (b) 1600 (c) 1550 (d) 1700 (e) None of these 144. During which of the following pairs of years, the strength of school is equal ? (a) 2001 and 2002 (b) 2002 and 2004 (c) 2003 and 2005 (d) 2002 and 2005 (e) 2000 and 2002 145. The number of students in 2003 is approximately what percent of the number of students in 2001 ? (a) 85 (b) 117 (c) 95 (d) 103 (e) 108 Directions (146-150) : In each of the following questions a number series is given. After the series, a number is given followed by (a), (b), (c), (d) and (e). You have complete the series starting with the number given following the sequence of the given series. Then answer the question given below it.

146. 9 19.5 41 84.5 12 (a) (b) (c) (d) (e) Which of the following numbers will come in place of (c) ? (a) 111.5 (b) 118.5 (c) 108.25 (d) 106.75 (e) None of these 147. 4 5 22 201 7 (a) (b) (c) (d) (e) Which of the following number will come in place of (d) ? (a) 4948 (b) 4840 (c) 4048 (d) 4984 (e) None of these 148. 5 5.25 11.5 36.75 7 (a) (b) (c) (d) (e) Which of the following number will come in place of (c) ? (a) 34.75 (b) 24.75 (c) 24.5 (d) 34.5 (e) None of these 149. 38 19 28.5 71.25 18 (a) (b) (c) (d) (e) Which of the following number will come in place of (d) ? (a) 118.75 (b) 118.25 (c) 108.25 (d) 118.125 (e) None of these

The information provided here is only for refrence. It may vary from the original. www.recruitment.guru

150. 25 146 65 114 39 (a) (b) (c) (d) (e) Which of the following number will come in place of (e) ? (a) 122 (b) 119 (c) 112 (d) 94 (e) None of these Directions (151-155) : Read the following statement carefully to answer the given question. A committee of 12 persons is to be formed from 9 women and 8 men. 151. In how many ways this can be done if atleast 5 women have to be included in a committee? (a) 6000 (b) 6010 (c) 6062 (d) 6005 (e) None of these 152. In how many of these committees, the women are in majority? (a) 2000 (b) 2700 (c) 2705 (d) 2702 (e) None of these 153. In how many of these committees, the men are in majority? (a) 1008 (b) 1100 (c) 1200 (d) 1225 (e) None of these 154. An urn contains 9 red, 7 white and 4 black balls, If two balls are drawn at random, find the probability that both the balls are red

(a)

(b)

(c)

(d)

(e) None of these

155. How many different words can be formed with the letters of the word ‘ALLAHABAD’? (a) 7500 (b) 7560 (c) 7510 (d) 7580 (e) None of these Direction (156-160) : Study the following graph carefully and answer the question given below. Percentage of different types of employees in a company in two consecutive years

The information provided here is only for refrence. It may vary from the original. www.recruitment.guru

F A 22% 20%

B 6% C E 10% 27% D 15%

2005 Total no. employees 42,980

F A 21% 22%

B 10% E 27% C 11% D 9%

2006 Total no. of employees

48,640

156. In 2005 the total no. of which the following types of pairs of employees was approximately equal to A type of employees in 2006? (a) B and C (b) A and C (c) D and E (d) C and D (e) C and F 157. From 2005 to 2006 in the case of which of the following types of employees the change was maximum? (a) B (b) D (c) C (d) A (e) None of these 158. What was the approximate different in the number of B type of employees during 2005 and 2006?

The information provided here is only for refrence. It may vary from the original. www.recruitment.guru

(a) 2285 (b) 2325 (c) 2085 (d) 2620 (e) 1825 159. If the no. D type of employees in 2006 was 5000, what would have been its approximate percentage in the company? (a) 8 (b) 12 (c) 14 (d) 16 (e) 10 160. The no. of A type of employees was approximately what per cent of the no. of A type of employees in 2005 ? (a) 115 (b) 140 (c) 125 (d) 130 (e) 95

161. 12 men take 36 day to do a work while 12 women complete th of the same work in 36

days. In how many days 10 men and 8 women together will complete the same work? (a) 6 (b) 27 (c) 12 (d) Date inadequate (e) None of these 162. Rs. 800 becomes Rs. 956 in 3 years at certain simple rate of interest. If the rate of interest is increased by 4%, what amount will Rs. 800 become in 3 years? (a) Rs. 1020.8 (b) Rs. 1025 (c) Rs. 1052 (d) Data inadequate (e) None of these

163. What least number would be subtracted from 427398 so that remaining number is divisible by 15? (a) 6 (b) 3 (c) 16 (d) 11 (e) None of these 164. A car covers its journey at the speed of 80 km/hour in 10 hours. If the same distance is to be covered in 4 hours, by how much the speed of car will have to increase? (a) 8 km/hr (b) 10 km/hr (c) 12 km/hr (d) 16 km/hr (e) None of these 165. What approximate value will come in place of the question mark (?) in the following equation ?

33 % of 768.9 + 25% of 161.2 – 58.12 =?

(a) 230 (b) 225 (c) 235

The information provided here is only for refrence. It may vary from the original. www.recruitment.guru

(d) 220 (e) 240 166. If on selling 12 notebooks any seller makes a profit equal to the selling price of 4 notebooks, what is his per cent profit? (a) 50 (b) 25

(c)

(d) Data inadequate (e) None of these 167. Present age of Rahul is 5 years less than Ritu’s present age. If 3 years ago Ritu’s age was x, which of the following represents Rahul’s present age? (a) (b) (c) (d) (e) None of these 168. A grocer purchased 2 kg . of rice at the rate of Rs. 15 per kg. And 3 kg. of rice at the rate

of Rs. 13 per kg. At what price per kg. should he sell the mixture to earn % profit on

the cost price. (a) Rs. 28.00 (b) Rs. 20.00 (c) Rs. 18.40 (d) Rs. 17.40 (e) None of these 169. A boat takes 6 hours to travel from place M to N downstream and back fro N to M upstream. If the speed of the boat in still water is 4 km/hr,. what is the distance between the two places? (a) 8 kms. (b) 12 kms. (c) 6 kms. (d) Data inadequate (e) None of these 170. Mr. Yadav spends 80% of his monthly salary on consumable items and 50% of the remaining on clothes and transport. He saves the remaining amount. If his Rs. 5370, how much amount per month he would have spent on clothes and transport? (a) Rs. 4,037 (b) Rs. 8,0763 (c) Rs. 9,691.20 (d) Rs. 4,845.60 (e) None of these Directions (176-180) : In each question below one or two equation (s) is/are provided. On the basis of these you have to find out relation between p and q. (a) If p =q (b) If p > q (c) If p < q (d) If p q and (e) If q p 171. I. II. 172. 173. 174. 175.

The information provided here is only for refrence. It may vary from the original. www.recruitment.guru

GENERAL AWARENESS 176. Why was Fillip Calderon is the new ? (a) He has been appointed the secretary General of European Union (b) He has been relected the President of Mexico (c) He has been awarded the international Astronaut of the Year Award by NASA. (d) He has been relected the Prime Minister of Guyana (e) None of these 177. Where was ninth China-European Union summit held ? (a) Olso (b) Sanya (c) Helsinki (d) Shanghai (e) None of these 178. ‘Crown Prince Cup Basketball’ in which India won a Bronze Medal was organised recently in- (a) India (b) Thailand (c) China (d) Japan (e) Singapore 179. ‘Jnanpith Award’ is given for the excellence in the field of (a) Music (b) Politics (c) Literature (d) Sports (e) None of these 180. Ozone gets depleted in the Stratosphere due to the presence of : (a) Active NO3 (b) Active NO2 (c) Active CI (d) Active SO2 (e) None of these 181. This of the following countries has awarded Congressional Gold Medal to Dalai Lama, Tenzing Gyatso? (a) Britain (b) France (c) Italy (d) USA (e) None of these 182. Who amongst the following is the author of the book ‘Bearders-My Life in Cricket’? (a) Shane Warne (b) Bill Frindall (c) Brian Lara (d) Imran Khan (e) None of these 183. Which step is taken first in designing a programme ? (a) Data validation (b) Task analysis (c) Input design (d) Problem identification (e) None of these 184. The government offices of which country are now become paperless? (a) Holland (b) Sweden (c) New Zealand

The information provided here is only for refrence. It may vary from the original. www.recruitment.guru

(d) Estonia (e) None of these 185. Who amongst the following is the author of the book ‘The city of Joy’? (a) Dominique Lapierre (b) Guentar Grass (c) Daniel Steele (d) Graham Green (e) None of these 186. Who amongst the following inaugurated the Hanover Technology Trade- 2006? (a) Dr. Manmohan Singh (b) Dr. P. Chidambaram (c) Mr. Kamal Nath (d) Mr. Pranb Mukherjee (e) None of these 187. How many items are deserved recently from the reserved list of small scale Industries ? (a) 60 (b) 68 (c) 100 (d) 180 (e) None of these 188. The Government of India decides the minimum support price of which of the following commodities? (a) Sugarcane (b) Green vegetables (c) Medicinal plants (d) Milk (e) None of these 189. The Mid day Meal Scheme is launched by the Union Ministry of- (a) Home Affairs (b) Social Welfare (c) Human Resource Development (d) Rural Development (e) None of these 190. The United Western Bank which was recently in news is a . (a) Nationalized Bank (b) Co-operative Bank (c) Private Bank (d) Foreign Bank (e) None of these 191. Dr. Manmohan Sing was on a visit to Uzbekistan a few months back where he signed seven MoUs/ Pacts. Who amongst the following signed these MoUs/pacts on behalf Uzbekistan? (a) Mr. Islam Karimov (b) Mr. Hamid Wazir (c) Mr. Hassan Wirajuda (d) Sultan Bin Kasim (e) None of these 192. Who amongst the following is the chairperson of the ‘National Knowledge Commission? (a) Mr. Sam Pitroda (b) Dr. Ashok Ganguly (c) Dr. Jayanti Ghosh (d) Dr. Pratap Bhanu Mehta (e) None of these

The information provided here is only for refrence. It may vary from the original. www.recruitment.guru

193. The final of the Sultan Azlan Shah Hockey Tournament 2006 was organised recently (18 June, 2006) in . (a) Kuala Lumpur (b) Jakarta (c) Manila (d) New Delhi (e) None of these 194. Ustad Bismillah Khan who died recently (21 August, 2006) was famous. (a) Classical Dancer (b) Classical Singer (c) Shehnai Player (d) Tabla Player (e) Santoor Player 195. The two day meeting of the Finance Ministers of G-8 nations was organised recently in (10-11 June 2006) (a) St Petersburg (b) Berlin (c) Milan (d) Moscow (e) Paris 196. Who amongst the following is the recipient of the ‘Rajiv Gandhi Khel Ratna Award’ given away recently (29 August, 2006)? (a) Mahesh Bhupathi (b) Sania Mirza (c) Rahul Dravid (d) Anju Bobby george (e) Pankaj Advani 197. Who amongst the following won the French Open 2006 Tennis cup held recently (May- June 2006) ? (a) Roger Federer (b) Rafael Nadal (c) Bob Bryan (d) Vera Zvonareva (e) None of these 198. Who amongst the following is the recipient of the ‘Sahitya Shiromani Samman’ given away recently ? (a) Javed Akhtar (b) Gulzar (c) Kamleshwer (d) Malti Joshi (e) None of these 199. Which of the following is a graphic package ? (a) Adobe Page Maker (b) Acrobat Reader (c) Microsoft Word (d) Corel Draw (e) None of these 200. Hardware that adds two numbers is . (a) The control unit (b) An internal CPU register (c) Arithmetic logical unit (d) Browser (e) None of these 201. Which of the following statements is/are True about the integrated Anti Submarine Warfare (ASW) equipment ?

The information provided here is only for refrence. It may vary from the original. www.recruitment.guru

(A) The Defence Research and Development Organisation (DEDO) handed over the integrated Anti Submarine Warfare equipment to Anti navy. (B) It has the ability to detect enemy usages while being underwater and also fire torpedoes or missiles. (C) The cost of this equipment is about Rs. 4 crores. (a) A and B (b) A and C (c) B and C (d) All are correct (e) None of these 202. In which plane is the Brahmos missile fitted ? (a) Sukhoi-30MKI (b) Jaguar (c) Mirage (d) Mig-29 (e) None of these 203. From which date the wildlife (Protection) Amendment Act 2006 has come into effect after the president’s approval? (a) 1st September, 2006 (b) 4th September, 2006 (c) 15th August, 2006 (d) 15th July, 2006 (e) 4th June, 2006 204. At which place would the Indian navy set up the second naval base? (a) Vasakhapatnam (b) Mangalore (c) Tuticorin (d) Nellore (e) None of these 205. How many agreements were signed between India and Brazil during the Prime Minister Manmohan Singh Brazil visit ? (a) 5 (b) 7 (c) 9 (d) 11 (e) None of these 206. Who amongst the following was the head of the Investment commission which submitted its report to the Government of India recently? (a) Mr. Ratan Tata (b) Dr. Rakesh Mohan (c) Mr. Kurnar Mangalam Birla (d) Mr. Rahul Bajaj (e) None of these 207. As per the report in the newspaper the old age pension is now raised to which of the following amounts from the present Rs. 75? (a) Rs. 100 (b) Rs. 150 (c) Rs. 175 (d) Rs. 200 (e) Rs. 250 208. The interest rate at which the RBI lends against the Government Securities is known as (a) CRR (b) SLR (c) Bank Rate

The information provided here is only for refrence. It may vary from the original. www.recruitment.guru

(d) Repo Rate (e) None of these 209. Which of the following is the abbreviated name of an Indian Organisation/agency associated with export of processed food products? (a) APEDA (b) AAPSO (c) AINEF (d) AITUC (e) None of these 210. Which of the following Industries suffered a huge loss of almost Rs. 20 crore pre day due to floods in Gujarat ? (a) Oil and Natural gas (b) Information Technology (c) Sugar mills (d) None of these 211. Which of the following is NOT one of the core and identified under the Bhart Nirman Programme? (a) Irrigation (b) Rural electrification (c) Rural housing (d) Computer education in schools (e) Drinking water supply 212. The Base II Accord for Banking Industry is base on three pillars. Which of the following is/are NOT included in the same ? (a) Minimum Capita Requirement (b) Supervisory Review (c) Market Discipline (d) Credit Risk Codes : (a) Both (A) and (C) (b) Only (D) (c) (A), (B) and (C) (d) Only (A) (e) None of these 213. ‘Shequel’ is the currency of- (a) Israel (b) Kenya (c) Iraq (d) Iran (e) None of these 214. Indian Exports recorded an increase in May 2006. What was the increase in terms of percentage ? (a) 15 per cent (b) 20 per cent (c) 25 per cent (d) 30 per cent (e) None of these 215. Who amongst the following is the first chairman of the newly Constituted National Statistical Commission ? (a) Prof. Suresh D. Tendulkar (b) Dr. Ashok Lahiri (c) Dr. Rakesh Mohan (d) Prof. S.K. Ghosh (e) None of these

The information provided here is only for refrence. It may vary from the original. www.recruitment.guru

216. Government of India recently was decided to import wheat at a lower rate of custom duty. The effective rate of the duty was. (a) 15 per cent (b) 12 per cent (c) 10 per cent (d) 5 per cent (e) None of these 217. As per the reports published recently in the newspapers banks surpassed the target set for farm credit was given in this sector in terms of percentage? (a) 15 (b) 20 (c) 26 (d) 33 (e) 43 218. As per the reports published in the newspapers the National Housing Bank (NHB) is planning to launch a Reserve Mortgage scheme specially to held which of the following sections of society ? (a) People in rural India (b) Senior Citizens (c) People living in government accommodations (d) Women who are sole bread winners (e) None of these 219. Which of the following companies has got two contracts to establish two power projects in Afghanistan ? (a) Bharat Heavy Electricals (BHEL) (b) National Hydro Electric Power Corporation (NHPC) (c) National Thermal Power Corporation (NTPC) (d) Reliance Energy Ltd. (e) None of these 220. Government of India recently introduced some measures/incentives to improve tourism in India. Which of the following is one of these measures ? (a) Issuance of electronic visa (b) Free stay for the three days in Government hotels in four metros for business travellers and frequent players (c) Highly subsidised air travel on metro routes for frequent flyers (d) Free trip to Agra, Delhi and Jaipur to those coming on educational/study tours (e) None of these 221. How much amount can the Postal Department invest in revenue generating instruments and/or stock market to reduce its budgetary deficit? (amount is from its insurance schemes). (a) Rs. 1,400 crore (b) Rs. 2,500 crore (c) Rs. 4,500 crore (d) Rs. 7,400 crore (e) Rs. 10,000 crore 222. Plotter are very useful in applications such as. (a) Computer aided design (b) Word Processing (c) Financial Accounting (d) Spell checking (e) None of these 223. Employees details is. (a) A master file for the payroll system (b) A replica of the monthly pay slip (c) Not related to the payroll system

The information provided here is only for refrence. It may vary from the original. www.recruitment.guru

(d) A transaction file for monthly pay slip (e) None of the 224. A computerized business information system includes (a) Hardware (b) Software (c) Data facts (d) All (e) None of these 225. Data processing cycle consists of – (a) Input cycle and output cycle (b) Output cycle and processing cycle (c) Input cycle, output cycle and processing cycle (d) Only input cycle (e) None of these

The information provided here is only for refrence. It may vary from the original. www.recruitment.guru

Answers

The information provided here is only for refrence. It may vary from the original. www.recruitment.guru

Explanations

(1-5) Person City Specialisation P. Hyderabad Literature Q. Calcutta Computers R. Mumbai Marketing S. Chennai Textile/ Economics T. Delhi Textile / Economics V Trivandrum Physics X. Bangalore Information Technology

(6-10). After careful analysis of the given passcodes for successive batches we can conclude that the passcode for the next batch is generated by rearranging the words of the passcode of previous batch in the following manner:

BATCH I ( 9 AM TO 10 AM)

Passcode:

1 2 3 4 5 6 7

Course easy set for each year was

BATCH II ( 10 AM to 11 AM)

Passcode:

2 5 1 4 7 3 6

Easy each course for was set year

1 2 3 4 5 6 7

BATCH III (11 AM to 12 Noon)

Passcode:

2 5 1 4 7 3 6

Each was easy for year course set

1 2 3 4 5 6 7

BATCH IV (12 Noon to 1 PM)

Passcode:

2 5 1 4 7 3 6

Was year each for set easy course

1 2 3 4 5 6 7

The information provided here is only for refrence. It may vary from the original. www.recruitment.guru

BATCH V (2 PM to 3 PM )

Passcode:

2 5 1 4 7 3 6

Year set was for course each easy

1 2 3 4 5 6 7

BATCH VI (3 PM to 4 PM )

Passcode:

2 5 1 4 7 3 6

Set course year for easy was each

1 2 3 4 5 6 7

BATCH VII (4 PM to 5 PM)

Passcode :

2 5 1 4 7 3 6

Course easy set for each year was

1 2 3 4 5 6 7

BATCH VIII (5 PM to 6 PM )

Passcode :

2 5 1 4 7 3 6

Easy each course for was set year

1 2 3 4 5 6 7

BATCH IX (6 PM to 7 PM

Passcode :

2 5 1 4 7 3 6

Each was easy for year course set

1 2 3 4 5 6 7

BATCH X (7 PM to 8 PM)

Passcode :

2 5 1 4 7 3 6

The information provided here is only for refrence. It may vary from the original. www.recruitment.guru

Was year each for set easy course

Thus, we see that after Batch VI, the passcode for each subsequent batch is repeated starting from the Batch I. In other words, the following pairs of Batches have the same passcode.

(a) Batch I and Batch VII (a) Batch II and Batch VIII (a) Batch III and Batch IX (a) Batch IV and Batch X

On the basis of above analysis we can write any required passcode directly from the given passcode in the following manner.

BATCH PASSCODE

I 1 2 3 4 5 6 7

Course Easy set for each year was

II 2 5 1 4 7 3 6

III 5 7 2 4 6 1 3

IV 7 6 5 4 3 2 1

V 6 3 7 4 1 5 2

VI 3 1 6 4 2 7 5

VII 1 2 3 4 5 6 7

VIII 2 5 1 4 7 3 6

IX 5 7 2 4 6 1 3

X 7 6 5 4 3 2 1

6. BATCH IV (12 Noon to 1 PM)

7 6 5 4 3 2 1

She the girl is clever very good

BATCH VI (3 PM to PM)

3 1 6 4 2 7 5

Clever good the is very she girl

7. The passcode for the batch IX will be similar to the passcode for the batch III.

The information provided here is only for refrence. It may vary from the original. www.recruitment.guru

8. BATCH (11 AM to 12 Noon )

5 7 2 4 6 1 3

Pin to the point is sharp not

BATCH V ( 2 PM to 3 PM)

6 3 7 4 1 5 2

Is not to point sharp pin the

9. BATCH II (10 AM to 11 AM )

2 5 1 4 7 3 6

For the life is good change got

Passcode :

6 3 7 4 1 5 2

got change good is life he for

Cleary, this is the passcode for Batch V.

10. BATCH IV (12 Noon to 1 PM)

7 6 5 4 3 2 1

Do how will the you job now

BATCH II (10 AM to 11 AM)

2 5 1 4 7 3 6

Job will now the do you how

11. It is clear from the statement that both the assumptions are implicit. The statement clearly implies the first assumption. Again, if it is not desirable complete issue why such statement would have been made.

12. Only assumption II is implicit in the statement. If two months period is not sufficient to implement such policy decision, the Pensioner’s Forum would not have made such a statement.

13. Neither of the assumption is implicit in the statement. On the basis of the precedent it is erroneous to hypothesise that the flyovers do not serve any public objective.

14. Both the assumptions are implicit in the statement. The point which is highlighted in an advertisement is liked by people and is also desirable.

15. Only assumption I is implicit in the statement. The traders of state ‘K’ resorted to observe bandh when other measures or efforts failed to resolve the problem. Note down the use of

The information provided here is only for refrence. It may vary from the original. www.recruitment.guru

phrase “the state has failed “ . It implies that the State ‘K’ made efforts to resolve the issue but failed. It does not imply that it is not interested in resolving the problem.

(16-20). First of all define each symbol clearly:

I. A + B means A < B or A > B It implies that A = B II. A x B means A = B or A < B It implies that A > B III. A? B means A > B or A = B It implies that A < B IV. A @ B means A > B V. A $ B means A = B It implies that A > B or A < B

16. Statements

C + D, D $ M, M? C

After conversion

C = D, D > M or D < M, M < C

Or (i) C= D > M < C

Or

(ii) C = D < M < C

Conclusions

I . M $ C = M < C or M < C

It is clear that M is smaller than C.

II. D + M = D = M

It is clear that D is either greater or smaller than M.

17. Statements

K $ M, M x B, K + B

After conversion

K > M or K < M, M > B, K = B or, M > B = K

Conclusions

I . M x K = M > K

K is equal to B and M is greater than B. Therefore, M is definitely greater than K.

II. K? M = K < M

The information provided here is only for refrence. It may vary from the original. www.recruitment.guru

It is clear that K is

18. Statements

F @ G, G? P, P x H

After conversion

F > G, G < P, P > H

Or (i) F > G < P > H

Or

(ii) F = G < P > H

Conclusions

I. F @ H = F > H

II. H x G = H > G

Neither conclusion I nor II is definitely true.

19. Statements

M @ T, T x R, R $ Q

After conversion

M > T, T > R, R > Q or R < Q

Or (i) M = T > R > Q

(ii) M > T > R > Q (iii) M > T > R < Q

(iv) M = T > R < Q

Conclusions

I . M ? R = M < R

It is clear that M is greater than R.

Therefore, conclusion I is not true.

II. M ? Q = M < Q

From statements (i) and (ii) M is greater than Q builts is not possible to determine any definite relation between M and Q from statements (iii) and (iv).

20. Statements

D x G, G @ K, K + M

The information provided here is only for refrence. It may vary from the original. www.recruitment.guru

After conversion

D > G, G > K, K = M

Or (i) D > G > K = M

Or

(ii) D > G = K = M

Conclusions

I . M + G = M = G

M is either equal to or smaller than G.

II. D x K = D > K

It is clear that D is greater than K.

Therefore, only conclusion II is definitely true.

21. From statement I

D B C N

NW NE

A W E

SW SE

S

It is clear that village C is to the North- East of village A

From statement

A

K

P E

We cannot solve the question with the help of this diagram.

22. From both the statements it is clear that if Rohan retires from office ‘X’ in January 2006 he could not get full pension benefits.

23. From statements I

The information provided here is only for refrence. It may vary from the original. www.recruitment.guru

S > T > P and Q

From statements II

> > > > R From both the statements

S > T > P and Q > R

Either P or Q will come at the third place.

24. From Statements I.

P is the husband of R, who is the daughter of J.

There is no information about the other daughter of J.

From statements II

Q is the mother of S and R.

S is daughter of Q and sister of R.

From both the statements

J is the husband of Q

Q is the wife of J.

J is father of R and S

R and S are daughter of J and Q.

Q is the mother of R and S

P is the husband of R and hence P is the son-law of Q.

25. From statement I.

De fu la pane = rose flower is beautiful la Quiz = beautiful tree

No answer.

From statement II

De la Chin = red rose flower Pa = tea Chin red Both the statements provide different coding pattern.

The information provided here is only for refrence. It may vary from the original. www.recruitment.guru

26. It is not judicious to regard asbestos industry as one of the largest industries in India considering the number of employees engaged in this industry and also the number of units of this industry in India.

27. It is said that asbestos industries in India are located in several states.

28. Advanced countries are banning cancer causing asbestos products: it imples that advanced countries are concerned and careful to product health hazards o their people.

29. It can be safely assumed that if the industry is growing in India there certainly exists demand for its products.

30. Consider the statement “… indifference of government machinery….”

36- 38 4 cm BLUE

3 cm

GREEN 5 cm RED 5 cm

3 cm

4 cm

36. The cubes located at the eight corners would have all the three colours.

37. The six inside cubes will have no face coloured.

38. Twelve cubes will have two colours red and green in their two sides.

39.

1, 2 (3) 4,5 6 7,8,9 10

A B C D A

2 Days 3 Days 1 Day 3 Days Saturday

10th Day  Saturday

3rd Day  Saturday

Now, 2nd Day  Friday

1st Day  Thursday

40. A T M O S P H E R E

The information provided here is only for refrence. It may vary from the original. www.recruitment.guru

Thus, there are two such pairs of letters.

41. lce Ring Paint

Gold

The shaded portion represents doubtful proposition.

Since two of the premises are particular, the Universal conclusions can be discarded. Therefore, conclusions I, II and IV are invalid.

For conclusion III, the second and the third premises are relevant. We can align the premises by converting the third premise and changing the order of the premises. Thus,

Some gold are rings

No ring is paint

We know that

I + E = O type conclusion

Therefore, our derived conclusion would be :

“Some gold are paints”.

Similarly, we can take the first and the second premises. Thus,

Some ice are rings

No ring is paint

We know that,

I + E = O type conclusion

Thus, our derived conclusion would be:

“Some ice are not paints”.

42.

Clip

Flowers Fruits Gates

The information provided here is only for refrence. It may vary from the original. www.recruitment.guru

For conclusion I the first and the second premises are relevant. We can align these premises by converting the second premise and changing the order of the premise.

Some fruits are gates

All gates are flowers

We know that,

I + A = I type conclusion.

Thus, our derived conclusion would be:

“Some fruits are flowers”

Conclusion I is the conversion of our derived conclusion.

Since all the premises are Affirmative, no Negative conclusion can be derived from them. Therefore, conclusion IV is invalid.

43. From the first to premises we can derive the conclusion:

“Some shoes are not candles”

No candle is bell

Some bells are shoes

We know that,

E + I = o, type conclusion

Tables Bell Candle Shoes

The shaded portion implies, doubtful proposition.

Conclusion I and II from complementary pair. Therefore, either conclusion I or II follows.

The information provided here is only for refrence. It may vary from the original. www.recruitment.guru

Similarly, conclusions III and IV form complementary pair.

44. All the three premises are Affirmative and hence the Negative conclusion is invalid.

Therefore, conclusions I dose not follow.

Clouds Rats

Films Chairs

Conclusion III is the conversion of the second premise.

45.

Lice Slates

Apples Car

For conclusion III the first and the second premises are relevant Both the premises are already aligned

Some lice are slates

All slates are apples

We know that

I + A = I type conclusion would be

“Some lice are apples”.

Conclusion III is the conversion of the derived conclusion.

Conclusion II and IV form complementary pair. Hence conclusion II or IV follows.

46. The given number ends with an even number and hence the last even number will be symbolised as © 6 7 3 2 5 8

• + ? * $ © . 47. 2 3 6 4 7 5

The information provided here is only for refrence. It may vary from the original. www.recruitment.guru

* ? • ÷ + $ 48. 8 4 6 7 2 1

! + • + * x 49. The number begins with odd number and hence 1 will be written as @ and the number ends with even number and hence 4 will be written as © 1 7 8 5 2 4

@ + ! $ * © 50. The given number ends with even number and hence 6 will be written as ©. 2 5 4 8 6

* $ ÷ ! © 51. According to question. 589 = 559 = 955 362 = 332 = 233 554 = 524 = 425 371 = 341 = 143 442 = 412 = 214

Descending order 955 > 425 > 233 > 214 > 143

Middle Number 23 3

Last digit 52. There are 11 letters each in the given word and its code. It means that there is one code letter for each letter of the given word. After careful observation of the given word and its code it is clear that the code has been generated by rearranging the letters of the given word. The middle letter of the given word remains at its position while other letters of both sides of it are rearranged in a certain way. 1 2 3 4 5 1 2 3 4 5 G E R M I N A T I O N

The letter in each half have been rearranged in the order: 54132 Now, G E R M I N A T I O N

I M G R E N N O A I T

Similarly,

1 2 3 4 5 1 2 3 4 5 L E S T A B I S H E D

B A E T S L D E I H S

Therefore, our required answer is option (b).

53. There is one letter more in the code as compared to the letters of the given word.

The information provided here is only for refrence. It may vary from the original. www.recruitment.guru

The pattern off coding is as follows: P +2 P R

O +2 Q

U +3 X

L +2 N

T + 2 V

R +2 T

Y Y Similarly, T

T +2 V

R +2 T

E +3 H

A +2 C

S +2 U

O +2 Q

N N Therefore, our required answer is option (d).

54. According to question the new sequence would be : ACEGIKOQS U WYZXVTRPNLJHFDB

11th from left 6th to the left of 17th left means (17-6) =11th letter left i.e. 21st letter from left in the original sequence.

55. 1 2 3 4 5 6 7 8 9 10 11 12 13

S T S B L I S H M E N T

Specified letters : S, A , B, E. T Meaningful words: BEAST, BEAST (56-60) : The given information can be analysed as:

Candidate CRITERIA

The information provided here is only for refrence. It may vary from the original. www.recruitment.guru

or or or

(i) (ii) (a) (iii) (iv) (c) (v) (b) Deepali √ __ √ √ √ __ √ __

Ajit √ √ __ √ NG __ √ __

Rajendra √ √ __ √ __ √ √ __

Harish N G __ __ √ √ √ √

Tarique √ √ __ √ √ __ √ __

56. Deepali Mirza satisfies all the criteria except (ii but she does satisfy (a) instead.

Therefore, her case may be referred to the Director.

57. In case of Ajit D’souza no information is given regarding the criterion (iv).

58. Rajendra Bhave satisfy all the criteria except (iv) but he does satisfy (c) instead. Therefore, his case may be referred to the President.

59. In case of Harish Chheda complete information (Mark in Graduation) is not given regarding the criterion (ii) or (a).

60. Tarique Answer satisfy all the criteria.

61. The movement and changes of designs in the subsequent figures can be shown as:

From Problem Figure (1) to (2)

• • New Design •

• • From Problem Figure (2) to (3)

• •

New Design •

• • These two steps are repeated alternately and a new design appears in the centre in each subsequent figure.

62. In the subsequent figures respectively one, one and half and half part (s) of triangle get (s) shaded in a set pattern.

The information provided here is only for refrence. It may vary from the original. www.recruitment.guru

63. From Problem Figure (1) to (2) the lower design ascends vertically and it is replaced by a new design, the left design moves anticlockwise and the right design moves to the left. Similar changes would occur from Problem Figure (5) to Answer figure.

64. From Problem Figure (1) to (2) the line segments at the left end rotate through 45˚ anticlockwise while that of upper line segments at the right end rotate through 45˚ anticlockwise. Similar changes occur from Problem Figure (3) to (4) and from Problem Figure (5) to Answer Figure.

65. From Problem Figure (1) to (2) all the designs moves half a step in clockwise direction and the first and the last designs interchange position while a new designs in introduced in front of the per-existing designs. From Problem Figure (2) to (3) all the designs move one step in clockwise direction and again the first and the last designs interchange position so as the two middle designs and a new design is introduced in front of the pre-existing designs. From Problem Figure (3) to (4) all the designs move 1½ step in clockwise direction and the first two designs interchange position with the last two designs and a new design is introduced in front of the pre-existing design. From problem Figure (4) to (5) all the designs move two step in clockwise direction and the first and the last designs have interchanged position so as the second and the two middle design have interchanged position while a new design is introduced in front of the pre-existing designs. Therefore, from Problem Figure (5) to Answer Figure all the designs would move two and one half steps in clockwise direction and also rearrangement of designs would take place and a new design will be introduced in front of the pre-existing designs.

66. Following changes occur from first figure to second figure:

• • New Design • 67. Second figure is the mirror image of the first figure. • • 68. From first figure to second figure black leaflet rotates through 45˚ anticlockwise and the white leaflet rotates through 90˚ clockwise.

69. From first figure to second figure the line segment with triangle rotates 90˚ clockwise while the line segment with circle rotates 135° anticlockwise.

70. From first figure to second figure the diagonally opposite designs interchange position.

71. The main design rotates 90° anticlockwise. The inner and outer line segments move two steps anticlockwise.

72. Except in Figure (2) in all other pairs of figures the shaded portion ascends or descends.

73. Except in figure pair (3) in all other pairs of figures from element I to II the number of arrows is decreased by one while the number of equal signs is increased by one. Again the extreme left arrow rotates 135° clockwise but n figure (3) it has been rotated through 180° clockwise.

74. The smaller designs rotates 90° anticlockwise and the smaller designs become larger while the larger designs become smaller from element I to II Again the right design is inverted.

75.In all other pairs of figures there is one unshaded triangle to the left of shaded triangle.

The information provided here is only for refrence. It may vary from the original. www.recruitment.guru

85. The word assiduously (Adverb) comes from ‘the words assiduous (Adjective). Assiduous means working hard and showing careful attention to detail. For example They planned their careers and worked Assiduously to see them achieved. On the other hand deliberately means intentionally carefully. So, we can choose option (d) as the answer.

86. Idiosyncrasy means a person’s particular way of thinking, behaving, etc. that is clearly different from that of others. For example One of her little idiosyncrasies is always washing in cold water. As ideologies is closest in meaning to it so the answer is (d)

87. The lexical meaning of shoddy (Adjective) is of poor quality, done or made badly: Shoddy goods/workmanship. In the passage Shoddy means cheap.

88. Inducement (Noue) means a thing that persuades somebody to do something, an incentive. For example, Estate agents are offering inducements to first time buyers. In the passage also inducements means incentive. As impediment means barrier, hurdled, so the answer is (e)

89.Unscrupulous means without moral principles; not honest or fair unscrupulous business methods.

90. Contemptuous (Adjective) means feeling or showing contempt. Contemptuously (Adverb) means with contempt.

(91- 95). First sentence (A) The application of economic, environmental and consumer pressures have on an increase in recent years. Second sentence (F) These have greatly influenced the development of the agriculture and food industries in our country. Third sentence (D) In turn, the support industries have developed to an equally advanced state. Fourth sentence (E) They have also equipped themselves with the necessary expertise to satisfy the most exacting requirements of the overseas markets Fifth sentence (C) They are thus able to provide expertise and technology to satisfy the needs of agro food production Sixth sentence (B) As a result, our agro- food production and technology are amongst the most advanced in the world

96. I did neither should be replaced by Nor did I. Structure would be: For example

… + neither/nor + auxiliary verb + subject

The information provided here is only for refrence. It may vary from the original. www.recruitment.guru

Ruth didn’t turn up and Kate did neither,(x) Ruth didn’t turn up and nor did Kate.(√)

Auxiliary Subject Verb

98. The sentence is in Past Perfect continuous. Hence extorted should be replaced by extorting.

99. Infinitive without ‘to’ is used after need. For example, He wondered whether they need send a deposit He wondered whether they need to send a deposit (x)

Infinitive with ‘to’ He wondered whether they need send a deposit(√)

Infinitive without ‘to’ 111. generally an objective is made when a proposal is founded on a wrong principal and is inconvenient.

112. Journalists don’t select; they judge. So, it is said- “The criterion for judging a player..” “Recent” and “earlier” denote different periods of time. So, option (b) is the correct one. 113. One devotes oneself in public affairs. So, we should use devoted in the first blank. Without taking a holiday denotes continuous services. So, without is the appropriate use in the second blank.

114. Sings of industry is an indication of prosperity. So, we can choose option (d)

115. One can be arrested on a particular charge of theft. But he can be released for the lack of evidence. 126. A. Total marks in 4 subject including English = 4 x 60 = 240 B. Total marks in English and Maths = 170 C. Total marks in Maths and Science =180 The question can’t be answered because nothing has been said about the marks in the fourth subject. Also, there are four unknowns but any three equations can be formed with given data.

127. Let the marked price be Rs. x A . Cost price (1 -0.15)x = Rs. 0.85x B. S.P. = Rs. 3060 C. Profit = 2% of x = 0.02x Profit earned on the cost price = 0.02x x 100 = 2.35% 0.85x 0.02x = 3060 – 0.85x 0.87x = 3060 x = 3060 0.87 Actual profit = 0.02x = 0.02 x 3060 = Rs. 70.34 0.87 128. A. x-y = 6

The information provided here is only for refrence. It may vary from the original. www.recruitment.guru

B. 0.4y =0.3x X = 4 Y 3 C. x : y =2:1 2 3 x x 3 = 2 y 2 1 x = 4 y 3 B and C give the same expression/information and hence are equivalent. x = 4 y 3 x- y = 6 4 y – y = 6 3 4 =6 3 y = 18 x = 4 x 18 = 24 3 129. Hypotenuse = √52 + 122

=√25 + 144

=√169 = 13 Base : Height : Hypotenuse = 5 : 12 : 13 Base : Height : Hypotenuse = 30 cm

:. Base = 5 x 30 = 5 cm 5 + 12 +13

Height = 12 x 30 = 12 cm 5 + 12 + 13 Area = 1 x base x height 2 = 1 x 5 x 12 = 30 cm2 2 130. The question cannot be answered because R’s share in investment is not given.

131.Percentage of candidates selected over appeared in 2000. 25 x 100 = 2.94% A = 850 35 x 100= 3.33% B = 1050 28 x 100 = 2.83% C = 990 36 x 100 = 3.33%

The information provided here is only for refrence. It may vary from the original. www.recruitment.guru

D = 1080 40 x 100 = 3.57% E = 1120

The percentage is lowest for state C.

132. Passing percentage over appeared for state D in: 300 x 100 = 27.7% 20001080 320 x 100 = 27.8% 2001--> 1150 280 x 100 = 28.1% 2002--> 995 260 x 100 = 26.67% 2003 --> 976 240 x 100=27% 2004 --> 888 216 x 100 = 25% 2005--> 864 The required percentage is highest in the year 2002.

133. Total no of candidates selected from state ‘A’ = 25 + 20 + 22 + 36 + 32 +28 = 163 Total no. of candidates selected from state ‘B’ = 35 + 30+ 28 + 32 + 40+ 38 =203 :. Required % = 163 x 100 = 80% 203

134. Percentage of candidates selected over passed for state ‘B’ in 35 x 100 = 14.3% 2000 --> 245 30 x 100 = 13 % 2001 --> 230 28 x 100 = 13.3% 2002--> 210 32 x 100= 16.8% 2003--> 190 40 x 100 = 17.8% 2004 --> 225 38 x 100 = 19% 2005--> 200 Required percentage is lowest in 2001

135. No. of candidates passed from state A in 1996 = 240 No. of candidates passed from state E in

The information provided here is only for refrence. It may vary from the original. www.recruitment.guru

1999= 280 :. Required ratio = 240 : 280 = 6 : 7 136. Production of company C in 1999 = 45 lakh tonnes Production of company A in 2004 = 58lakh tonnes :. Required difference = 50- 45 = 5 lakh tonnes 137. Required percentage 55 -40 x 100 = 75 = 37.5% 40 2 139. Total production of company C in 2001 and 2002 = 120 lakh tonnes Total production of company A in 1999 and 2000 = 90 lakh tonnes :. Required percentage = 120 x100 90 = 133 1% 3 140. Average production of company A = 50 + 40 + 55 + 45 +60 + 50 6 = 300 = 50 lakh tonnes 6 Average production of company B = 55 + 60 + 50 + 55 +50 + 55 6 = 325 = 54.17 lakh tonnes 6 Average production of company C = 45 + 60 + 60 + 45 + 40 6 = 300 = 50 lakh tonnes 6 :. Required difference = 54. 17 – 50 = 4.17 lakh tonnes. 141. No. of students who left in the beginning of 2002 = 400 No of students who joined in the beginning of 2002 = 500 :. There was net increase of 100 (=500 – 400) students in the strength of the school from 2001 to 2002. 142. We can come to this conclusion even without performing complete exact calculation, simply by looking / comparing the terms written within the brackets.

143. Total no. of students who joined till 2003 = 300 + 250 +500 + 450 = 1500 Total no. of student who left till 2003 = 250 + 350 + 400 +300 = 1300 Net increase in the strength of the school = 1500 – 1300 = 200 Initial strength of the school in 1999 = 1500 :. Strength of school during 2003 = 1500 + 200 = 1700

Year Strength

1999 1500 2000 1500 + 300 – 150 = 1550

The information provided here is only for refrence. It may vary from the original. www.recruitment.guru

144. 2001 1550 + 250-350 =1450 2002 1450 + 500- 400= 1550 2003 1550 +450 – 300 =1700 2004 1700 +400 -500=1600

2005 1600 +550 – 500= 1650

Strength of the school was equal in 2000 and 2002. 145.From solution to Q. No. 159 we have no. of students in 2003 = 1700 and students in 2001 = 1450 :. Required % = 1700 x 100 1450 = 117% (approx.) 146. The given series is based on the following pattern: 9 x 2 + 1. 5 = 19. 5 19.5 x 2 + 2 = 41 41 x 2 + 2.5 = 84. 5 Therefore, the new series is as follows: 12 x 2 + 1.5 = 25. 5 ... (a) 25.5 x 2 +2 = 53 … (b) 53 x 2 +2.5 = 108.5 …. (c) 108.5 x 2 + 3 = 220 ... (d) 220 x 2 +3.5 = 443.5 .... (e) Therefore, the number 108.5 will come in place of (c) in the new series. 147. The series is based on following pattern 4 x 1 + 1 = 5 + 3 5 x 4 + 2 = 22 + 5 22 x 9 + 3 =201 Similarly the new series is as follows: 7 x 1 + 1 = 8 ..(a) 8 x 4 + 2 = 34 .. (b) 34 x 9 + 3 = 309 .. (c) 309 x 16 + 4 = 4948 ..(d) Therefore, the number 4948 will come in place of (d) in the new series

148. The series is based on following pattern: 5 x 1 + 0.25 x 1= 5.25 + 3 5.25 x 2 + 0.25 x 4 = 11.5 + 5 11.5 x 3 +0.25 x 9 =36.75 Similarly, the new series is as follows: 3 x 1 +0.25 x 1 = 3 .25 ... (a) 3.25 x2 + 0.25 x 4 = 7.5 ...(b) 7.5 x 3+0.25 x 9 = 24.75 ...(c) Therefore, the number 24.75 will come in place of (c) in the new series.

The information provided here is only for refrence. It may vary from the original. www.recruitment.guru

149. The series is based on following pattern: 38 x 0.5 = 19 19 x 1.5 = 28.5 28.5 x 2.5 = 71.25 Similarly, the new series is as follows: 18 x0.5 =9 … (a) 9 x 1.5 = 13.5 … (b) 13.5 x2.5 =33.75 ...(c) 33.75 x 3.5 = 118.125.. (d) Therefore, the number 118.125 will come in place of (d) in the new series. 150. The series is based on following pattern: 25 + (11)2 = 25 + 121 = 146 146 – (9)2 = 1446 -81 = 65 65 + (7)2 = 65 + 49 =114 Similarly, the new series is as follows: 39 + (11)2 = 39 + 121 = 160 … (a) 160- (9) 2 = 160 -81 = 79 …(b) 79 +(7)2 = 79 + 49 = 128 …(c) 128 + (5)2 = 128 – 25 = 103 .. (d) 103 +(3)2 = 103 + 9 =112 ..(e) Therefore, the number 112 will come in place of n(e) in new series. 151. There are 9 women and 8 men. A committee of 12, consisting of at least 5 women, can be formed by choosing: (i). 5 women and 7 men (ii) 6 women and 6 men (iii) 7 women and 5 men (iv) 8 women and 4 men (v) 9 women and 3 men :. Total number of ways of forming the committee = 9C5 x 8C 7 + 9 C6 x 8C6 + 9C7 x 8C5 + 9C8 x 8C4 + 9C9 x 8C3 = 126 x 8 +84 x 28 + 36 x 56 + 9 x 70 + 1 x 56 = 6062 152. Women are in majority in (iii), (iv) and (v) cases as discussed in question 151. :. Total number of such committees = 9C5 x 8C5 + 9C8 x 8C4 + 9C9 x 8C3 = 36 x 56 +9 x70+1 x 56= 2702 153. Men are in majority in only (i) case as discussed in question 151. :. Total number of such committees = 9C5 x 8C7 = 126 x 8 = 1008 154. There are 20 balls in the turn out of which 2 balls can be drawn in 20C2 ways :. Total number of elementary events = 20C2 = 190 9 There are 9red balls out of which 2balls can be drawn in C2 ways.

:. Favourable number of elementary events = 9C2 = 9 x 8 = 36 1 x 2 :. Required probability 36 = 18 190 95

155. There are 9 letters in the word ALLAHABAD out of which 4 are As, 3 are Ls and the rest are all distinct. So, the requisite number of words

The information provided here is only for refrence. It may vary from the original. www.recruitment.guru

= 9! = 7560 4! 2! 156. No. of A type of employees in 2006. = 0.22 x 48640 = 10700 In 2005, A = 0.20 x 42980 = 8596 B = 0.06 x 42980 = 2579 C = 0.10 x 42980 = 4298 D = 0.15 x 42980 = 6447 E = 0.27 x 42980 = 11605 F = 0.22 x 42982 = 9455 C + D = 4298 + 6447 = 10745 157. In 2006, A : 10700 B : 0.10 x 48640 = 4864 C : 0.11 x 48640 = 5450 D : 0.09 x 48640 = 4377 E : 0.27 x 48640 = 13133 F : 0.21 x 48640 = 10214 % change during 2005-2006 A : 10700 – 8596 x 100 = 24.5% 8596 B : 4864 -2579 x 100 = 88.6% 2579 C : 5350 – 4298 x 100 = 24.5% 4298 D: 6447 – 4377 x 100 = 32.1% 6447 E : 13133 – 11605 x 100 = 13.2% 11605 F : 10214 – 9455 x 100 = 8% 9455 The % change was maximum for B.

158. 4864 – 2579 = 2285

159. 5000 x 100 = 10.3% =10% 48640 160. 10700 x100 = 125% 8596

161. In 36 days 12 men can do 1 complete work. In 36 days 12 women can do ¾ th of the work. Since time and the no. of persons is the same in both cases. 1 women’s daily work = ¾th of 1 men’s daily work\ 8 women’s daily work = ¾ x 8 = 6 men’s daily work (10) men + 8 women daily work) = (10 men + 6 men) = 16 men’s daily work. 12 men can do the work in 369 days :. 16 men can do the work in 36 x 12 = 27 days 16

The information provided here is only for refrence. It may vary from the original. www.recruitment.guru

162. Increase is interest in 3 years due to increase in rate by 4% = 800 x 3 x 4 = Rs. 96 100 Total amount at the end of 3 years = Rs . 956 + Rs. 96 = rs. 1052 163. 427398 = 15 x 28493 +3 :. The least number which should be subtracted from 427398 so that it becomes divisible by 15 = 3.

164. Initial speed = 80 km/hr Total distance = 80 x 10 = 800 km. Now speed =800 =200 km/hr 4 Increase in speed = 200 – 80 = 120 km/hr.

165. 33 1% of 768.9 +25% of 161.2 – 58.12 3 = 100 x 788. 9 + 25 x 161.2 -58.12 3x100 100 = 256.3 + 40.3- 58.12 = 238.48 = 240 (approx.) 166. Profit = Selling price of 4 notebooks Cost price = selling price of (12 -8) = 4 notebooks. :. %profit = 4 x 100 =50 8 167. 3 years ago Ritu’s age = x :. Ritu’s present age =x + 3 Rahul’s present age = Ritu’s present age -5 = x + 3 -5 = x – 2 168. Mixture : 2 kg of rice at Rs. 15 kg + 3 kg of rice at Rs. 13kg Total weight = 2 + 3 = 5 kg Total cost price = (2 x 15) + (3 x 13) = 30 + 39 = Rs. 69 Cost price per kg of the mixture = 69 = Rs. 13.80 5 Selling price to get 33 1 % profit 3 100 + 33 1 3 = x Rs. 13.80 = 400 x Rs .13.80 3 x 100 = 4 x Rs. 13.80 3 =Rs. 18.40 169. Total time = 6 hours. Speed of the both in still water = 4 km/hr let the distance between M and N be D and the speed of the stream be x. D 1 + 1 = 6 4+ x 4 - x

The information provided here is only for refrence. It may vary from the original. www.recruitment.guru

D 4 – x + 4 + x = 6 (x + x) (4- x)

D 8 = 6 42 - x2

8D = 6 16 – x2 D = 6 (16 –x2 ) = 3 (16 –x2) 8 4 Since the speed of the stream (x) is not given, the distance D cannot be determined.

170. Let Mr. Yadav’s annual salary be x. Amount spent on consumables = 0.80x Amount spent on clothes and transport = 0.50 (x-0.80x) = 0.50 x 0.20x = 0.10x Savings = x – 0.80x – 0.10x = 0.10x :. 0.10x = Expenditure on clothes and transport = Rs. 5370 :. Monthly expenditure =5370 12 = Rs. 447.50

171. I. p2 + 24 = 10p p2 - 10p + 24 = 0 p2 – 6p -4p +24 =0 p(p-6) -4 (p – 6) = 0 (p -6) (p – 4) =0 P = 4, 6 II. 2p2 + 18 = 12q q 2 + 9 = 0 q2 – 6q + 9 =0 (q -3)2 =0 q = 3 Thus, p > q

172. I. pq + 30 = 6p+ 5q 6q + 5q – pq = 30 p + q - pq = 1 5 6 20

P + q (p + q) = 1 5 6 5 6 :. P = 5 and q =6 q > p

173. I. q2 + q = 2 q2 + q - 2= 0 q2 - q + 2q – 2 =0 q(q-1) + 2(q-1) =0 (q-1) (q+2) = 0

The information provided here is only for refrence. It may vary from the original. www.recruitment.guru

q = -2, 1 p2 +7p + 10= 0 p2 + 5p + 10 =0 p2 + 5p + 2p +10 =0 II. p(p+5) +2(p+5) = 0 (p+5) (p+2)=0 P = 5, -2 Thus, q > q

174. I. p2 + 16=8p p2 - 8p + 16 = 0 (p-4)2 =0 P =4 II. 4q2 + 64 = 32q q2 + 16 = 8q q2 -8q + 16 = 0 (q -4)2 =0 q =4 Thus, p=q

175. 2p2 + 12p + 16 =0 p 2 +6p +8 =0 p2 +2p + 4p +8=0 p(p+2) +4(p+2)=0 (p+2)(p+4)=0 P = -2, -4 II. 2q2 + 14q +24 =0 q2 +7q +12=0 q2 +3q +4q +12=0 q(q+3)+4(q+3) =0 (q +3)(q+4) =0 q=-3, -4, Thus,, p> q

The information provided here is only for refrence. It may vary from the original.